Pharma Superexam

April 3, 2018 | Author: Velasco John | Category: Receptor Antagonist, Chemotherapy, Acetylcholine, Enzyme Inhibitor, Anesthesia


Comments



Description

PHARMACOLOGY Superexam TOPNOTCHStudy online at quizlet.com/_34okv8 1. 1. Which of the following medications is NOT contraindicated in a patient 81 A. Medscape expect questions on never before taking RANOLAZINE? a. Losartan b. Itraconazole c. clarithromycin d. heard drugs. metronidazole e. prednisone 2. 2. Phoebe Marie, G2 P1 (1001), recently traveled to Palawan and is taking 82 A. Halofantrine is not used for chemoprohylaxis Quinine for malaria. Which of the following medications can she take? a. because of its potential for quinidine like clindamycin b. doxycycline c. primaquine d. Halofantrine e. all of the cardiotoxicity (QT prolongation) and embryotoxicity. above Likewise, doxycycline and primaquine are contraindicated during pregnancy. Katzung Pharmacology Examination and Board Review 10th edition p.453 3. 3. Future clinicians may screen every patient for a variety of individual 683 C. Answer: C. PharmaCogenetiCs (pp. 3 and 82 differences in drug response. Differences in the rate of acetylation of [.pdf file] Katzung Pharmacology, 9th edition) isoniazid among groups of individuals exemplifies A. Pharmacodynamics B. Posology C. Pharmacogenetics D. Biochemorphology 4. A 3/M was brought to the hospital for decreased sensorium. On further 366 B. Antidotes for ethylene glycol or methanol history, the child accidentally ingested a mouthful of antifreeze. You then poisoning include ethanol and fomepizole. Although proceed to give the child which antidote? A. Disulfiram B. Fomepizole C. ethanol is effective, it is difficult to achieve safe and Famotidine D. Methanol E. There is no antidote effective blood levels that is why fomepizole is preferred. Katzung 12th ed pg 1037 5. 3. Which of the following drugs is a reversible acetylcholinesterase 83 C. Memantine - low to moderate affinity inhibitor that causes an increase in concentrations of acetylcholine and is uncompetitive NMDA receptor antagonist blocking metabolized by cholinesterase and excreted mainly by the kidneys and receptor only under conditions of excessive used in Alzheimer's disease? a. Memantine b. Galantamine c. Rivastigmine stimulation without effect of normal d. Donepezil e. none of the above neurotransmission Donepezil - acetylcholinesterase inhibitor but metabolized by hepatic P - 450 enzymes Galantamine - increases acetylcholine from surviving presynaptic nerve terminals by modulating nicotinic Ach receptor Medscape 6. 4. Which of the following is the mechanism why Cefuroxime is not used for 84 C. meningitis? a. poor penetration to CSF b. levels achieved in CSF not http://pedsinreview.aappublications.org/content/2 adequate to sterilize it c. potential for delayed sterilization and 9/8/264.full therapeutic failures d. presence of efflux pumps e. none of the above 7. A 4 year old child, febrile at 38deg. C was given Paracetamol at 15mkd. 476 C. Paracetamol or acetaminophen selectively What is the mechanism of action of this drug? A. COX-2 selective inhibitor inhibits COX-3. COX-2 selective inhibitors are coxib B. IL-1 and IL-6 inhibitors C. COX-3 selective inhibitor D. Irreversible COX-1 such as celecoxib. Irreversible COX-1 and 2 inhibitor and COX-2 inhibitor E. TNF - α inhibitor is aspirin. TNF - α inhibitors are infliximab, adalimumab and etanercept. 8. 5. Which of the following is the drug topically used for acne? a. allylamine 85 B. Goodman Gilman 12th edition. Dermatology b. clindamycin c. mupirocin d. bacitracin e. none of the above section 9. A 5 year old boy with Trisomy 21 was diagnosed to have acute 638 D. Asparaginase, depletes serum asparagine. It is lymphoblastic leukemia. What is the most important drug in the the main and the most impt drug in the management therapeutic regimen of this disease entity? A. Rituximab B. Cytarabine C. od ALL. Idarubucin D. Asparaginase E. Prednisone 10. A 5 year old known case of seizure disorder maintained on phenytoin was 583 C. Phenytoin is an inducer, which will lower rushed to the ER due to an asthma attack. IV theophylline was given to theophylline levels. Asthma may not be adequately control the asthma using usual dose computations. What are the treated with this monotherapy. consequences of the drug-drug interactions? A. The patient will be more prone to the toxic side-effects of theophylline. B. The patient may be at risk for a seizure attack. C. The patient is not being treated adequately for his asthma. D. There will be an increased in plasma levels of phenytoin. E. No drug interactions exist beween theophylline and phenytoin 11. A 6/F is brought to the clinic due to frequent episodes of sudden activity 550 B. Ethosuzimide and valproic acid are the arrest with staring and minimal eyelid flutter occurring for 10 - 20 preferred drugs for absence seizures because they seconds, 5 - 10 times per day. During such episodes, patient is cause minimal sedation. unresponsive to voice or tactile stimulation. She is diagnosed with absence seizure. Which of the following antiseizure drugs is most appropriate? A. Carbamazepine B. Valproic acid C. Phenytoin D. Phenobarbital E. Levetiracetam 12. 6. Which of the following anti folate drugs is a blood schizonticide? a. 86E Katzung Pharmacology Examination and Board Atovaquone b. Artesunate c. Halofantrine d. Doxycycline e. Review 10th edition p.453 Pyrimethamine 13. A 6-year-old patient was recently diagnosed with Cat I PTB. DOTS 1 B. Know the doses of anti-TB drugs for BOTH Treatment is initiated for this patient. The dosing should be: A. INH children and adults. Adults: INH 5mg/kg + Rif 10mg/kg 5mg/kg + Rif 10mg/kg + Pyz 25mg/kg + Eth 15mg/kg B. INH 10mg/kg + Rif + Pyz 25mg/kg + Eth 15mg/kg Generally just make 15mg/kg + Pyz 35mg/kg + Eth 20mg/kg C. INH 15mg/kg + Rif 10mg/kg + sure you are familiar with TB! It can come out over Pyz 35mg/kg + Eth 15mg/kg D. INH 25mg/kg + Rif 10mg/kg + Pyz and over in different subjects from pharma, micro, 25mg/kg + Eth 11mg/kg E. INH 5mg/kg + Rif 20mg/kg + Pyz 15mg/kg + pedia, im, patho, even gyne. Source: National TB Eth 10mg/kg Control Program Manual of Procedures 14. 7. Which of the following is a long acting insulin? a. Lantus b. Lente c. 87 C. D and E - short acting A and B - intermediate Glargine d. Detemir e. Glulisine acting Katzung Pharmacology Examination and Board Review 10th edition p.363 15. A 7 year-old male child with nephritic syndrome on steroid therapy was 696 B. Answer: B After susceptible person is exposed exposed to a sister with measles. You would administer: A. Measles to measles, either measles vaccine given within 72 hrs vaccine B. Human immunoglobulin C. Antisera D. Vitamin A or immune globulin given within 6 days can prevent or modify disease. However, px has nephritic syndrome & is immunocompromised so an active measles vaccine may even be harmful. 16. 7 year old male was noted to have difficulty sustaining attention in tasks 198 B. Methylphenidate is the drug of choice for or play activities in school and at home. He often leaves his seat inside ADHD in children 6 years old and above. Mechanism the classroom and runs about and climbs excessively. What is the of action is unknown but it mainly acts as a CNS treatment of choice for this case? A. sibutramine B. methylphenidate C. stimulant similar to amphetamines. diazepam D. behavioral therapy E. magnesium pemoline 17. A 7 yo male was brought to ER secondary to symptoms suggestive of 222 D. Botulinum toxin inhibits the release of vesicular botulinum toxin ingestion, which of the following are likely included in the contents from all types of cholinergic nerve endings. symptoms of the patient? A. Muscle fasciculation B. Bronchoconstriction Since both ganglionic/autonomic and peripheral C. Increased bowel movement D. blurred near vision E. hypertension motor synapse are affected. Patient will present with hypotension and lack of muscular contraction. Blurred near vision is a result of paralysis of ciliary muscle aka cycloplegia 18. 8. Which of the following is NOT true of Gentamicin? a. killing action 88 B. has greater efficacy when administered as a continues when plasma levels decline below measurable levels b. lesser single large dose than when given as multiple smaller efficacy when administered as a single large dose c. in vivo efficacy not doses Katzung Pharmacology Examination and Board directly related to time above MIC d. as plasma level is increased above Review 10th edition p.399 MIC, it kills an increasing proportion of bacteria at a more rapid rate e. toxicity depends on both critical plasma concentration and the time that such level is exceeded 19. An 8-year-old being treated with a combination of chemotherapeutic 492 A. Major toxicities of methotrexate include agents had difficulty eating and drinking due to red, inflamed sores in gastrointestinal mucositis, bone marrow suppression, her mouth and esophagus. Which of the following antineoplastic agents is skin erythema and hepatic dysfunction. SIMILAR TO the most likely etiology? A. Methotrexate B. Vinblastine C. Doxorubicin D. PREVIOUS BOARD EXAM CONCEPT/PRINCIPLE Prednisone 20. 9. Which of the following chemotherapeutic drugs is used for bladder 89 B. Medscape expect questions on never before cancer (intravesical administration) and control of malignant pericardial/ heard drugs. pleural/ peritoneal effusions (intracavitary)? a. cyclophosphamide b. thiotepa c. carmustine d. lomustine e. none of the above 21. A 9-year old Filipino child was brought to you in the clinics by her 413 A. Valproate is the DOC for Absence Seizures, most mother due to an observed blank-stare episodes noted by her especially in our setting here in the Philippines, Ethosuximide mother which lasts from a few seconds to a few minutes. Which of is NOT commercially available. the following medications is the most suitable for this patient? A. Valproate B. Phenytoin C. Phenobarbital D. Carbamazepine E. Lorazepam 22. A 10/F, diagnosed case of HSV encephalitis, was given IV 559 A. Acyclovir, foscarnet and ganciclovir are DNA acyclovir. Acyclovir is an efffective antiviral because it inhibits: A. polymerase inhibitors. DNA polymerase B. RNA polymerase C. Aspartate protease D. Neuraminidase E. Hemagluttinin 23. 10. Which of the following is a mixed FSH and LH agonist and is 90 A. B - FSH only C - LH only D - GH agonist E - GnRH used to stimulate gonadal function including spermatogenesis and agonist Katzung Pharmacology Examination and Board ovulation? a. menotropin b. urofollitropin c. Lutropin d. Review 10th edition p.327 Somatropin e. Gonadorelin 24. A 10 year old child, known asthmatic had episodes of nocturnal 475 E Salbutamol is the druig of choice for acute asthma asthma attacks approximately 2-3x/week. In order to prevent attack, while aminophylline is used for prophylaxis against subsequent attack, what drug is used for prophylaxis against nocturnal attacks. nocturnal attack? A. Salbutamol + Ipratropium B. Salbutamol C. salmeterol D. cromolyn E. Aminophylline 25. A 10 year-old child with asthma was prescribed with a drug which 460 C. Montelukast blocks leukotriene receptor for inhibits binding of leukotriene D4 to its receptor on target tissues. leukotrienes C4, D4, E4 preventing airway inflammation and The drug prescribed is: A. Zileuton B. Ipratropium C. Montelukast bronchoconstriction in asthmatic patients. D. Omalizumab E. Theophylline 26. 11. Which of the following is not likely used for treatment of 91 D. D is used for CMV Katzung Pharmacology Examination Herpes Simplex virus? a. Acyclovir b. Penciclovir c. Famciclovir d. and Board Review 10th edition p.429 Ganciclovir e. none of the above 27. 12. A neonate in the NICU was observed by the Neonatologist 92 D. fellow to have epicanthal folds, low nasal bridge, short palpebral fissures, flat midface, and indistinct philtrum. Which of the following medications did the mother likely take during pregnancy? a. phenytoin b. carbamazepine c. valproic acid d. ethyl alcohol e. warfarin 28. A 12-year-old boy presents with a rash on the palms and soles of 39 D. Doxycycline, a tetracycline, is the antibiotic of choice his feet as well as fever and headache. He was camping last to treat Rocky Mountain spotted fever, a rickettsial disease. weekend and admits to being bitten by a tick. His Weil-Felix test Streptomycin can be used to treat plague and brucellosis. result is positive, suggesting Rocky Mountain spotted fever. What Bacitracin is only used topically. Ciprofloxacin can be used antibiotic should be given? A. Streptomycin B. Bacitracin C. to treat anthrax, and erythromycin is the most effective drug Ciprofloxacin D. Doxycycline E. Erythromycin for the treatment of Legionnaires disease. 29. 13. which of the following is the chemoprophylactic drug of choice 93 B. http://www.who.int/ith/ITH_chapter_7.pdf for pregnant patients visiting falciparum malaria endemic places? a. chloroquine b. mefloquine c. doxycycline d. atovaquone - proguanil e. artemether - lumefantrine 30. 14. Which of the following is NOT true of phenytoin? a. metabolism 94 D. metabolism is enhanced by presence of inducers of is nonlinear b. drug binds extensively to proteins in plasma c. liver metabolism (phenobarbital, rifampin) and inhibited by fosphenytoin in the water soluble prodrug for parenteral other drugs (cimetidine, isoniazid) Katzung Pharmacology administration d. metabolism is induced by isoniazid e. drug of Examination and Board Review 10th edition p.217 choice for grand mal seizures 31. A 14 year old child was diagnosed with Schizophrenia, which of the 408 B. following is the only antipsychotic approved for pediatric patients? A. Clozapine B. Risperidone C. Quetiapine D. Olanzapine E. Thioridazine 32. 15. A 54 year old male presents to the clinic with a circular lesion on 95 B. acedapsone is the repository form that provides the arm and anesthesia on pin prick of the center of the lesion. Patient inhibitory plasma concentrations for several months. It is was then brought to Cebu Skin Clinic for further evaluation. Dr. AM the most active drug for leprosy. Mechanism of action is impression was leprosy tuberculoid type and decides to start the inhibition of folic acid synthesis. It is contraindicated to patient with antimycobacterials. Which of the following is true of G6PD patients. Katzung Pharmacology Examination and Dapsone? a. fluorodapsone is the respository form b. it is used as an Board Review 10th edition p.415 alternative drug for treatment of PCP pneumonia c. it is not the most active drug for Leprosy d. mechanism of action is to inhibit arabinosyl transferase e. can be given to G6PD patients 33. 15 units of Drug X was given to a patient. If this drug undergoes first- 104 C. Page 4 of Topnotch Handout. In a drug order elimination how many half-lifes will it take for the drug undergoing first-order elimination concentration concentration to reach 1.88 units: A. 1 B. 2 C. 3 D. 4 E. 5 decreases by 50% for every half-life. SO if the drug was 15 units at the start, 1st half life- 7.5, 2nd half life 3.75, third half life 1.875. 34. A 15 year old post partum girl in Fabella hospital was bleeding 74 B. SIMILAR TO PREVIOUS BOARD EXAM profusely after giving birth despite oxytocin administration, what CONCEPT/PRINCIPLE. Methylergonovine can be given class of drug can you give as second line treatment? A. 5-HT1 agonist which is a 5-HT2 receptor antagonist selective to the B. 5-HT2 antagonist C. 5-HT3 antagonist D. 5-HT4 antagonist E. Alpha uterus. 1 agonist 35. 16. DC, a 69 year old female, presented with resting tremors and 96 B. Katzung Pharmacology Examination and Board shuffling gait. Upon cleaning her bathroom, she accidentally damaged Review 10th edition p.249 the bathroom sink thus prompting consult. Having a good understanding of the pathophysiology of Parkinson’s disease, Dr. MC starts the patient with dopamine agonists. Which of the following can be given as monotherapy for mild parkinsonism and has high affinity for D3 receptor? a. bromocriptine b. pramipexole c. ropinirole d. apomorphine e. cabergoline 36. 17. Which of the following is the regimen for a 29 year old male with a 97 E A - hodgkin’s lymphoma B - ovarian C - testicular mass? a. ABVD regimen b. paclitaxel + carboplatin c. pancreatic D - colorectal Katzung Pharmacology gemcitabine + erlotinib d. fluorouracil and leucovorin + oxaliplatin e. Examination and Board Review 10th edition p.468 PEB regimen 37. A 17-year-old boy is brought to the emergency department by his 526 D. parents because of bizarre behavior for 6 hours. Last night he was out with friends, and since returning, he has been confused and has "trashed" his room. His blood pressure is 165/95 mm Hg. He is hypervigilant, has little spontaneous speech, and is disoriented to place and time. He appears catatonic but abruptly becomes assaultive two times and needs to be restrained. Which of the following is the most likely substance taken? A) Cocaine B) Ecstasy C) LSD D) PCP 38. A 17-year-old girl sees her physician for swollen lymph nodes in the 34 E ABVD is a treatment regimen used for Hodgkin supraclavicular region. A core biopsy demonstrates disease and includes adriamycin, bleomycin, vinblastine, Reedâ€"Sternberg cells and fibrotic bands, a finding characteristic of and dacarbazine. R-CHOP is used for treating non- nodular sclerosis Hodgkin disease. Which of the following combined Hodgkin’s disease. CMF, or cyclophosphamide, regimens might be used in this patient? A. R-CHOP B. CMF C. FOLFOX methotrexate, and fluorouracil, is used for breast cancer. D. BEP E. ABVD FOLFOX, a regimen that uses 5-fluorouracil, oxaliplatin, and leucovorin, is used in the treatment of colon cancer. BEP (bleomycin, etoposide, and platinum [cisplatin]) is used in the management of metastatic testicular neoplasms. 39. A 18/M previously diagnosed with epilepsy and maintained on 372 B. Memorize inducers & inhibitors VERY HIGH YIELD phenobarbital seizure free for the past 2 years, presented to you with a 3 month history of cough, recurrent fever and weight loss. AFB was positive on two occasions and CXR show cavitations on the upper lung fields. You conclude that he has PTB and you start him of HRZE. After a few days, you were informed that you patient was in the ER and their working impression is status epilepticus. Which of the following drugs could have caused the breakthrough seizures? A. Isoniazid B. Rifampicin C. Pyrazinamide D. Ethambutol E. None of the above 40. 18. NT a 39 year old male with adult polycystic kidney disease 98 A. Cyclosporine binds to cyclophilin and tacrolimus underwent kidney transplant. Which of the following immunophilin binds to FKBP. Both complexes inhibit calcineurin, a inhibitors bind to FK - binding protein 12 and inhibit the mTOR cytoplasmic phosphatase. Calcineurin regulates the ability pathway which inhibits the T - cell proliferation response to IL - 2? a. of the nuclear factor of activated T cells to translocate to sirolimus b. tacrolimus c. cyclosporin d. cyclophiline e. none of the the nucleus and increase the production of key cytokines above such as IL 2, 3, and IFN gamma. Cyclophilin and tacrolimus prevent the increased production of cytokines that normally occurs in response to T cell receptor activation. Katzung Pharmacology Examination and Board Review 10th edition p.482 41. 19. A 23 year old female came in for unilateral throbbing headache 99 C. Ergot alkaloids can produce marked and prolonged associated with nausea and photophobia. Which of the following alpha receptor mediated vasoconstriction. Ergotamine is ergot alkaloids has no effects on the dopamine receptor D2 but the prototype and has been a mainstay of treatment of stimulates the uterine smooth muscle? a. bromocriptine b. acute attacks of migraine. Katzung Pharmacology ergonovine c. ergotamine d. LSD e. methylergometrine Examination and Board Review 10th edition p.160 42. 20. Which of the following prostaglandin derivatives increases 100 E Latanoprost increases outflow of aqueous humor outflow of aqueous humor? a. misoprostol b. alprostadil c. and reduces intraocular pressure in glaucoma. Katzung dinoprostone d. epoprostenol e. none of the above Pharmacology Examination and Board Review 10th edition p.176 43. A 20 year old male develops tachycardia, hypotension, muscle 347 C. This is a case of malignant hyperthermia, which is rigidity, and spiking fever immediately after start of surgery. Which associated with the combination of inhaled anaesthetics of the following are commonly associated with this anaesthetic and succinylcholine. complication? A. IV anesthetic and succinylcholine B. IV anesthetic and midazolam C. Inhalational anesthetic and succinylcholine D. Inhalational anesthetic and midazolam 44. A 21/M presented with recurrent fever, night sweats, enlarged 375 A. ABVD (doxorubicin, bleomycin, vinblastine, and abdomen and generalized lymphadenopathy. Biopsy of a lymph dacarbazine) and MOPP (mechlorethamine, vincristine, node showed Reed-Sternberg cells. Which of the following procarbazine, and prednisone) have been used to treat chemotherapeutic regimen/s can be given to the patient? A. ABVD B. Hodgkin's lymphoma. ABVD has supplanted MOPP as the CHOP C. MOPP D. Rituximab E. A and C regimen of choice since it is more effective and less toxic. CHOP and Rituximab are used for NHL Katzung 12th ed pg 970 45. A 23-year-old presents with abdominal pain, tenesmus, and mucoid 485 A. Ciprofloxacin is the recommended empiric diarrhea with frank blood. Which is the best empiric treatment? A. treatment for patients with acute dysenteriae. SIMILAR TO Ciprofloxacin B. Metronidazole C. Amoxicillin D. Cotrimoxazole PREVIOUS BOARD EXAM CONCEPT/PRINCIPLE 46. A 23 y/o female previous RHD patient s/p valve replacement is 118 A. Page 39 of Topnotch Handout. CYP 450 inhibitors being maintained on warfarin. 3 days prior she had symptoms of PUD prolong/increase warfarin's anticoagulant effectsince they for which she self medicated. She now presented at the ER due to inhibit drug clearance. Cimetidine is a CYP450 inhibitor. black tarry stools, epigastric pain, coffee ground emesis, gum bleeding and dizziness. Laboratory showed prolonged PT. Which of the following drugs caused a drug interaction with warfarin prolonging its anticoagulant effect? A. Cimetidine B. Omeprazole C. Bismuth D. Sucralfate E. Maalox 47. A 24/G3P2(2002) who came in with a BP of 200/140 was given Hydralazine. What is the 545 C. A describes the MOA of mechanism of action of hydralazine? A. Reduction of Ca influx via L-type channels B. verapamil; D describes minoxidil; E Antagonism of alpha-adrenergic receptors in blood vessels C. Release of NO D. describes fenoldopam. All are direct Hyperpolarization of smooth muscles due to opening of K channels E. Activation of vasodilators. dopamine D1 receptors 48. A 24 year old female G1P0, 12 weeks AOG, sought consult due to purulent vaginal 293 E Cephalosporins should be discharge and dyspareunia. Gram stain of the vaginal discharge showed intracellular avoided in patients with history of gram negative diplococci. The patient disclosed that she had a severe allergic reaction severe allergic reaction to penicillin. to amoxicillin a year ago. Which of the following is the safest antibiotic for this case ? A. FQ and doxycycline are avoided in Doxycyline B. Ciprofloxacin C. Ceftriaxone D. Gentamicin E. Azithromycin pregnancy since it can cause cartilage and bone damage. Azithromycin, a macrolide antibiotic is the safest drug for this case. 49. A 24 year old female is taking oral contraceptives as family planning method. Which of 256 C. SIMILAR TO PREVIOUS BOARD the following drugs is considered a CYP450 enzyme inducer which can cause rapid EXAM CONCEPT/PRINCIPLE, what metabolism of other drugs such as oral contraceptives which can potentially decrease came out as an enzyme inducer in ours their effectiveness? A. isoniazid B. trimethoprim sulfamethoxazole C. griseofulvin D. was phenobarbital, griseofulvin is an Amiodarone E. ketoconazole inducer all the others are inhibitors, memorize at least the most common of them inducers and inhibitors, topnotch handouts are good reference 50. A 24-year-old student has been taking over-the-counter diphenhydramine for her 22 A. Diphenhydramine blocks H1- allergy symptoms most of her life. Lately, however, she has had more frequent receptors in the brain, thereby symptoms, so he increased the dose of the medication. She now asks her friend, who is a producing sedation. The release of medical student, to explain to her how exactly this agent makes her more sleepy lately. dopamine and serotonin is modulated What is the most likely answer regarding diphenhydramine? A. It blocks H1-receptors in via H3- receptors. Diphenhydramine the brain B. It modulates the release of dopamine and serotonin C. It acts peripherally, readily crosses the bloodâ€"brain since it does not cross the bloodâ€"brain barrier D. It exerts its effects via muscarinic- barrier. This agent has cholinergic agonist activity E. It contains tryptophan, which produces sedation muscariniccholinergic agonist properties. It is not known to contain tryptophan. 51. A 25 year-old farmer was brought to the emergency room after having allegedly drank 445 E Pralidoxime is a cholinesterase a poison. PE revealed that he had constricted pupils, excessive salivation and sweating, regenerator, an antidote for wheezing and had several bouts of vomiting and diarrhea. He was given antidotes organophosphate poisoning. Atropine and Pralidoxime. Pralidoxime exert its effects by: A. Destruction of the enzyme Pralidoxime is an example of chemical B. Competitive antagonism C. Noncompetitive antagonism D. Physiologic antagonism E. antagonism. Chemical antagonism is Chemical antagonism when an antagonist directly interacts with the drug being antagonized to remove it or prevent it from reaching its target. 52. 25 year old female came in with chief complaint of cough of 3 weeks duration, this was 241 B. SIMILAR TO PREVIOUS BOARD accompanied by night sweats, weight loss, body malaise. PE revealed slightly pale EXAM palpebral conjunctiva, multiple bilateral posterior cervical lymphadenopathies but with CONCEPT/PRINCIPLE.,Considered as no adventitious breath sounds on bilateral lung fields. The rest of the physical the most active anti-TB drug, in fact the examination was unremarkable, which of the following drugs is considered to be the only drug given to treat latent most active for her condition? A. Rifampicin B. Isoniazid C. Pyrazinamide D. Ethambutol tuberculosis E. streptomycin 53. A 25-year-old woman with AIDS comes to the physician because of a 10-day 539 D. history of fever, shortness of breath, night sweats, fatigue, and a non- productive cough. She had been treated with antiretroviral agents for 2 years but stopped taking her medications 6 weeks ago. She has no known allergies. Her temperature is 38.6 C, pulse is 110/min, respiration are 20/min, and blood pressure is 110/60 mm Hg. Examination shows no cyanosis. Diffuse fine crackles are heard bilaterally. An x-ray of the chest shows bilateral interstitial infiltrates. A silver stain of sputum is positive for cysts and organisms. Which of the following is the most appropriate pharmacotherapy? A) Ceftriaxone B) Erythromycin C) Cotrimazole D) Co-trimoxazole 54. A 26/M presented with RUQ pain and fever. Imaging showed an abscess and 361 E For cases of amebic hepatic abscess you aspiration biopsy was done. The aspirate resembled anchovy paste in color can give either Metronidazole, 750 mg 3 times and in consistency. What is/are the possible drug/s that you can give for this daily (or 500 mg IV every 6 hours) for 10 days or patient? A. Metronidazole B. Tinidazole C. Chloroquine D. A and B E. All of Tinidazole, 2 g daily for 5 days PLUS a luminal the above agent. Alternatively you can give Dehydroemetine2 or emetine,2 1 mg/kg SC or IM for 8â€"10 days, followed by (liver abscess only) chloroquine, 500 mg twice daily for 2 days, then 500 mg daily for 21 days PLUS a luminal agent Luminal agent (Diloxanide, iodoquinol, paramomycin) Katzung 12th ed pg 928 55. 27-year old female presents to the ER approximately one hour after 62 C. The physiological antagonism is related to swallowing 70 propranolol tablets with suicidal intent. At the time of the fact that glucagon activates cAMP through assessment she is drowsy (GCS 13) with a heart rate of 46 bpm and BP non- adrenergic pathways. Because the 100/60. What drug/class can be given by virtue of physiologic antagonism? adrenergic pathways are blocked by propanol, A. Beta agonists B. Alpha agonists C. Glucagon D. Diazepam E. Atropine glucagon essentially uses a back door to enhance myocardial activity. 56. A 27-year-old nulligravid woman comes to the physician for 540 C. preconceptional counseling. She has a mechanical mitral heart valve and chronic rheumatoid arthritis. Her cardiac status is New York Heart Association Class II. She feels well. Current daily medications include warfarin, prednisone, and acetaminophen with codeine. Examination shows no abnormalities except for audible clicking from the heart valve. Which of the following is the most appropriate advice for this patient? A) Chemical dependency counseling before pregnancy B) Discontinuation of anticoagulant therapy during pregnancy C) Discontinuation of prednisone during pregnancy D) Switching from warfarin to heparin before pregnancy 57. A 28/M car mechanic was brought to the emergency room after ingesting 548 E Ethylene glycol and methanol poisoning automobile antifreeze. ABG reveals high anion gap metabolic acidosis; while may be treated with fomepizole, an alcohol urinalysis reveals calcium oxalate crystals. You suspect ethylene glycol dehydrogenase inhibitor, which prvents poisoning. Which of the following drugs is recommended? A. Naltrexone B. conversion of methanol/ehtylene glycol to toxic Diazepam C. Disulfiram D. Acamprosate E. Fomepizole metabolites. 58. A 28 year-old woman wanted to get pregnant. A substance that enhances 452 C. Clomiphene citrate enhances probability of probability of ovulation by blocking the inhibitory effect of estrogen and ovulation by blocking the inhibitory effect of thus stimulating the release of gonadotrophin from the pituitary is: A. estrogen and thus stimulating the release of Tamoxifen B. Danazol C. Clomiphene D. Progesterone E. Oxytocin gonadotrophin from the pituitary. 59. A 29 year old woman presented with amenorrhea, galactorrhea and loss of 282 A. Bromocriptine is an effective dopamine sexual libido. On evaluation her serum prolactin was noted to be markedly agonist in the CNS with the advantage of oral elevated. Which of the following medications is most useful in the treatment activity. The drug inhibits prolactin secretion by of her condition? A. Bromocriptine B. Cimetidine C. Sumatriptan D. activating pituitary dopamine receptors Ergotamine E. Ondansetron 60. A 30 year old female with bipolar disorder delivered a baby with Ebstein 345 B. anomaly. The drug that the mother was taking during her pregnancy that caused this congenital anomaly is: A. Olanzapine B. Lithium C. Carbamazepine D. Risperidone 61. A 30-year-old woman has suffered from cyclical migraines for many years. 38 B. Sumatriptan is a 5-HT1D agonist. An She now presents to her physician asking for a medication designated example of an agent known as a 5-HT1A agonist specifically for migraines, not just a general pain reliever. Her physician would be buspirone, an antianxiety agent. decided to prescribe sumatriptan as a trial medication. The patient, who is a Fluoxetine is an example of a serotonin-reuptake biochemist, would like to know how this medication works. A. It is a 5-HT1A inhibitor. Ondansetron, an antinausea medication, agonist B. It is a 5-HT1D agonist C. It blocks reuptake of serotonin D. It is a 5- is a 5-HT3 antagonist. The antipsychotic HT3 antagonist E. It is a 5-HT2A antagonist medication Risperdal is an example of a 5-HT2A antagonist. 62. A 30 y/o male patient developed methemoglobinemia after receiving 119 B. Page 52 of Topnotch Handout. For prilocaine during a minor surgery. What should be administered to this methemoglobinemia the antidote is methylene patient? A. NAC B. Methylene blue C. EDTA D. Dimercaprol E. Deferoxamine blue. 63. A 32 year old G2P1 mother delivered a live term baby Boy with an upturned 478 A. The physical description of the neonate is nose, mild midfacial hypoplasia, long upper lip, and lower distal digital consistent with fetal hydantoin syndrome which is hypoplasia. Upon review of maternal history, the mother was taking an anti- the side effect of phenytoin epileptic drug. The drug that is most likely used was: A. phenytoin B. valproic acid C. diazepam D. carbamazepine E. lamotrigine 64. A 32-year-old woman comes to the physician because of a 3-month history of 530 C. increasing pain and stiffness in her wrists, hands, and ankles. During this period, she also has had progressive fatigue and morning stiffness lasting 2 hours. She has a 1-year history of rheumatoid arthritis treated with naproxen. Examination shows redness, swelling, and warmth over the wrist, hand, and ankle joints bilaterally. There are nontender subcutaneous nodules over the extensor surfaces of both elbows. X-ray films of the hands show diffuse osteopenia and erosions over several of the distal metacarpal bones. Which of the following is the most appropriate pharmacotherapy? A) Add oral cyclophosphamide B) Add oral gold C.) Add oral Methotrexate D.) Add Oral Penicillamine 65. A 33-year old farmer, after 1 hour of spraying insecticides, started to note 614 A. The patient had an insecticide poisoning. generalized body weakness, drowsiness, shortness of breath, numbness and Insecticides usually contain organophosphates or severe epigastric pain. After 10 hours, he was brought to the ER due to loss of carbamates. Before 6 hours, pralidoxime and consciousness. He was soaking in sweat, drooling with saliva and diarrheic. atropine should be both administered. In this What should be administered to this patient? A. Atropine B. Pralidoxime C. patient, 10 hours has already elapsed, hence Neostigmine D. A and B E. B and C pralidoxime can no longer be effective. 66. A 34/F asthmatic, scheduled to undergo cholelithiasis, was being induced by 369 C. The patient is most likely experiencing the anesthesiologist. After starting the volatile anesthetic, the bronchospasm and although all volatile anesthesiologist noted increased peak air pressure, prolonged expiration with anesthetics possess varying degrees of the classic "shark fin" capnograph. Which of the following most likely have bronchodilating properties airway irritation caused the above findings? A. Halothane B. Sevoflurane C. Desflurane D. induced by the pungency of isoflurane and Nitrous oxide E. None of the above desflurane may induce bronchospasm. These rarely occur with halothane, sevoflurane and nitrous oxide since they are non pungent. Katzung 12th ed pg 436 67. A 34/F came in to the ER complaining of severe eye pain. On PE you notice 379 C. All of the choices can decrease IOP and that the cornea is steamy with accompanying conjunctival injection. You are used in the treatment of glaucoma. Beta proceed to administer a drug that will cause an increase in the outflow of blockers, osmotic agents, alpha-2 agonists and aqueous humor through the canal of Schlemm resulting in a decrease in IOP. carbonic anhydrase inhibitors decrease secretion Which drug was administered? A. Timolol B. Apraclonidine C. Latanoprost D. of aqueous humor from the ciliary epithelium. Pilocarpine E. Epinephrine Cholinomimetics cause contraction of the ciliary muscle causing the trabecular meshwork to open, increasing outflow. Prostaglandins increase outflow through the canal of Schlemm. Nonselective alpha agonists increase outflow via the uveoscleral veins. Topnotch 68. A 34/F was diagnosed to have lepromatous leprosy. She was given the 364 C. Clofazimine is a phenazine dye which causes appropriate drugs which she took religiously. One day she came back to red- brown to nearly black skin discoloration. SE of you complaining that her skin changed to a red-brownish color. Which of dapsone: methemoglobinemia; Rifampicin: red orange the following drug/s most likely have caused her predicament? A. discoloration of body fluid Katzung 12th ed pg 846 Dapsone B. Rifampicin C. Clofazimine D. Rifabutin E. Cycloserine 69. A 34-year-old carpenter presents to the ER after an accident in which he 37 D. Epinephrine is contraindicated as an anesthetic inadvertently chopped off the tip of his index finger. He is taken to the adjuvant for surgeries involving most facial structures, OR for reattachment of the digit, and after sedation, a local anesthetic is digits, and the penis, because of the risk of vascular administered around the site of the injury. The local anesthetic used in compromise. This agent causes decreased blood loss the procedure did not contain any epinephrine, as it usually does for for most other surgeries because of vasoconstriction. most surgical procedures. The reason for this is: A. Epinephrine causes Although local anesthetic agents such as lidocaine or increased blood loss during delicate surgery B. Epinephrine causes xylocaine can cause mild local tissue swelling, swelling of the tissues, making surgery more challenging C. Epinephrine epinephrine does not; either way, it is not a is contraindicated in emergency surgery D. Epinephrine causes contraindication for hand surgery. Epinephrine causes vasoconstriction, which can lead to vascular ischemia in digits E. elevated blood pressure when administered Epinephrine can cause hypotension when administered with sedative systemically; however, it has no systemic side effects agents when administered locally. 70. A 35 year old female underwent treatment for endometriosis for 12 603 B. Danazol is a derivative of the synthetic steroid months. She then developed male-pattern hair growth, deepening of the ethisterone, a modified testosterone, also known as 17- voice and acne. The drug given is most likely: A. GnRH agonist B. alpha-ethinyl testosterone. Danazol inhibits ovarian Danazol C. Combined oral contraceptive pills D. Medroxyprogesterone steroidogenesis resulting in decreased secretion of acetate E. GnRH antagonist estradiol and may increase androgens. This drug causes androgenic side effects, such as hirsutism, deepening of the voice, acne and oily skin. 71. A 35 year old woman came to your clinic because of dysuria. She has 518 D. Quinolones act by blocking bacterial DNA been experiencing burning on urination with increased frequency and synthesis by inhibitting bacterial topoisomerase II and urgency to urinate for 5 days now. Urinalysis revealed WBC of 20- IV. Moxifloxacin is the only non-renally cleared 30/hpf, bacteria=moderate, nitrite=positive. You may prescribe the fluoroqionolone and thus achieves low urinary level following drugs to her except for: A. Ciprofloxacin B. Levofloxacin C. making it ineffective for cases of UTI. Katzung 10th Ofloxacin D. Moxifloxacin E. None of the above ed., 768. 72. A 37 year old female diagnosed with chronic schizophrenia maintained 291 C. Clozapine can cause agranulocytosis in a small on an antipsychotic medication was brought to the clinic due to a 3 day but significant number of patients (1-2%). This serious, history of fever. PE showed presence of tonsillar exudates. A CBC was potentially fatal effect can develop rapidly usually requested which showed a markedly decreased WBC count. The between the 6th to 18th week of treatment. It appears medication was immediately discontinued by the patient's physician. to be reversible upon discontinuation of treatment. What is the anti-psychotic medication that could have caused this Because of this risk, patient on clozapine must have problem? A. Haloperidol B. Quetiapine C. Clozapine D. Risperidone E. weekly blood counts for the first 6 months of Olanzapine treatment and every 3 weeks thereafter 73. A 37 year old man comes to the physician because of a 1 week history of 521 C. pain with swallowing. He received the diagnosis of AIDS 4 years ago, but has not been able to tolerate highly active antiretroviral therapy. He currently takes TMP-SMX. Vital signs are within normal limits. Examination shows a few white plaques over the pharynx. CD4+ count is 50/mm3 and plasma HIV viral load is 50000 copies/mL. Which of the following is the most appropriate pharmacotherapy? A) Acyclovir B) Amphotericin B C) Fluconazole D) Foscarnet 74. A 37 year old man comes to the physician for a follow-up examination. He was diagnosed 535 C. Start HAART with CD4 with HIV infection 3 weeks ago. He is asymptomatic and takes no edications. His below 350 and Pneumocystis temperature is 37.2 C, pulse is 100 bpm, and blood presure is 100/60 mmHg. Examination prophylaxis with CD4 below 200 shows no abnormalities except for erythematous scaling at the hairline and nasolabial folds. His CD4+ T-lymphocyte count is 160/mm3 and plasma HIV viral load is 25,000 copies/mL. Which of the following is the most appropriate pharmacotherapy? A) Pneumocystis jiroveci prophylaxis only B) Three-drug antiretroviral therapy only C) Three- drug antiretroviral therapy and P. jiroveci prophylaxis D.) Two Drug Antiretroviral therapy and P. iroveci Prophylaxis 75. A 39 year old female, known case of Graves' disease but poorly compliant with 298 A. beta blockers are the most medications, presented to the ER due to palpitations and shortness of breath. She was effective agents in acute thyrotixc hooked to a cardiac monitor which showed supraventricular tachycardia. Which of the arrythmias . Esmolo is a rapid following drugs would be most suitable for this case ? A. Esmolol B. Quinidine C. Flecainide parenteral beta blocker D. Disopyramide E. Lidocaine 76. A 39 y/o male went for follow up consult secondary to open angle glaucoma. He was 309 C. dermanent discoloration of compliant with all the topical medications given to him. Indirect ophthalmoscopy revealed the iris and eyelashes is a well brownish discoloration in his iris. What is the mechanism of action of the drug that causes known side effect of latanoprost. such change?. A. decrease aqueous secretion due to lack of HCO3 B. increase outflow via Its mechanism of action is to uveoscleral veins C. increase aqueous outflow D. decrease aqueous secretion from the increase aqueous outflow. A- ciliary epithelium E. none of the above brinzolamide, dorsolamide, B- epinephrine. D- timolol. Katzung review p 87. 77. A 41/M, presenting with high fever and chills, is admitted at the hospital. He reports having 557 D. Aztreonam is active only had serious allergic reactions to amoxicillin one year ago. Blood GS/CS reveals growth of against Gram-(-) organisms and has Gram negative bacteremia. Which of the following antibiotics is MOST appropriatE? A. NO cross-reactivity with penicillins. Cefazolin B. Ampicillin-sulbactam C. Piperacillin-tazobactam D. Aztreonam E. Vancomycin 78. A 42-year-old MDS patient presents to the emergency room with mental status changes 28 C. Toxoplasmosis is treated with and a headache. A computed tomography scan is ordered and demonstrates a ring- a combination of pyrimethamine enhancing lesion. You decide to treat him empirically due to the possibility of and sulfadiazine. Ivermectin is used Toxoplasmosis gondii abscess. Which agent should be included in his treatment? A. to treat filariasis, whereas Ivermectin B. Praziquantel C. Sulfadiazine D. Niclosamide E. Pyrantel pamoate praziquantel is used to treat schistosomiasis. Niclosamide can be used to treat tapeworm infections, and pyrantel pamoate is used to treat many helminth infections. 79. A 42-year-old woman, gravida 2, para 2, comes to the physician because of a 3-month 533 A. history of swelling of her legs and mild abdominal pain and bloating. Abdominal examination shows no abnormalities. Rectovaginal examination shows fullness in the right adnexa. Transvaginal ultrasonography shows an irregular mass in the right ovary with some solid components to a predominantly cystic lesion. Her serum CA 125 level is 120 U/mL (N<35). Treatment with which of the following is most likely to have prevented this patient's symptoms? A) Antiestrogens B) Antiprogestationals C) Medroxyprogesterone D) Oral contraceptives 80. A 43 year-old male was diagnosed with Pulmonary Tuberculosis. He was given Anti-TB 451 D. Pyrazinamide provoke avute medications. However, he developed hyperuricemia. Which among these drugs inhibits the gouty arthritis by inhibiting urate excretion of urate causing the above findings: A. Ethionamide B. Streptomycin C. excretion. Ethambutol D. Pyrazinamide E. Rifampicin 81. A 45 year old male complained of chest pain upon exertion but relieved by rest. The 464 B. drugs used in angina doctor prescribed him a beta blocker. What is the effect of beta blocker? A. Increase pectoris are the following: those oxygen supply B. Decrease oxygen demand C. Increase oxygen demand D. decrease that increase the oxygen supply oxygen supply E. A and B are: nitrates and calcium blockers; those that decrease the oxygen demand are: beta blockers and still calcium blockers 82. A 45 year old male presented to the ER due to sudden onset of right 295 D. Atropine is a mydriatic and cyloplegic and can eye pain accompanied by blurring of vision. On examination there was further increase IOP hence worsen angle closure note of ciliary injection over the right eye, IOP pressure was noted to glaucoma. be elevated. A diagnosis of acute angle closure glaucoma was made. Which of the following ophthalmic medications should be avoided ? A. Brinzolamide B. Timolol C. Apraclonidine D. Atropine E. All of the above 83. A 45 year old male with history of polyuria, polyphagia and weight loss 10 D. The first and second generation sulfonylureas are was recently started on antidiabetic medications after workup associated with weight gain. Source: Katzung Basic and confirmed DM type II. 2 months later, he notes increasing weight. Which Clinical Pharmacology 11th ed p748 of the following antidiabetic medications is associated with this? A. Chlorpropramide B. Glimepiride C. Exenetide D. A and B E. All of the above 84. A 45 year old man who has received long treatment for schizophrenia 623 B. this is TARDIVE DYSKINESIA, which is a has recently been noted to display involuntary movements that include complication of long term antipsychotic therapy with lateral deviations of the jaw and "fly catching" motions in his tongue. the older agents like, FLUPHENAZINE. Which of the following agents is the most likely cause of his involuntary movements? A. Clozapine B. Fluphenazine C. Lithium D. Risperidone E. selegiline 85. A 45-year-old man with a history of medication-controlled 21 D. Hydrochlorothiazide, a thiazide diuretic, can hypertension presents to your office with complaints of a painful, precipitate a gouty attack in predisposed individuals. swollen big toe on the left foot. You suspect gout and check his uric This is because these agents increase serum uric acid as acid levels, which are elevated. From looking at the list of the a result of competition for the organic acid carrier. medications the patient is taking, you realize that one of the Loop diuretics can have this effect too. Acetazolamide medications may be the cause of his current symptoms. Which is a carbonic anhydrase inhibitor; this agent does not medication might that be? A. Acetazolamide B. Amiloride C. have a significant impact on the levels of uric acid. Spironolactone D. Hydrochlorothiazide E. Mannitol Amiloride and spironolactone are potassium-sparing diuretics, and they do not have a significant impact on the levels of uric acid either. The same is true for mannitol, an osmotic diuretic. 86. A 45 year old soldier has been maintaining on fluoxetine for his major 520 E This is a case of serotonin syndrome which is a depressive disorder. Which of the following drugs can precipitate a condition precipitated when MAO inhibitors are given potentially fatal syndrome of hypertension, hyperreflexia, tremor, with serotonin agaonists, especially antidepressants of clonus, hyperhtermia, diarrhea, mydriasis and agitation within hours the SSRI class. Amytriptyline and Imipramine are TCAs. when taken consurrently with fluoxetine? A. Amytriptyline B. Fluoxetine, Sertraline and Citalopram are SSRIs. Imipramine C. Sertraline D. Citalopram E. Phenelzine Phenelzine is a MAO inhibitor. Katzung 10th ed., 267. 87. A 47-year-old man is admitted to the hospital after threatening to harm 528 C. a radio announcer he believed was broadcasting his thoughts. Over the past 20 years, he has had multiple psychiatric hospitalizations for threatening people who he believed were plotting against him, trying to control his mind, or causing him to hear voices by implanting devices in his head. Past symptoms improved with neuroleptic therapy; after discharge, he discontinued the medication and his symptoms worsened. Which of the following is the most appropriate pharmacotherapy to decrease this patient's risk for future hospitalization? A) Clozapine B) Fluphenazine hydrochloride C) Haloperidol decanoate D) Risperidone 88. A 47-year-old man is admitted to the hospital after threatening to harm 529 C. a radio announcer he believed was broadcasting his thoughts. Over the past 20 years, he has had multiple psychiatric hospitalizations for threatening people who he believed were plotting against him, trying to control his mind, or causing him to hear voices by implanting devices in his head. Past symptoms improved with neuroleptic therapy; after discharge, he discontinued the medication and his symptoms worsened. Which of the following is the most appropriate pharmacotherapy to decrease this patient's risk for future hospitalization? A) Clozapine B) Fluphenazine hydrochloride C) Haloperidol decanoate D) Risperidone 89. A 47 year old obese male, who is a heavy alcoholic beverage drinker, was 283 D. Metformin is associated with lactic acidosis. recently diagnosed with diabetes mellitus. He was started on Metformin 500 It should be avoided or used with extreme caution mg/tab TID. This patient is at risk of developing which of the following in patients who are heavy alcoholics since acute complications? A. Hypoglycemia B. Disulfiram like reaction C. Congestive ethanol ingestion increaes the risk of lactic acidosis heart failure D. Lactic acidosis E. Diarrhea 90. A 47 year old woman comes to the physician for a routine health 523 D. maintenance examination. She has a 2 year history of venous insufficiency and noted daily swelling of her ankles that worsens in the evening. She takes no medications. Her pulse is 80 bpm, and blood pressure is 160/100 mmHg. Cardiopulmonary examination shows no abnormalities. Examination of the lower extremities shows 2+ edema and increased pigmentation .Which of the following antihypertensive medications would most likely exacerbate this patient’s swelling? A) Atenolol B) Lisinopril C) Losartan D) Nifedipine 91. A 48/M farmer with a history of alcohol abuse, is rushed to the emergency 542 E A chemical antagonist interacts directly with room after the patient has deliberately ingested 300 mL of an unknown the drug being antagonized to remove it or pesticide while drunk. On examination, you note that the patient has prevent it from binding to its target. Pralidoxime pinpoint pupils and frothy secretions at the mouth. After you determine that which combines avidly with the phosphorus in the patient has ingested malathion; you then immediately start treatment organosphosphate cholinesterase inhibitors is an with pralidoxime. Pralidoxime and malathion are examples of: A. example. (KBR9e, p. 14) Competitive pharmacologic antagonists B. Irreversible pharmacologic antagonists C. Physiologic antagonists D. Structural antagonists E. Chemical antagonists 92. A 48-year old Caucasian male is referred to you for chronic cough, night 543 A. 50% of white and African american persons sweats and weight loss. Sputum AFB was positive on three occasions; are slow acetylators of INH, hydralazine and findings of chest X ray done was also consistent with pulmonary procainamide. KBR9e, p33 tuberculosis. Before you start drug therapy, you remember that a much larger percentage of Caucasians are slow acetylators of one of the anti-TB drugs. The dosage of which of the following may need to be modified. A. Isoniazid B. Rifampicin C. Streptomycin D. Pyrazinamide E. Ethambutol 93. A 48 year old female, a known case of Acute Myelogenous Leukemia (AML), 287 A. Valaciclovir is a prodrug, an esterified who is on reinduction chemotherapy was started on anti viral prophylaxis version of aciclovir that has greater oral with Valacyclovir 500 mg/tab TID. What is the mechanism of action of this bioavailability (about 55%) than aciclovir anti viral medication? A. Inhibition of Viral DNA polymerase B. Blockage of (10â€"20%). It is phophorylated three times and this M2 proton channel C. Inhibition of neuraminidase D. Inhibition of viral form acts as a competitive substrate for DNA reverse transcriptase E. Prevents fusion of virus with the host cellular polymerase and it leads to chain termination membrane 94. A 49 year old made came to the ER with a 3 hour history of anginal chest 3 C. Other than Aspirin, NSAIDs are absolutely pain. Past medical history is significant only for uncontrolled hypertension. contraindicated to patients with acute MI. Drugs Cardiac markers and ECG both confirm the diagnosis of acute myocardial which are administered post MI include beta- infarction probably of the anterior wall. Currently the patient has stable vital blockers (unless with contraindications), ACE signs within normal range with persistence of angina. The following should inhibitors (beneficial in preventing cardiac be given except: A. Lisinopril B. Simvastatin C. Ibuprofen D. Metoprolol E. remodeling), Statins (stabilizes the atheromatous None of the above. All should be administered. plaque), antiplatelet therapy like aspirin or clopidogrel, aldosterone antagonist (epleronone only), nitrates (increase O2 supply). 95. A 50/M is about to be given 6-mercaptopurine for his ulcerative colitis as 371 D. Because allopurinol inhibits xanthine an off-label use. The patient has multiple comorbidities which include oxidase, simultaneous therapy with allopurinol and hypertension, diabetes, asthma and gout and takes losartan, metformin, 6-MP would result in increased levels of 6-MP, salmeterol, allopurinol and diclofenac for these conditions. Which of the thereby leading to excessive toxicity. In this above drugs would prompt you to rethink the dose of 6-MP that you are setting, the dose of mercaptopurine must be about to administer? A. Losartan B. Metformin C. Salmeterol D. Allopurinol reduced by 50â€"75%. Katzung 12th ed pg 961-962 E. Diclofenac 96. A 50/M previously diagnosed with BPH develops mild fever, 373 B. Sympathomimetic agents may cause or worsen urinary cough and rhinorrhea. Thinking that he just has the common difficulty in patients with prostate enlargement due to smooth cold, he self medicates with over the counter medication. muscle contraction in the bladder neck via stimulation of alpha-1 After 12 hours he rushes to the ER and complains of inability adrenergic receptors. Katzung 12th ed pg 139 to void. Which of the following OTCs may have caused his condition? A. Chlorphenamine B. Phenylephrine C. Acetaminophen D. Ibuprofen E. None of the above 97. A 50-year man with mild hypertension complains of 24 B. SIMILAR TO PREVIOUS BOARD EXAM discomfort in his chest. He has slightly enlarged fat deposits CONCEPT/PRINCIPLE. Spironolactone antagonizes the action of in his breasts with prominent nipples. Which of the following the mineralocorticoid, progesterone, and androgen receptors. medications might be causing this adverse effect? A. Inhibition of androgen receptors can lead to gynecomastia and Amiloride B. Spironolactone C. Metolazone D. breast tenderness, most often in men. Hydrochlorothiazide E. Acetazolamide 98. A 50 year old female consulted due to palpitations. This was 651 D. RAI may induce production of cancer cells but ALL patients accompanied by frequent bowel movements and heat administered with RAI will suffer from permanent hypothyroidism intolerance. You gave her Radioactive iodine and you know and will require exogenous thyroxine as supplement that its most common side effect is? A. Papillary Thyroid CA B. Medullary Thyroid CA C. Rebound hyperthyroidism D. Permanent hypothyroidism E. Rebound tachycardia 99. A 50-year old male with DM type II had been on metformin 128 C. This is a case of hypomagnesemia, which can be for nine years and presented with diarrhea, paresthesia, and exacerbated by administration of a loop diuretic Furosemide is a muscular weakness. On examination, he has hyperactive loop diuretic that acts on the thick ascending limb of the loop of DTRs. Blood electrolytes showed serum Na 138 mEq/L, henle. This tubular segment is responsible for a significant sodium Serum K= 3.4 mEq/L, serum Ca=8.2 mg/dL and Mg 0.8 mEq/L. chloride reabsorption, as well as the site for calcium and Which of the following medications should you avoid to magnesium reabsorption. Side effects of Furosemide : prevent exacerbation of his symptoms? A. Spironolactone B. Hypokalemia, hypocalcemia, hypomagnesemia, hyperuricemia, Magnesium sulfate C. Furosemide D. Indomethacin E. dehydration, metabolic alkalosis, ototoxicity, sulfa allergy, nephritis Acetazolamide 100. A 50 year old obese patient was diagnosed to have DM type 479 C. Metformin is the first line treatment for type 2 DM and is 2. There were no other comorbidities. What hypoglycemic the drug of choice for obese patients. It is contraindicated in drug should you prescribed with this patient? A. Insulin B. patients with renal disease, hepatic disease, and alcoholism Gliclazide C. Metformin D. Pioglitazone E. Sitagliptin 101. A 50 year old smoker, hypertensive patient complained of 2 462 A. month cough and dyspnea. He was on Carvedilol for his hypertension. What is the side effect of this drug that aggravates the condition. A. bronchospasm B. bronchoconstriction C. tachycardia D. tremor E. all of the choices 102. A 53/M, admitted at the Medical ICU, after having a 552 D. Lidocaine and other Class IB drugs act primarily in ischemic myocardial infarction, develops ventricular tachycardia. tissues and are best for arrhythmias following MI. Which of the following anti-arrhythmics is most appropriate? A. Quinidine B. Mexiletine C. Adenosine D. Lidocaine E. Flecainide 103. A 53-year-old woman with breast cancer undergoes a 27 E Paclitaxel is often used in the treatment of breast as well as breast-conserving lumpectomy and lymph node biopsy. The ovarian and lung cancer. Its main toxicities are myelosuppression pathology report returns with mention of cancer cells in two and peripheral neuropathy that usually manifest as numbness and of eight lymph nodes removed. Following radiation therapy, tingling in the distal extremities. Blood in the urine can indicate chemotherapy is started that includes the use of paclitaxel. hemorrhagic cystitis, a complication of cyclophosphamide use. Which side effect is the patient likely to complain of? A. Easy bruising can result from mechlorethamine use. Hot flashes Blood in the urine B. Easy bruising C. Hot flashes D. are a common complaint in patients using tamoxifen. Shortness of Shortness of breath E. Numbness and tingling breath can result from pulmonary fibrosis secondary to busulfan or bleomycin use. 104. A 53 year old woman with newly diagnosed type 2 diabetes presents to 621 E This is a disulfiram like reaction after drinking mergency department complaining of vomiting, severe headache, alcohol at a cocktail party. TOLBUTAMIDE is dizziness, blurry vision, and DOB. She says that she had been at a cocktail associated with this kind of reaction party when the symptoms began. her skin is notably flushed on physical examination. Which of the following medications is responsible for this reaction? A. Acarbose B. Glipizide C. Glyburide D. Metformin E. Tolbutamide 105. A 55/M comes in due to sudden headache described as the "worst 560 A. Nimodipine given for 21 days should be headache of his life." He is a diagnosed hypertensive, with poor compliance started on admission for vasospasm prophylaxis. to medications. PE reveals nuchal tenderness (positive Kernig's sign), and a left cranial nerve palsy. Brain CT reveals diffuse subarachnoid hemorrhage in the basal cisterns. Which of the following drugs has a proven role in the management of his condition? A. Nimodipine B. Propanolol C. Captopril D. Nifedipine E. Aspirin 106. A 55-year-old man has had crushing substernal chest pain on exertion over 524 B. the past 6 weeks. He had a myocardial infarction 2 months ago. He takes nitroglycerin as needed and one aspirin daily. He has smoked two packs of cigarettes daily for 30 years. Examination shows normal heart sounds and no carotid or femoral bruits. Treatment with a β-adrenergic blocking agent is most likely to improve his symptoms due to which of the following mechanisms? A) Decreasing diastolic relaxation B) Decreasing myocardial contractility C) Dilating the coronary arteries D) Peripheral vasodilation 107. A 55-year-old woman is admitted to the surgical intensive care unit after 40 B. Milrinone reduces left ventricular filling having a coronary artery bypass grafting of four of her coronary vessels. pressure and thus enhances cardiac output. It is Overnight she develops hypotension, and her cardiac output, as measured related to the anticholinergic agent biperiden. by the Swan-Ganz catheter, is significantly lower than it had been Milrinone inhibits cardiac phosphodiesterase type postsurgery. You decide to give her a dose of milrinone. This results in an 3. It increases cAMP, and therefore intracellular increase in her cardiac output. How does milrinone work? A. It is a calcium. cholinergic agonist B. It reduces left ventricular filling pressure C. It potentiates cardiac phosphodiesterase type 3 D. It decreases cyclic AMP E. It decreases intracellular calcium 108. A 55-year-old woman was recently diagnosed with an adrenal mass due to 536 B. Phenoxybenzamine >> nonselective, symptoms of sweating, elevated blood pressure, and severe headaches. IRREVERSIBLE alpha blocker..Phentolamine>> Which of the following nonselective, irreversible alpha blocker is the drug reversible alpha blocker. of choice as a preoperative agent used in this disease? A) Phentolamine B) Phenoxybenzamine C) Prazosin D) Yohimbine 109. A 55 y/o male presented at the ER with complaints of 3 day diarrhea, 109 D. Page 12 of hand out. The patient is frequent urination, increased salivation and sweating. You noted his pupils experiencing symptoms of organophosphate to be constricted. He works in a factory manufacturing pesticides for farms. poisoning. Remember the DUMBBELSS mnemonic! Which of the following drugs will you give this patient? A. Pyridostigmine B. Diarrhea, Urination, Miosis, Bronchospasm, Edrophonium C. Pilocarpine D. Atropine E. Varenicline Bradycardia, Excitation (skeletal muscle and CNS), Lacrimation, Sweating and Salivation. The antidote to be given in organophosphate poisoning is Atropine (Cholinergic antagonist, muscarinic) and Pralidoxime (Cholinesterase regenerator) 110. A 56 year old patient diagnosed with rheumatic heart disease underwent 242 A. SIMILAR TO PREVIOUS BOARD EXAM mitral valve replacement and is placed on oral anticoagulant Warfarin, she CONCEPT/PRINCIPLE, which asked us directly comes to your clinic for a follow up check up and complaints of heartburn which of the following is a CYP inhibitor: and epigastric pain, which of the following medications will you avoid cimetidine, Warfarin is majorly metabolized by CYP prescribing? A. Cimetidine B. Aluminum Magnesium Hydroxide C. 2C9, cimetidine inhibits this enzyme among others lansoprazole D. ranitidine E. sucralfate which inhibits warfarin metabolism which increases its toxicity 111. A 57-year-old man has been hospitalized for 2 days for treatment of unstable angina 532 D. Heparin-induced pectoris. He is currently receiving intravenous heparin and undergoing evaluation for thrombocytopenia >>> the Heparin- coronary artery bypass grafting. His blood pressure is 160/90 mm Hg, pulse is 88/min, Platelet4 complex are attacked by IgG and respirations are 16/min. Laboratory studies show: Platelet count 90,000/mm3 in the human bodies that essentially Prothrombin time 12 sec (INR=1.1) Partial thromboplastin time 35 sec Which of the activate the platelet and thus form following is the most likely cause of these findings? A) Excessive platelet destruction clots. These clots are the reason free B) Factor VIII deficiency C) Inadequate platelet production D) Uncontrolled activation platelets are low. of coagulation and fibrinolytic cascades 112. A 58 year old male hypertensive was diagnosed with benign prostatic hyperplasia. 471 B. Alpha1 blockers are What is the antihypertensive drug that is the most selective for prostatic smooth antihypertensive that is also used for muscle? A. prazosin B. tamsulosin C. losartan D. metoprolol E. Terazosin BPH. Prazosin, tamsulosin, and terazosin are alpha1 blockers, but the most selective for prostatic smooth muscle is tamsulosin. 113. A 58-year-old woman with newly diagnosed type 2 diabetes mellitus presents to the 538 A. tolbutamide >> disulfuram emergency department complaining of vomiting, severe headache, dizziness, blurry reaction vision, and dyspnea. She says that she had been at a party when the symptoms began. Her skin is notably flushed on physical examination. Which of the following medications is responsible for this reaction? A) Tolbutamide B) Glipizide C) Glyburide D) Metformin 114. A 59-year-old female nurse who has been diagnosed with type 2 diabetes is admitted 26 C. Any of the sulfonylureas can to the emergency room. She is tachycardic, tachypneic, and appears very disoriented; cause hypoglycemia which can she does not remember the day of the week or her address or any emergency contact produce shock-like symptoms. numbers. She vaguely remembers taking her “sugar medicine†earlier in the day. Metformin and the α-glycosidase Which of the following drugs is most likely responsible for her condition? A. Metformin inhibitors such as acarbose rarely B. Acarbose C. Glipizide D. Glucagon E. Pioglitazone cause ypoglycemia. Glucagon would raise plasma glucose. 115. 59 year old male, hypertensive for 22 years, came in the ER due to difficulty of 182 A. my mnemonics: improves breathing. PE revealed crackles on bilateral lower lobes of the lungs and bipedal QUANTITY of life in CHF- ABS edema. Which of the drugs used in CHF can prolong survival of patients? A. carvedilol (ACEI/ARBS, Beta-Blockers, B. atenolol C. digitalis D. furosemide E. A and B Spironolactone). Improves QUALITY of life- Digital Film (Digitalis, Furosemide). Only 3 beta blockers are found to be useful in CHF: Carvedilol, metoprolol and bisopolol. 116. A 60/M was diagnosed to have refractory follicular B-cell lymphoma. He was 368 A. Cetuximab targets EGFR; prescribed with an agent which binds CD20 on both normal and malignant B Bevacizumab targets VEGF; lymphocytes. Which of the following is this drug? A. Rituximab B. Cetuximab C. Omalizumab is an anti-IgE useful in Bevacizumab D. Omalizumab E. Natalizumab asthma therapy; Natalizumab binds to α4-subunit of α4β1 and α4β7 integrins expressed on the surfaces of all leukocytes except neutrophils. Used in MS and Crohn's disease. Katzung 12th ed pg 991-993 117. A 60 year-old female patient was diagnosed with breast cancer. She was given an 457 B. Acrolein is the metabolite of alkylating agent, Cyclophosphamide. Later in the treatment, she developed Cyclophosphamide that causes hemorrhagic cystitis due to this toxic metabolite: A. Amifostine B. Acrolein C. hemorrhagic cystitis. Mercaptopurine D. Lomustine E. Mesna 118. A 63 year old male patient presenting with intermittent complaints of chest pain upon 269 A. Nitroglcerin decreases walking several blocks or climbing several flights of stairs is prescribed with afterload, thus, decreasing end systolic nitroglycerin to used on an as needed basis. What will be the expected physiologic volume. Compensatory sympathetic changes as a result of this drug? A. Increased HR and and decreased left ventricular activity increases heart rate. end-systolic volume B. Decreased HR and decreased left ventricular end-systolic volume C. Decreased HR and increased left ventricular end-systolic volume D. Increased HR and increased left ventricular end-systolic volume E. None of the above 119. A 65/M hypertensive, diabetic, smoker presents to you with fever, 363 D. Vancomycin and meropenem need to be productive cough with an infected non healing wound in the foot. Labs adjusted Katzung 12th ed pg 798 Metoprolol is showed anemia and very elevated creatinine.You schedule the patient metabolized in the liver and requires no adjustment pg for dialysis. As the prescribing physician, which of the following drug/s 179 dose/s would you have to modify? A. Vancomycin B. Meropenem C. Metoprolol D. A and B E. All of the above 120. A 65/M was diagnosed to have CAP-HR. You plan to start Piperacillin- 377 B. Clearance is involved in determining the Tazobactam with a loading dose of 4.5g IV. Which among the following maintenance dose Topnotch variables is not involved in the calculation of the loading dose? A. Volume of distribution B. Clearance C. Bioavailability D. Desired concentration E. All are needed to calculate the LD 121. A 65 year old female presented to the ER due to progressive dyspnea. 284 C. This is a case of an acute decompensated heart She is a known hypertesive but is poorly compliant with medications. failure. Due to the marked systolic dysfunction as On history, patient claims to experience orthopnea, paroxysmal evident by a depressed ejection fraction, beta blockers nocturnal dyspnea and easy fatigability. On PE, her BP is 80/50. There are contraindicated at this time as it may further lower is prominent neck vein distention. S3 gallop, bibasal crackle and a the cardiac output. grade 3 bipedal edema were also appreciated. A 2d echo was done which showed a depressed ejection fraction of 32%. Which of the following medications should not be given at this time for this patient ? A. Furosemide B. Digoxin C. Metoprolol D. Dobutamine E. None of the above 122. A 65 year old man comes to the physician because he awakens to 622 A. BPH. Finasteride is a 5 alpha reductase inhibitor urintae several times per night and has developed problems starting that inhibits conversion of testosterone to DHT. and stopping his stream of urine. A biopsy of the prostate shows enlargement and dilation of the prostate glands but no dysplasia. Which of the following is the most appropriate pharmacological treatment for this patient? A. Finasteride B. Flutamide C. Ketoconazole D. Spirinolactone E. Yohimbine 123. A 65 y/o male on Clonidine for a few months for his hypertension 112 A. Page 17 of Topnotch Handout. For rebound abruptly discontinued his medication because he felt he was feeling hypertension due to clonidine withdrawal the DOC is better. After 2 days, during which he had strong headaches. Three days Phentolamine. Clonidine is an alpha 2 agonist, after withdrawal, the patient had home blood pressure levels of sympathetic outflow blocker. Phentolamine Its primary 140â€"150/95â€"105 mmHg. Ten days after, the patient went to the action is vasodilation due to α1 blockade, but can also emergency room with a symptomatic hypertensive crisis at 220/130 lead to reflex tachycardia because of hypotension and mmHg. What is the drug of choice to treat the rebound hypertension? α2 inhibition, which increases sympathetic tone. The A. Phentolamine B. Hydrochlorthiazide C. Propanolol D. Hydralazine E. primary application for phentolamine is for the control Nicardipine of hypertensive emergencies. 124. A 67-year old male, on warfarin maintenance after undergoing valve 611 A. Cimetidine inhibits metabolism of warfarin, replacement surgery, was brought to the ER due to unresolving thereby prolonging its effect. epistaxis and easy bruising noted for the past week. In asking the history, the following substances are also being taken by the patient. Which of them contributed to the patient's condition? A. Cimetidine B. Digoxin C. Griseofulvin D. Procainamide 125. A 67-year-old male with ischemic heart disease, compliant with his 17 E This is a case of acute renal failure secondary to medications is admitted for acute renal failure. Labs reveal statin induced rhabdomyolysis. Statins inhibit hyperkalemia, a rapidly elevating creatinine, and markely increased CK cholesterol synthesis. One of the intermediates of and myoglobin levels. Which of the following medications he was taking cholesterol synthesis is farnesyl. Farnesyl is needed for could have precipitated the condition? A. Aspirin B. Carvedilol C. the synthesis of Coenzyme Q, which is important for Losartan D. Captopril E. Rosuvastatin ATP production in metabolically active cells like the muscles. Depletion of this can result to rhabdomyolysis. To prevent this, one can administer Coenzyme Q10 with statins. Source: Katzung Basic and Clinical Pharmacology 11th ed p 1122 126. A 67-year-old male with ischemic heart disease, compliant with his 16 E This is a case of acute renal failure secondary to medications is admitted for acute renal failure. Labs reveal hyperkalemia, statin induced rhabdomyolysis. Statins inhibit a rapidly elevating creatinine, and markely increased CK and myoglobin cholesterol synthesis. One of the intermediates of levels. Which of the following medications he was taking could have cholesterol synthesis is farnesyl. Farnesyl is needed precipitated the condition? A. Aspirin B. Carvedilol C. Losartan D. for the synthesis of Coenzyme Q, which is important Captopril E. Rosuvastatin for ATP production in metabolically active cells like the muscles. Depletion of this can result to rhabdomyolysis. To prevent this, one can administer Coenzyme Q10 with statins. Source: Katzung Basic and Clinical Pharmacology 11th ed p 1122 127. A 67-year-old man is hospitalized for treatment of renal insufficiency. 534 C. insulin and glucose >> fastest way to lower Three days after admission, his pulse is 40/min. An ECG shows tall, tented down serum K T waves. Serum studies show a sodium level of 134 mEq/L, potassium level of 6.9 mEq/L, and glucose level of 85 mg/dL. The most appropriate next step in management is intravenous administration of which of the following? A) Calcium, furosemide, and 3% saline B) Calcium, insulin, and digitalis C) Calcium, insulin, and glucose D) Glucose, furosemide, and phosphate 128. A 67 year old woman, known diabetic for 20 years and maintained on 299 B. Metoclopramide is DOC for diabetic Gliclazide, complained of severe bloating and post prandial fullness. gastropathy Evaluation was done and she was assessed to have diabetic gastropathy. Which of the following medications would be most appropriate for this case? A. Cimetidine B. Metoclopramide C. Alosetron D. Loperamide E. Bismuth subsalicylate 129. A 70 year old male who is diagnosed with Colon Cancer Stage IV with 289 D. Imatinib is a tyosine kinase inhibitor used in metastasis to the liver who is currently on FOLFOX regimen was started on CML. Sorafenib inhibit multiple tyrosne kinase a monoclonal antibody which binds to the vascular endothelial growth receptors and are primarily used in HCC. factor (VEGF) receptor. What is the medication that was given to the Trastuzumab inhibits cells that overexpress the Her- patient? A. Imatinib B. Sorafenib C. Trastuzumab D. Bevacizumab E. 2/neu recepto in Breast CA. Erlotinib inhibits Erlotinib epidermal growth factor receptor and is used for non small cell lung CA and pancreatic cancer 130. A 72-year-old man comes for a routine follow-up examination. He has 525 C. COPD and Peripheral arterial disease >> chronic obstructive pulmonary disease treated with β-adrenergic agonists contraindications for Beta-Blockers and ipratropium by metered-dose inhaler and mild arterial insufficiency of the lower extremities treated with aspirin. His blood pressure is 160/60 mm Hg, pulse is 70/min, and respirations are 12/min. Funduscopic examination shows arteriovenous nicking. Pedal pulses are decreased bilaterally. Which of the following antihypertensive drugs is most likely to cause adverse effects in this patient? A) α2-Adrenergic agonist B) α-Adrenergic blocking agent C) β-Adrenergic blocking agent D) Angiotensin-converting enzyme (ACE) inhibitor 131. A 72-year-old man comes to the physician because of a 2-month history of urination 527D. twice nightly and occasional urinary frequency and urgency. He has a 15-year history of type 2 diabetes mellitus now moderately well controlled with glyburide. His father was diagnosed with prostate cancer at the age of 70 years, and his sister died of complications from systemic lupus erythematosus. His blood pressure is 135/86 mm Hg. Cardiopulmonary examination shows no abnormalities. Abdominal examination shows no suprapubic fullness or tenderness. There is mild enlargement of the prostate with no palpable nodules. His postvoid residual volume is 10 mL. Serum studies show a urea nitrogen (BUN) level of 45 mg/dL and creatinine level of 3.8 mg/dL. Urine dipstick shows 3+ protein. Which of the following is most likely to have prevented progression of this patient's renal disease? A) Oral finasteride therapy B) Oral terazosin therapy C) Oral cyclophosphamide and prednisone therapy D) Oral enalapril therapy 132. A 72 year old man is brought to the physician by his daughter because of a 7 month 522 B. history of difficulty with memory. He frequently loses his keys and misses appointments and regular family dinners. He lives alone, but recently family members had to drive him on errands and help him with daily tasks at home. He has not had difficulty sleeping and does not use illicit drugs. Mental status examination shows an irritable mood and labile affect. There is no evidence of depressed mood, anxiety, or hallucinations. Which of the following is the most appropriate pharmacotherapy for this patient? A) Alprazolam B) Donepezil C) Dextroamphetamine D) imipramine 133. A 72-year-old man was diagnosed to have glaucoma. Which of the following drugs 537 D. Pilocarpine >> ciliary muscle induces ciliary muscle contraction thereby opening the trabecular meshwork and contraction >> opening trabecular increasing the outflow? A) Timolol B) Mannitol C) Epinephrine D) Pilocarpine meshwork and increasing the outflow 134. A 74-year-old man with a 100-pack/year history of smoking is evaluated for hemoptysis. 29 A. Etoposide is used in the A computed tomography (CT) scan of the chest shows numerous pulmonary nodules. A treatment of small-cell lung nodule on the pleural surface is selected for CT-guided biopsy by the interventional carcinomas as well as testicular radiologist. The biopsy report is small-cell carcinoma of the lung, and chemotherapy tumors. Its mechanism of action is containing etoposide is started. This drug works by A. Inhibiting topoisomerase II B. related to its ability to inhibit Inhibiting dihydrofolate reductase C. Alkylating double-stranded DNA D. Stabilizing topoisomerase II. Methotrexate microtubules, with resultant mitotic arrest E. Causing DNA chain scission and inhibits dihydrofolate reductase. fragmentation Alkylating agents include mechlorethamine, cyclophosphamide, and ifosfamide. Paclitaxel and docetaxel stabilize microtubules and thereby disrupt mitosis. Bleomycin causes DNA chain scission and fragmentation. 135. A 74-year-old woman who is undergoing chemotherapy for advanced lung cancer 36 B. Oprelvekin has been shown to presents to the infusion center for her next treatment. Before each treatment her white reduce the need for platelet count, emoglobin, and platelet counts are checked to make sure she is not experiencing transfusions following chemotherapy-related cytotoxicity. Her blood sample is run in the analyzer, and her myelosuppressive chemotherapy. platelet count is reported to be at a dangerously low level. Which medication is her Erythropoietin is used for anemia. oncologist likely to prescribe in this situation, along with a platelet transfusion? A. Filgrastim and sargramostim are Erythropoietin B. Oprelvekin C. Filgrastim D. Sargramostim E. Leucovorin used for neutropenia. Leucovorin is used in patients undergoing treatment with methotrexate, to prevent some of its side effects. 136. A 75 year old male diabetic, hypertensive and a diagnosed case of CHF came in due to 472 E Spironolactone, triamterene, edema and dyspnea. If the patient had hypokalemia, a potassium sparing diuretic is and amiloride are all potassium indicated. This potassium sparing diuretic inhibits ENaC - epithelial sodium channel in sparing diuretic. Triamterene and collecting duct. A. Amiloride B. triamterene C. spironolactone D. all of the above E. A and amiloride inhibit the ENaC of B only collecting duct, while spironolactone inhibits aldosterone receptor in collecting ducts. 137. A 75 year old male was brought to the ER due to sudden onset of right sided 285 D. Mannitol is used to decrease the ICP weakness accompanied by a progressive deterioration in sensorium. On PE, BP in patient's with hemorrhagic stroke. It acts was elevated at 220/100. Neurologic exam showed a GCS of 11, an MMT of 2/5 on as an osmotic diuretic thus removes water the both right upper and lower extremities and a positive babinski on the right. A from the intracellular compartment. This CT scan showed a massive intracerebral hemorrhage involving the left basal rapid fluid shifting may cause hyponatremia ganglia. A diuretic was started to decrease the patient's intracranial pressure. and pulmonary edema and as the water is Which of the following is an associated adverse effect of this medication ? A. excreted, hypernatremia may follow. Ototoxicity B. Hyperlipidemia C. Gynecomastia D. Pulmonary Edema E. Thrombocytosis 138. A 100kg male patient had elevated LDL and VLDL. What vitamin is also used as 473 C. antihyperlipidemic drug? A. Vitamin A B. pyridoxine C. niacin D. resin E. Cholestyramine 139. The absorption of most orally administered drug, which occur in the small 699 A. Answer: A Increasing gastric intestine, may be enhanced by A. Taking agents that accelerate gastric emptying emptying time allow drugs to reach the time B. The presence of food C. Stimulation of hepatic microsomal CYP3A4 small intestine sooner, hence increasing the enzymes D. Increasing intestinal motility rate of drug absorption. 140. Administration of most inhaled anesthetics cause a decrease in the following 551 E except: A. Arterial blood pressure B. Brain metabolic rate C. Myocardial function D. Tidal volume E. None of the above 141. Adrenoreceptor blockers such as acebutolol and pindolol are unique for having 266 B. intrinsic sympathetic activity. What is meant by this characteristic? A. has a membrane-stabilizing activity B. has partial agonist activity C. has receptor selectivity D. A and B E. All of the above 142. Adverse events associated with the use of quinolones include all of the following 700 E Answer: E Quinolones inhibit except: A. Tendinitis and possible tendon rupture B. Seizures C. Arthropathy in topoisomerase II (DNA gyrase) & IV children D. Clostridium difficile colitis E. Narrowing of QT interval impairing DNA synthesis. Adverse effects include GI intolerance, antibiotic- asso.colitis, cutaneous rxns, hepatotoxicity, prolongation of QT interval and Achilles tendon rupture. 143. After the oath-taking, you decided to establish a clinic on your province. A 63 655 C. The antidote for organophosphate year-old farmer was brought to you, soaked in sweat and salivating. When you poisoning is Pralidoxime. BUT the golden asked his son about the history, you were told that he was found in that condition period of 6 hours had already passed. And at the middle of the field, which is an 8-hour travel from your clinic, with a bottle of the best medication to give in this case is Malathion on his side. Assuming you have all the following medications in hand, ATROPINE. what is the most appropriate to give? A. Pralidoxime B. Physostigmine C. Atropine D. Scopolamine E. Atracurium 144. . A good drug combination along with thiazides by virtue of its effect on potassium 575 A. Urinary K+ loss can be a problem retention: a. losartan b. amlodipine c. verapamil d. metoprolol with thiazides. Angiotensin converting enzyme (ACE) inhibitors and angiotensin receptor antagonists will attenuate diuretic- induced loss of potassium to some degree, and this is a consideration if a second drug is required to achieve further blood pressure reduction beyond that attained with the diuretic alone. 145. Allen Dimo, is a 38 y/o G8P8(6208), known hypertensive and diabetic, turned out 641 B. Captopril is an ACE inhibitor and its to be pregnant for 5 weeks already. What anti-hypertensive medication should use during gestation may result in IUGR, you AVOID to give to the patient? A. Propranolol B. Captopril C. Hydralazine D. acute fetal renal failure, PDA, and even fetal Methyldopa E. None of the above death. 146. All fo the ff are side effects of lithium therapy EXCEPT? A. Tremors B. 68 B. Lithium decreases thyroid function in Hyperthyroidism C. Nephrogenic diabetic insipidus D. Edema E. Sinus node most patients exposed to the drug and depression some show symptoms of hypothyroidism. Tremors is the most common adverse effect and occurs even on therapeutic doses. 147. All of the following are mechanism of action of propylthiouracil EXCEPT: A. 166 D. SIMILAR TO PREVIOUS BOARD EXAM inhibits thyroid peroxidase-catalyzed reactions B. Blocks iodine organification CONCEPT/PRINCIPLE. Katzung 11th edition C. Blocks coupling of the iodotyrosinase D. Block uptake of iodide by the gland page 672. All of the choices are mechanisms of E. inhibits peripheral deiodination action of PTU except for the blocking of uptake of iodide by the gland. Inhibition of thyroid peroxidase-catalyzed reaction and the blocking of iodine organification. 148. All of the following drugs are considered disease-modifying antirheumatic 169 E ketoprofen is a non selective COX drugs (DMARDs) except: A. Abatacept B. Azathioprine C. Chloroquine D. inhibitor Methotrexate E. Ketoprofen 149. All of the following reactions are considered Phase 2 reactions in drug 175 B. other choices are phase 1 reaction metabolism except: A. glucoronidation B. hydoxylation C. Glutathione conjugation D. sulfation E. Methylation 150. Allopurinol exerts its theraputic effect by: A. Inhibiting excretion of uric acid at 150 D. Allopurinol inhibits the enzyme xanthine the PCT B. Increasing purine catabolism C. Competes with uric acid for oxidase which metabolizes purines to form uric excretion in the kidney D. Inhibiting formation of uric acid E. None of the above acid. Therefore its activity inhibits the formation of uric acid 151. Among insulin secretagogues, the risk of hypoglycemia is least in? A. 321 A. Biguanides are not insulin Meglitinides B. 1st generation sulfonylureas C. 2nd generation sulfonylureas D. secretagogues. Biguanides 152. Among the beta adrenergic blockers, which of the following will less likely to 305 C. pindolol, acebutolol, carteolol, cause bradycardia and increased VLDL concentration? A. metoprolol B. bopindolol, penbutolol,celiprolol are the beta nebivolol C. carteolol D. labetalol E. None of the above blockers having also partial b agonist activity ( intrinsic sympathomimetic activity). The advantage of this property is that it is less likely to cause elevation of plasma lipids without affecting its antihypertensive effect. Katzung 11th ed p 159. 153. Among the following Anti-arrhythmics, which of the following has a highest 410 C. K channel blockers like Sotalol can potential for Torsade de pointes? A. Lidocaine B. Propranolol C. Sotalol D. increase the QT interval thereby increasing the Procainamide E. Verapamil propensity to develop Torsade de Pointes. 154. Among the inhaled anesthetics, nitrous oxide has the _______ MAC and the ______ 215 C. The MAC is a measure of an inhaled potency. A. Lowest, lowest B. Lowest, highest C. Highest, lowest D. Highest, anesthetic's potency. The relationship is inverse. highest E. Intermediate, intermediate 155. Among the thioamide antithyroid drugs, which of the following is the one 165 E SIMILAR TO PREVIOUS BOARD EXAM preferred for pregnant women because it crosses the placental barrier less CONCEPT/PRINCIPLE. Katzung 11th edition readily giving lesser effect to the fetus A. potassium iodide B. Ipodate C. page 672. PTU is preferable because it is more carbimazole D. methimazole E. PTU strongly protein-bound and therefore crosses the placcenta less readily. 156. Ampicillin is eliminated by first-order kinetics. Which of the following 53 B. "First-order" means that the elimination statements best describes the process by which the plasma concentration of rate is proportional to the concentration this drug declines? A. There is only 1 metabolic path for drug elimination B. The perfusing the organ of elimination. The half-life half-life is the same regardless of the plasma concentration C. The drug is is a constant. The rate of elimination is largely metabolized in the liver after oral administration and has low proportional to the rate of administration only bioavailability D. The rate of elimination is proportional to the rate of at steady state. The order of elimination is administration at all times E. The drug is distributed to only 1 compartment independent of the number of compartments outside the vascular system into which a drug distributes. 157. Antibiotic which inhibits transpeptidation by binding to 50s subunit, used in the 678 A. Chloramphenicol is a bacteriostatic drug treatment of meningitis secondary to strep, H.inf, Neisseria neningitidis, causes that stops bacterial growth by inhibiting protein aplastic anemia as its most common idiosyncratic effect? A. Chloramphenicol synthesis. Chloramphenicol prevents protein B. Erythromycin C. Linezolid D. Clindamycin E. None of the above chain elongation by inhibiting the peptidyl transferase activity of the bacterial ribosome. 158. An anticancer agent that is prescribed for melanoma: A. Carmustine B. 455 A. Carmustine is an alkylating agent, cell Cytarabine C. Vincristine D. Vinblastine E. Cisplatin cycle non- specific that is given for melanoma. 159. An anticoagulant which has a more selective effect on factor 671 C. It is an anticoagulant used to prevent and treat deep vein Xa? A. hepanin B. Lepirodin C. enoxaparin D. warfarin E. thrombosis or pulmonary embolism, and is given as a None of the above subcutaneous injection (by a health care provider or the patient). 160. Antifolates sulfamethoxazole and trimethoprim are 436 D. bactericidal by virtue of their synergistic actions. Nonetheless, resistence can develop via plasmid-mediated: A. Decrease in sensitivity of dihydropteroate synthase to sulfonamides B. Increase in PABA synthesis by the organism C. Poor intracellular accumulation of the drugs D. All of the above E. None of the above 161. An antihelminthic agent for strongyloidiasis: A. Nifurtimox B. 456 C. Ivermectin is the drug of choice for strongyloidiasis. It Praziquantel C. Ivermectin D. Diethylcarbamazine E. intensifies GABA-mediated neurotransmission in nematodes and Niclosamide immobilizes parasites. 162. Antihyperlipidemic agent which is contraindicated in patients 672 A. Fibrates and cholestipol increases risk of forming prone to GB stone formation? A. Genifibrozil B. Niacin C. gallstones. Ezetimibe D. Simvastatin E. None of the above 163. These anti-neoplastic agents are cell-cycle specific, except: 353 B. A. 5-fluorouracil B. Cyclophosphamide C. Bleomycin D. Vincristine 164. These are agents or drugs that bind to a different receptor, 121 D. Competitive/Reversible antagonist - bind to receptors in a producing an effect opposite to that produced by the drug it reversible way without activating the effector system; Non- is antagonizing: A. Competitive antagonists B. Irreversible competitive/Irreversible antagonist - cause downward shift of the antagonists C. Chemical antagonists D. Physiologic DRC; Chemical antagonist - interact directly with the drug being antagonists E. Partial agonists antagonized; PHYSIOLOGIC ANTAGONIST is the answer. Examples are histamine and epinephrine, propanolol and thyroid hormone. SIMILAR TO PREVIOUS BOARD EXAM CONCEPT. 165. Aside from its lipid-lowering action, statins are used in 428 B. coronary artery disease because it can also: A. Vasodilate stenotic segments of the coronary B. Stabilize atherosclerotic plaques C. Inhibits platelet aggregation D. Decrease myocardial oxygen demand 166. Aspirin toxicity results in increased respiratory drive that 431 C. Aspirin is not an inhibitor of electron-transport chain (ETC), leads to hyperventilation and respiratory alkalosis. Which of but an uncoupler. Uncouplers of oxidative phosphorylation in these statements is not true about aspirin intoxication? A. It mitochondria inhibit the coupling between the electron transport is achieved when given at a dose of 150 mg/kg. B. It can and phosphorylation reactions and thus inhibit ATP synthesis manifest as a pure high anion gap metabolic acidosis in without affecting the respiratory chain and ATP synthase (H(+)- children. C. It is a known inhibitor of the electron transport ATPase), thus heat is just produced without producing the needed chain. D. All of the abbove E. None of the above ATPs. 167. Assuming that the patient above is NOT pregnant. What is 642 B. ACE inhibitors block the formation of Angiotensin II which is the best medication for her? A. Propranolol B. Captopril C. a potent vasoconstrictor of the afferent and efferent Renal blood Hydralazine D. Methyldopa E. None of the above vessels. When administered to the patient, it prevents diabetic nephropathy by increasing blood flow to the kidneys. 168. An asthmatic patient whose symptoms occur less than once a 698 B. Answer: B Px’s asthma is classified as intermittent. week, lasts a few hours to a few days and has nighttime *Review GINA guidelines symptoms twice a month is best managed with an: A. Inhaled long-acting B2 agonist at H.S. B. Inlahed short-acting B2 agonist PRN C. Inhaled steroid OD D. Oral B2 agonist TID 169. At the E.R., you saw a 24-yr old woman complaining of 697 C. Answer: C Lab picture: N.meningitides. Tx = Pen G 4 million severe, acute, generalized headache. PE revealed T 400C units IV q4h with (+) nuchal rigidity. Her CSF showed gram-neg diplococci & 200 leukocytes/mm3 (98% PMNs). Administration of which of these is the next important management: A. IV sulphonamides B. Intrathecal penicillin C. IV penicillin D. Intratechal chloramphenicol 170. Baby B. Ulate, suffers from infestation of the “unholy 688 B. Answer: B. SeleCtively inhibits miCrotubule synthesis and trinity†of roundworms, namely Ascaris lumbricoides, gluCose uptake in nematodes is the meChanism of aCtion of Trichuris trichiura and Ancylostoma duodenale. What is the Mebendazole, the drug of ChoiCe for AsCariasis (pp. 40 of part2, mechanism of action of the drug suitable for the Pharmacology, Topnotch Handouts). Notes: • Increases patient’s condition? A. Increases permeability to permeability to calcium causing muscle paralysis, vacuolization and calcium causing muscle paralysis, vacuolization and death. death â€" Praziquantel (DOC for Trematodes and Cestodes) • B. Selectively inhibits microtubule synthesis and glucose Immobilizes the parasite and alters their surface structure displacing uptake in nematodes. C. Immobilizes the parasite and them and making them more susceptible to destruction by host alters their surface structure displacing them from the defense mechanisms â€" Diethylcarbamazine (DOC for Filariasis) tissues and making them more susceptible to destruction • Intensifies GABA-mediated neurotransmission in nematodes by host defense mechanisms. D. Intensifies GABA- immobilizing the parasite â€" Ivermectin (DOC for Strongyloidiasis) mediated neurotransmission in nematodes immobilizing • “Unholy trinity†of roundworms â€" Ascaris, Whipworm the parasite. and Hookworms 171. B - Blocker with intrinsic sympathomimetic activity? A. 668 A. Acebutolol and Pindolo has ISA. Acebutolol B. Atenolol C. esmolol D. metroprolol E. None of the above 172. Before intubation, prior to surgery, a 66 year-old male 646 E Succinylcholine is being metabolized by the enzyme Asthmatic was injected with Succinylcholine. He was noted pseudocholinesterase, hence deficiency or lack of this enzyme will to remain apneic and paralyzed for an extended period of lead to extended drug effect of Succinyhlcholine. time. The patient most likely A. Received an excessive dose of the drug B. has been recieving aminoglycosides C. has impaired renal function D. had an allergic reaction E. has pseudocholinesterase deficiency 173. Being an IM resident, you are assigned to watch over a 660 C. Ondansetron is a 5TH-3 anatagonist and is the drug of patient who is undergoing chemotherapy. Which drug is choice for post-chemo vomiting. best to be given to prevent post-chemo vomiting? A. Metoclopramide B. Hyoscine N-Butyl-bromide C. Ondansteron D. Methylergonovine E. Methyrgine 174. The best treatment option for an obese patient with Type 2 459 B. Metformin is a biguanide which is the first-line treatment for Diabetes Mellitus is: A. Glibenclamide B. Metformin C. Type 2 DM, also a drug of choice for obese diabetics. Repaglinide D. Acarbose E. Insulin 175. Beta-lactamase inhibitors are usually given with beta- 610 B. Synergism is giving two active drugs and the combined effect lactam containing antibiotics, such as penicillins. This is is greater than the sum of their effects (1+1=3). Potentiation is the done to produce a favorable drug interaction, specifically: addition of an inactive drug to an active drug, leading to increased A. Synergism B. Potentiation C. Additive D. Antagonism E. effect of the active drug (1+0=3). None of the above 176. Blocks the final common pathway of platelet aggregation 60 B. ASA - inhibits COX; Abciximab - GPIIBIIIA inhibitor; A. ASA B. Abciximab C. Clopidogrel D. Both A and B E. Clopidogrel - ADP receptor inhibitor AOTA 177. The Board of Medicine decided to change the program of 685 C. Answer: C. Loperamide (pp. 713 [.pdf file], Katzung examination for the August 2013 Physician Licensure Pharmacology, 9th edition) â€" must know, MPL 1.0 Examination (PLE) changing the first subject to Pharmacology. Because of this, a non-prepared Non- Topnotch examinee suddenly got an attack of diarrhea while on his way to the testing site. If he will stop at a nearby drugstore to buy an over-the-counter opioid with an antidiarrheal action, he will be asking for A. Codeine B. Dextromethorphan C. Loperamide D. Diphenoxylate 178. By which mechanism does 113 B. Page 19 of Topnotch Handout.ACE inhibitors produce vasodilation by inhibiting the Angiotensin converting enzyme formation of angiotensin II. ACE also breaks down bradykinin (a vasodilator substance). inhibitors cause cough? A. By Therefore, ACE inhibitors, by blocking the breakdown of bradykinin, increase bradykinin inhibiting ACE and formation of levels, which can contribute to the vasodilator action of ACE inhibitors. The increase in angiotensin II B. By blocking the bradykinin is also believed to be responsible for a troublesome side effect of ACE inhibitors, breakdown of bradykinin C. By namely, a dry cough. Angiotensin II also facilitates the release of norepinephrine from facilitating the release of sympathetic adrenergic nerves and inhibits norepinephrine reuptake by these nerves. This norepinephrine from sympathetic effect of angiotensin II augments sympathetic activity on the heart and blood vessels. ACE adrenergic nerves D. By inhibiting inhibitors promote renal excretion of sodium and water (natriuretic and diuretic effects) by norepinephrine reuptake E. By blocking the effects of angiotensin II in the kidney and by blocking angiotensin II stimulation blocking angiotensin II stimulation of aldosterone secretion. This reduces blood volume, venous pressure and arterial pressure. of aldosterone secretion 179. A cancer patient is scheduled for 178 C. dexrazoxane is a rescue therapy for doxorubicin; mesna is for cyclophosphamide; methotrexate chemotherapy. amifostine is for cisplatin Which of the following rescue agent/s is used to alleviate the toxic effect of the drug? A. Dexrazoxane B. Mesna C. Leucovorin D. Amifostine E. None of the above 180. Causes analgesia, amnesia and 676 B. Dissociative anesthesia catatonia with retained conciousness? A. Midazolam B. Ketamine C. Thiopental D. Fentanyl E. None of the above 181. Ceftriaxone is commonly given as 558 C. Organisms NOT covered by cephalosporins are L.A.M.E.: Listeria monocytogenes, an empiric antibiotic in the Atypicals (chlamydia, mycoplasma), MRSA, and Enterococci. treatment of bacterial meningitis. However, it must be noted that this antibiotic will NOT cover the following etiologic agent in meningitis: A. Neisseria meningitidis B. Streptococcus pneumoniae C. Listeria monocytogenes D. Group B streptococcus E. C and D 182. A child was brought to you with 73 D. SIMILAR TO PREVIOUS BOARD EXAM CONCEPT/PRINCIPLE. Among the choices only severe itching due to multiple fexofenadine is a 2nd generation antihistamine. Montelukast is a leukotriene receptor insect bites. The mother requested antagonist. All the others arer 1st gen antihistamines for a non sedating anti histaminic agent. What drug can you give? A. Chlorpheniramine B. Montelukast C. Meclizine D. Fexofenadine E. Hydroxyzine 183. Chlorpromazine may be used not 458 A. Chlorpromazine may be used not only in treating schzophrenia but is also effective in only in treating schzophrenia but reducing nausea and vomiting. is also effective in: A. Reducing nausea and vomiting B. Allergies C. Hypertension D. Treating bipolar disorders E. Sleep disorders 184. Chronic administration of phenobarbital causes metabolic tolerance via 152 C. Stimulation of the microsomal ethanol oxidizing which mechanism? A. Stimulation of aldehyde dehydrogenase B. system is implicated in the development of tolerance Stimulation of alcohol dehydrogenase C. Stimulation of MEOS D. to phenobarbital aside from playing a minor role in Stimulation of Monoamine oxidase E. None of the above individuals with chronic alcoholism. The rest have no role in the development of tolerance to phenobarbital 185. Cimetidine is used as anti-PUD drug and it acts as a/an? A. 653 E Cimetidine is a H2 blocker mucuprotectant B. H1 blocker C. pH neutralizer D. A and B E. None 186. A Class I-B anti-arrhythmic drug which has a high degree of 510 D. Katzung 10th ed., 225 Quinidine- Class IA, effectiveness in arrhythmias associated with acute myocardial associated with cinchonism; Flecainide- Class IC, very infarction. A. Quinidine B. Flecainide C. Propranolol D. Lidocaine E. effective in suppressing premature vebtricular Amiodarone contractions; Propranolol-Class 2, Amiodarone-broad spectrum of actions 187. Clomiphene acts to induce ovulation by: A. Diminishing estrogen 356 A. mediated negative feedback at the pituitary B. Increasing the action of estrogen in the hypothalamus C. Increasing the action of estrogen in the ovary D. Increasing the amount of estrogen receptors 188. Clomiphene is a selective estrogen-receptor modulator commonly used 392 B. C refers to leuprolide when administered in to induce ovulation in patients undergoing assisted reproduction pulsatile fashion. therapy. Which of the following best describes its mechanism of action? A. It potentiates the action of FSH and LH on the ovaries. B. It acts as a partial agonist in the pituitary to block negative feedback from estradiol. C. It acts as a GnRH analogue that increases the release of FSH and LH from the pituitary. D. It inhibits the release of inhibin from ovarian follicles, thus facilitating FSH and LH secretion. E. It is an LH analogue and mimics the physiologic LH surge when given just before ovulation. 189. Clopidogrel exerts its antithrombotic effect through which of the 612 A. Clopidogrel works by irreversibly inhibiting a following mechanisms: A. Irreversible inhibition of ADP receptor B. receptor called P2Y12, an adenosine diphosphate Inhibition of thromboxane synthesis C. Reversible blockade of (ADP) chemoreceptor on platelet cell membranes. glycoprotein IIb/IIIa D. Conversion of plasminogen to plasmin E. Posttranslational modification of vitamin K-dependent clotting factors 190. Cocaine has which of the following mechanism of action? A. Promotes 420 E Adrenergic Synthesis B. Inhibits Adrenergic Storage C. Promotes Adrenergice Release D. Inhibits Adrenergic Metabolism/Degradation E. Inhibits Adrenergic Reuptake 191. Colchicine is a widely used drug in the treatment of gout. Which among 170 A. B. are uricosuric agents like probenecid and the following describes the mechanism of action of the drug? A. Binding sulfinpyrazone… C. allopurinol….D. Febuxostat to intracellular tubulin to excert anti inflammatory effect B. It increases SIMILAR TO PREVIOUS BOARD EXAM the excretion of uric acid through urine C. inhibits xanthine oxidase CONCEPT/PRINCIPLE catalized reactions D. non purine xanthine oxidase inhibitor E. none of the above 192. A college friend consults you regarding the suitability of the medication 684 A. Answer: A. A drug whiCh inhibits the release of given by his doctor for severe hypertension. He complains of postural norepinephrine from sympathetiC nerve endings. The and exercise hypotension (“dizzinessâ€), some diarrhea, and adverse effects described pertains to Guanethidine problems with ejaculation during sexual activity. Given those adverse (pp. 237, 241, 244 and 251 [.pdf file], Katzung effects stated, which of the following mechanisms of action is most Pharmacology, 9th edition). Notes: • A drug which likely related to the drug described by your friend? A. A drug which decreases blood pressure as a result of a decrease in inhibits the release of norepinephrine from sympathetic nerve endings. cardiac output due to its nonselective β-blockade â€" B. A drug which decreases blood pressure as a result of a decrease in Propranolol • An oral vasodilator which relaxes cardiac output due to its nonselective β-blockade. C. An oral smooth muscles of arterioles, thereby decreasing the vasodilator which relaxes smooth muscles of arterioles, thereby systemic vascular resistance â€" Hydralazine • A decreasing the systemic vascular resistance. D. A drug which inhibits the drug which inhibits the converting enzyme peptidyl converting enzyme peptidyl dipeptidase that hydrolyzes angiotensin I dipeptidase that hydrolyzes angiotensin I to to angiotensin II and also inactivates bradykinin. angiotensin II and also inactivates bradykinin â€" Captopril 193. The combination of metronidazole and alcohol will most 131 D. Metronidazole has a disulfiram-effect, so that nausea and likely cause? A. Ataxia B. Blurring of vision C. Dizziness D. vomiting can occur if alcohol is ingested during therapy. Nausea and vomiting E. Pancreatitis (Katzung) SIMILAR TO PREVIOUS BOARD EXAM CONCEPT. 194. A commercial product for colds contain the following: 218 C. Phenylephrine constricts blood vessels through stimulation Phenylephrine, chlorpheniramine, and paracetamol. Which of adrenergic receptors. This acts as a decongestant. acts as the decongestant? A. Paracetamol B. Chlorpheniramine C. Phenylephrine D. Both chlorpheniramine and phenylephrine E. This product does not contain a decongestant. 195. A condition characterized by the acute onset of 573 A. Due to the association with Reye's syndrome, aspirin and encephalopathy, liver dysfunction, and fatty infiltration of other salicylates are contraindicated in children and young adults the liver and other viscera known as Reye's syndrome is less than 20 years old with fever associated with viral illness. associated with what drug? a. Aminosalicylate b. Long chain Reye's syndrome is characterized by the acute onset of fatty acids c. Indomethacin d. nitric oxide encephalopathy, liver dysfunction, and fatty infiltration of the liver and other viscera 196. A condition where there is depletion of needed substrates as 421 C. Idiosyncratic means unusual drug reponse, infrequently a result of continuous activation by a drug and hence observed in most patients; tolerance on the other hand, refers to decrease in responsiveness, such as in chronic nitroglycerin decrease in the intensity of response to drug as a consequence use, is known as: A. Tachyphylaxis B. Idiosyncratic response of continued administration. C. Tolerance D. Resistance 197. Constipation is one of the major side effects of this drug: A. 495 B. Aluminum hydroxide causes constipation (ALang tae). Magnesium hydroxide B. Aluminum hydroxide C. Omeprazole Magnesium hydroxide causes diarrhea (MaGtatae). Colchicine D. Colchicine also causes diarrhea if taken in excess. 198. A COPD patient with severe concomitant heart condition is 274 E Ipratoropium is an anticholinergic used in COPD because of being prescribed a drug that will relieve his dyspnea and less cardiac effects shortness of breath. Which of the following bronchodilators will be the primary choice? A. Salmeterol B. Salbutamol C. Terbutaline D. Atropine E. Ipratropium 199. The danger of long term use of propylthiouracil is: A. 139 D. The most dangerous complication of thioamides is Cholestatic jaundice B. Exfoliative dermatitis C. Liver failure agranulocytosis, (granulocyte <500) an infrequent but potentially D. Agranulocytosis E. Arthralgia fatal adverse reaction. Hepatitis can also be fatal. The most common adverse effect of PTU is maculopapular rash. (Katzung). SIMILAR TO PREVIOUS BOARD EXAM CONCEPT. 200. Dantrolene is the drug of choice to treat malignant 625 A. hyperthermia caused by succinylcholine because? A. It blocks Ca release from sarcoplasmic reticulum B. It induces contraction of skeletal muscle C. It increases the rate of succinylcholine metabolism D. Succinylcholine binding to nicotinic receptors is antagonized by dantrolene E. Dantrolene acts centrally to reduce fever. 201. Dicycloverine is used for: A. Paralytic ileus B. Biliary colic C. 129 C. Dicycloverine is an antispasmodic, prescribed for Intestinal spasm D. Constipation E. Urinary retention gastrointestinal tract spasm and irritable bowel syndrome. It blocks the activity of acetylcholine on muscarinic receptors. It should not be used in patients with PARALYTIC ILEUS, myasthenia gravis, narrow angle glaucoma, enlarged prostate, or pyloric stenosis. Constipation is a side-effect of this drug. SIMILAR TO PREVIOUS BOARD EXAM CONCEPT. 202. Disruption of cell membrane is not seen with A. Amphotericin 48 B. Fluconazole inhibits ergosterol syntehsis B B. Fluconazole C. Nystatin D. Polymxin B E. Griseofulvin 203. Disseminated intravascular coagulation (DIC) is a condition 620 E DIC clinical findings: â†" Fibrinogen, â†' fibrin split products, characterized by widespread stimulation of thrombosis. â†" platelets, â†' PT, â†' PTT, fragmented red blood cells on Which of the following is NOT consistent with the clinical peripheral smear findings in DIC? A. Increased PTT B. Increased PT C. Decreased fibrinogen D. Increased fibrin split products E. None of the above 204. Dissociative anesthesia is effectively achieved by the 214 B. combination of: A. Thiopental, diazepam, ketamine B. Ketamine, nitrous oxide, fentanyl C. Succinylcholine, thiopental, diazepam D. Nitrous oxide, halothane, fentanyl E. Succinylcholine, halothane, thiopental 205. Disulfiram causes extreme discomfort in patients who 512 B. Katzung 10th ed., 371. this was SIMILAR TO PREVIOUS BOARD drink alcoholic beverages by inhibitting which enzyme EXAM CONCEPT/PRINCIPLE resulting to accumulation of acetaldehyde causing flushing, throbbing headache, nausea, vomiting, sweating, hypotension and confusion within few hours of drinking alcohol: A. alcohol dehydrogenase B. aldehyde dehydrogenase C. alcohol decarboxylase D. aldehyde decarboxylase E. none of the above 206. Doxylamine is an ethanolamine H1 antagonist drug 681 C. Answer: C. Vitamin B6 (pp. 389 [.pdf file], Katzung Pharmacology, previously utilized as a possible treatment of nausea 9th edition) Notes: Several H1 antagonist drugs have been studied for and vomiting of pregnancy. Now, this drug is still possible use in treating "morning sickness." The piperazine derivatives available over-the-counter as sleep aid. What B- were withdrawn from such use when it was demonstrated that they have vitamin is a component of Doxylamine? A. Vitamin B2 teratogenic effects in rodents. Doxylamine, an ethanolamine H1 B. Vitamin B3 C. Vitamin B6 D. Vitamin B12 antagonist, was promoted for this application as a component of Bendectin, a prescription medication that also contained pyridoxine. * SIMILAR TO PREVIOUS BOARD EXAM CONCEPT/PRINCIPLE 207. a dreadful side effect, although not common, of this 571 B. Clozapine is notorious for its side effect of agranulocytosis, atypical antipsychotic is agranulocytosis. a. Quetiapine although uncommon b. Clozapine c. Risperidone d. Aripiprazole 208. A drug is 90% cleared by the liver and 10% by the 106 D. Page 7 of Topnotch Handout. Normal creatinine clearance for kidney. The normal dosage of the drug is 500mg/d. If healthy men is 97-137 mL/min. Normal creatinine clearance for healthy this drug is to be given for a patient with a creatinine women is 88-128 mL/min. In this patient his crea clearance is only clearance of 30ml/min, what should be the corrected 30ml/min. Hence to compute for the corrected dose the formula is dose: A. 15 B. 50 C. 450 D. 465 E. 500 Corrected dose= Average dose (aaplied only to the part of the dose cleared by the kidney) x patient's crea clearance/ 100ml/min. In this case 500 x 0.10= 50; 30/100= 0.3; ).3 x 50= 15. Corrected dose is 500 x 0.90=450+ 15=465. 209. the drug of choice for Chagas disease acts through 318 B. the drug of choice for Chagas disease or american what mechanism of action? A. Inhibit mitochondrial trypanosomiasis is nifurtimox. Its anitprotozoan action is due to inhibition electron transport B. Trypanothione reductase of trypanothiane reductase. A- atovaquone MOA ( use for PCP and inhibition C. Suicide substrate of ornithine chloroquine resistant malaria) C- eflornithine ( use in cerebral stage of decarboxylase D. Inhibits enzyyme sulfhydryl groups african trypanosomiasis). D- melarsoprol ( african sleeping sickness). E. none of the above SIMILAR TO PREVIOUS BOARD EXAM CONCEPT/PRINCIPLE Katzung review p 438-439. 210. The drug of choice for clostridium tetani infection 561 B. The use of penicillin (10â€"12 million units IV, given daily for 10 because of the drug's excellent antimicrobial activity days) has been recommended, but metronidazole (500 mg every 6 h or and the absence of the GABA-antagonistic activity a. 1 g every 12 h) is preferred by some experts on the basis of this drug's Penicillin G b. Metrobidazole c. Vancomycin d. excellent antimicrobial activity and the absence of the GABA- Chloramphenicol antagonistic activity seen with penicillin. 211. Drug of choice for hydatid disease? A. Niclosamide B. 186 C. Albendazole is the drug of choice for hydatid disease and Praziquantel C. Albendazole D. Mebendazole E. cysticercosis. Pyrantel pamoate 212. The drug of choice for leptospirosis is: A. Doxycycline 484 B. SIMILAR TO PREVIOUS BOARD EXAM CONCEPT/PRINCIPLE B. Penicillin C. Ceftriaxone D. Erythromycin The rest of the choices are alternate drug choices for leptospirosis. 213. The drug of choice for paroxysmal Supraventricular 124 E SIMILAR TO PREVIOUS BOARD EXAM CONCEPT. tachycardia is: A. Procainamide B. Verapamil C. Amiodarone D. Lidocaine E. Adenosine 214. Drug of choice for partial seizures? A. Diazepam B. 675 D. Diazepam - s. epilepticus Phenobarbital - s. epilepticus Valproic Phenobarbital C. Valproic Acid D. Carbamazepine E. Acid - absence, myoclonic Carbamazepine - partial seizures None of the above 215. Drug of choice for prostatic carcinoma? A. gemcitabine B. Paditaxel C. 680 D. Leuprolide acts as an agonist at pituitary GnRH Cisplatin D. Leuprolide E. None of the above receptors. By interrupting the normal pulsatile stimulation of, and thus desensitizing, the GnRH receptors, it indirectly down regulates the secretion of gonadotropins luteinizing hormone (LH) and follicle-stimulating hormone (FSH), leading to hypogonadism and thus a dramatic reduction in estradiol and testosterone levels in both sexes. 216. The drug of choice for the treatment of bladder and bowel atony is: A. 609 D. Bethanechol is a muscarinic agonist that is Neostigmine B. Metoclopramide C. Pilocarpine D. Bethanecol E. None of administered orally to promote GI and bladder the above motility, usually used in post-op ileus and diabetic neurogenic bladder. 217. Drug of choice for uncomplicated essential hypertension? A. 667 C. Thiazide diuretics are first line of treatment for Furosemide B. Metoprolol C. HCTZ D. Captopril E. None of the above uncomplicated hypertension. 218. These drugs act by preventing the binding of the agonist molecule to 501 A. Katzung 10th ed, p. 11 - Pharmacologic the receptor but do not activate generation of a signal: A. anatgonists bind to the receptor preventing agonist Pharmacologic antagonists B. Inverse agonists C. Competitive agonists binding thereby blocking the agonist's biologic D. Partial agonists E. Orphan drug actions. 219. These drugs increase the outflow of aqueous humor used in glaucoma 193 A. Drugs that increase outflow of aqueous humor: except? A. timolol B. latanoprost C. epinephrine D. pilocarpine E. pilocarpine, physostigmine, latanoprost, epinephrine. physostigmine Drugs that decrease aqueous humor secretion; mnemonics- TAMAD: Timolol, Acetazolamide, Mannitol, Apraclonidine, Dorzolamide. 220. Drugs like quinidine, procainamide and disopyramide are very useful in 449 E Drugs like quinidine, procainamide and treatment of: A. Congestive heart failure B. Hypertension C. disopyramide are very useful in treatment of Thyrotoxicosis D. Status asthmaticus E. Arrhythmia arrhythmia. 221. Drugs which undergo zero order kinetics, except? A. Warfarin B. Heparin 663 C. WHATPET - Warfarin, Heparin, ASA, C. Hydralazine D. Aspirin E. None of the above Tolbutamide, Phenytoin, Ethanol, Theophylline 222. The drug that is used for cardiogenic shock as a last resort. A. 470 D. epinephrine B. dopamine C. dobutamine D. norepinephrine E. Vasopressin 223. A drug used for treatment of advanced prostatic carcinoma by 358 C. Leuprolide combined with an androgen producing continuous gonadal suppression: A. Ketoconazole B. receptor antagonist such as flutamide is the primary Testosterone undecanoate C. Leuprolide D. Cyproterone acetate medical therapy for advanced prostate cancer and is as effective as surgical castration. Reference: Katzung. Basic and Clinical Pharmacology, 11th ed. p. 654 224. A drug with diuretic activity is being studied based on its effects on 271 D. Thiazide increases Na and K excretion and electrolyte levels in the urine. It was found to moderately increase urine causes metabolic alkalosis. It increases Ca NaCl and urine K while decreasing urine Ca with associated slight reabsorption increase in body pH and minimal change in urine HCO3-. This drug has characteristics similar to that of? A. Loop diuretic B. Carbonic anhydrase inhibitor C. K-sparing diuretic D. Thiazide diuretic E. Osmotic diuretic 225. Drug X has greater affinity for albumin than Drug Y. Considering all 213 D. Only the free, unbound drug can interact with other parameters are the same, what significant drug-drug interaction is receptors, exert biologic activity, and be excreted by expected to take place when both drugs are administered the kidneys. simultaneously? A. Drug X will have a higher plasma concentration than Drug Y. B. Drug Y will not be filtered by the renal glomerulus. C. Drug Y will not be available to interact with its receptors in the tissues. D. The free plasma drug concentration of Drug Y will increase. E. Drug X can exert its full biologic activity. 226. Drug X is currently undergoing clinical 376 A. Phase 1- tested on small number of healthy volunteers in order to find the trials with it being tested on a small maximum tolerated dose and prevent severe toxicity; Phase 2- studied on a modest number of volunteers with the goal number of patients with the target disease to test efficacy; Phase 3- evaluated on a being to find the maximum tolerated large number of patients to further establish efficacy and safety, usually a RCT with dose. It is now on what phase of clinical blinding and crossover; Phase 4- post marketing surveillance Katzung 12th ed pg 75 testing? A. Phase 1 B. Phase 2 C. Phase 3 D. Phase 4 E. Phase 5 227. During anaphylaxis, administration of 301 B. physiologic antagonist binds to a different receptor molecules producing an IM epinephrine is essential because of effect opposite to that produced by the drug it antagoniszes. It differs from a its bronchodilatory effect in the lungs. pharmacologic antagonist, which interacts with the same receptors as the drug it is How does epinephrine acts to produce inhibiting. the antagonism of bronchoconstrictor effect of histamine ( through histamine such an effect in this particular receptors) by epinephrine bronchodilator action ( beta receptors). chemical antagonist condition? A. Chemical antagonist B. - reacts directly with the drug being antagonized to remove it or to prevent it from physiologic antagonist C. partial reaching its target. Katzung review 8th ed p 13. agonist D. noncompetitive antagonist E. none of the above 228. Early morning hyperglycemia 677 C. Waning of insulin dose - low dose insulin Dawn Phenomenon - wrong type of characterized by development of insulin Somogyi Effect - high dose of insulin hypoglycemia around 3AM and pre- breakfast hyperglycemia:? A. Waning of insulin dose B. Dawn Phenomenon C. Somogyi Effect D. Combined Waning + Dawn E. None of the above 229. Early morning hypoglycemia is a 415 A. challenge among physicians in treating diabetic patients. When a diabetic patient monitors his Glucose at 3 AM revealing Hypoglycemia with a Hyperglycemic 7 AM CBG, This effect is termed as: A. Somogyi B. Waning C. Dawn D. Dawn and Waning E. Somogyi and Waning 230. Ephedrine can cause increased blood 31 C. Ephedrine acts indirectly to release norepinephrine from nerve terminals, causing pressure by A. Indirect action on effects similar to those of catecholamines, including elevated blood pressure. This cholinergic receptors B. Blockade of potentially dangerous agent has been removed from the OTC market because of an adrenergic receptors C. Stimulation of increasing number of deaths being reported as caused by this agent. An example of an release of epinephrine D. Inhibition of indirect- acting cholinergic agonist is edrophonium, which is used for diagnosis of reuptake of catecholamines E. Direct myasthenia gravis. Some adrenoceptor blockers, such as atenolol, are used for the action on dopamine receptors treatment of hypertension. Catecholamine reuptake inhibition is a property of some antidepressant medications. Dopamine receptor agonists are used in the treatment of Parkinson disease. 231. Escitalopram is an antidepressant agent 197 A. Drugs that belong to SSRI: fluoxetine, sertraline, paroxetine, fluvoxamine, that belongs to which drug class? A. citalopram, escitalopram SSRI B. SNRI C. TCA D. MAOI E. 5HT2 antagonist 232. Excessive use of Nitroprusside may 643 A. Methylene blue result in Methemoglobinemia. The antidote for this condition is? A. Methylene blue B. Silver Nitrate C. 100% oxygen D. Dialysis E. Blood transfusion 233. Factor/s that influence/s passage of 441 E All are faCtors that influenCe passage of drugs through Cell membranes. Lipid drugs through cell membranes: A. solubility is the most important limiting factor for permeation of drugs because cells are Presence of pores in the membrane B. covered by lipid membranes. Amount of protein binding C. Lipid solubility D. pH E. All of the above 234. A few substances that are almost completely inert in the 682 B. Answer: B. Antiflatulent (pp. 1520 [.pdf file], Katzung chemical sense nevertheless have significant pharmacologic Pharmacology, 9th edition) Notes: Some antacid preparations effects. An example of these substances is Simethicone, which is include simethicone, an antiflatulent to relieve symptoms of included in many antacid preparations in order to act as A. bloating and pressure. *SIMILAR TO PREVIOUS BOARD EXAM Antispasmodic B. Antiflatulent C. Antiemetic D. Antiminth CONCEPT/PRINCIPLE 235. Fifith Generation cephalosporin with activity against 59 B. Both A and B are fifth generation but only B has activity Pseudomonas aeruginosa A. Ceftraline B. Ceftabiprole C. against Pseudomonas. Ceftazidime D. Cefepime E. Cefixime 236. Flumazenil may be effective in patients who overdosed on the 553 C. Most S-H drugs facilitate the actions of GABA, a major following sedative-hypnotics except: A. Alprazolam B. Zolpidem inhibitory transmitter. Benzodiazepines potentiate GABA by C. Thiopental D. Midazolam E. Lorazepam increasing frequency of Cl ion channel opening; its action is blocked by flumazenil, a BZ receptor antagonist. Flumazenil, on the other hand, does NOT block the effects of barbiturates. 237. Fluoxetine (prozac) is one of the most commonly use 167 A. SIMILAR TO PREVIOUS BOARD EXAM antidepressants worldwide. Which of the following is the CONCEPT/PRINCIPLE. Katzung 11th edition page 513. SSRIs mechanism of action of the drug? A. Selectively inhibits like Fluoxetine are agents that have their primary action the serotonin reuptake B. Inhibits serotonin and norepinephrine inhibition of serotonin transporter. reuptake C. selectively inhibits reuptake of norepinephrine D. directly antagonize serotonin E. directly antagonize epinephrine 238. Flushing is a unique side effect of Nicotinamide used in 569 B. The cutaneous effects include flushing and pruritus of hyperlipidemia. The drug that may be used to counter this effect the face and upper trunk, skin rashes, and acanthosis nigricans. is: a. Acetaminophen b. Aspirin c. Chlorphenamine d. Arginine Flushing and associated pruritus are prostaglandin-mediated. Taking an aspirin each day alleviates the flushing in many patients. 239. The following antibiotics eliminate intestinal flora thus causing 146 D. Erythromycin is not associated with pseudomembranous colitis except A. 3rd gen cephalosporins B. pseudomembranous colitis Aminoglycosides C. Clindamycin D. Erythromycin E. None of the above 240. The following anti-convulsant is not sedating A. Phenobarbital 51 B. Phenytoin is the oldest non-sedating anti-seizure drug B. Phenytoin C. Clonazepam D. Gabapentin E. NOTA 241. The following anti-tuberculosis drugs are correctly paired with 328 B. Rifampicin inhibits DNA dependent RNA polymerase their mechanisms of action, except: A. INH - inhibits mycolic acid synthesis B. Rifampin - inhibits RNA dependent DNA polymerase C. Ethambutol - inhibits arabinosyl transferase D. none of the above 242. The following are adverse effects of amiodarone except? A. 189 E Adverse effects of amiodarone: bradycardia, heart hyperthyroidism B. hypothyroidism C. photodermatitis D. block, pulmonary fibrosis, elevated liver enzymes, pulmonary fibrosis E. None of the above photodermatitis, corneal microdeposit, halos in peripheral visual field, optic neuritis, hypothyroidism (blocks the peripheral conversion of T4 to T3) and hyperthyroidism (because it is also a potential source of large amounts of inorganic iodine) 243. The following are bactericidal, except: A. TMP-SXZ B. 327 E All drugs stated are bactericidal. TMP and SXZ alone Vancomycin C. Metronidazole D. Cefalexin E. none of the above are bacteriostatic but are bactericidal when in combination. 244. The following are true of sulfonamides except: A. Sulfonamides are structural 519 B. It is interesting that rickettsiae are not analogs of p-aminobenzoic acid and act by inhibitting dihydropteroate synthase inhibitted by sulfonamides but are actually and folate production. B. Sulfonamides are very effective against rickettsiae. C. stimulated in their growth. Katzung 10th ed., Combination of a sulfonamide with an inhibitor of dihydrofolate reductase 763. provides synergistic activity because of sequential inhibition of folate synthesis. D. trimethoprim-sulfamethoxazole is the drug of choice for infections such as Pneumocystis jiroveci pneumonia. E. Sulfadiazine with pyrimethamine is first-line therapy for treatment of acute toxoplasmosis. 245. The following drugs accelerate substrate metabolism causing a decrease in the 690 D. Answer: D. Cimetidine (pp. 8 [part 1], pharmacologic action of the co-administered drug, except; A. Griseofulvin B. Pharmacology, Topnotch Handouts; pp. 77 Rifampicin C. Ethanol D. Cimetidine [.pdf file], Katzung Pharmacology, 9th edition). The question defines a CYP450 Inducer. The only CYP450 inhibitor among the choices is Cimetidine. 246. The following drugs are included in the ABVD regimen for Hodgkin’s 689 A. Answer: A. Vinblastine (pp 41 [part 1], Lymphoma, except: A. Vinblastine B. Doxorubicin C. Dacarbazine D. Oncovin E. Pharmacology, Topnotch Handouts). Notes: Bleomycin A â€" Adriamycin (Doxorubicin) B â€" Bleomycin V â€" Vincristine (Oncovin) D â€" Dacarbazine 247. The following drugs' efficacy is directly related to time above the minimal 326 D. What is described is the time- inhibitory concentration and becomes independent of concentration once the dependent killing property for which MIC has been reached, except? A. Penicillins B. Cephalosporins C. A & B D. none penicillins and cephalosporins are known. of the above Aminoglycosides are known for their concentration-dependent killing. 248. The following drugs have concentration dependent killing, except A. 43 C. Concentration dependent or dose- Aminoglycosides B. Fluoroquinolones C. Penicillin D. Metronidazole E. NOTA dependent killing means that the higher the concentration, the greater the bactericidal effect. Examples are aminoglycosides, fluoroquinolines, daptomycin, metronidazole, streptogramins. Penicillin employs time-dependent killing 249. The following drug/s is/are used to augment labor: A. Oxytocin B. Misoprostol C. 359 A. Methyl ergonovine D. All of the above 250. The following drugs promote induction of liver CYP enzymes except ? A. 303 D. CYP inducers enhances drug Rifampin B. Carbamazepine C. Phenylbutazone D. Disulfiram E. Glutethimide metabolism. Ex. Benzopyrene, carbamazepine, chlorcyclizine, glutethimide, griseofulvin, phenobarbital, phenylbutazone, phenytoim, rifampin, ritonavir ( chronic administration, acute -acts as inhibitor), st johns wort. Katzung 11th ed p 64-65. 251. The following has increased bioavailabity when taken with food, except A. 42 E All of the choices has increased Cefuroxime B. Fusic acid C. Nitrofuratoin D. Griseofulvin E. NOTA bioavailability when taken with food. 252. The following helminths are matched with their respective drug of choice except: 513 C. The drug of choice in strongylodiasis A. Fasciola hepatica - Bithionol B. Echinococcus granulosus - Albendazole C. and onchocerciasis that acts by intensifying Stongyloides stercoralis - Thiabendazole D. Schistosoma mansoni - Praziquantel GABA- mediated transmission of signals in E. Dracunculus medinensis - Metronidazole peripheral nerves resulting to paralysis of the worms is IvermeCtin. Thiabendazole is an alternative drug only. Katzung 10th ed., 870; See also p.868 for the table of drug of choice for other helminthic infections 253. The following laxative mechanisms are correctly paired with its representative 337 D. Polyethylene glycol is an osmotic laxative drug, except: A. bulk forming - psyllium B. stool softening - docusate C. laxative. Examples of stimulant laxatives osmotic - lactulose D. stimulant - polyethylene glycol include senna, bisacodyl, castor oil. 254. The following statements are true of Aminoglycosides, 409 A. EXCEPT: A. Act by Time-Dependent Killing Action B. As the plasma level is increased above the MIC an increasing proportion of bacteria are killed and at a more rapid rate C. Has a post-antibiotic effect D. Requires Oxygen for uptake E. Amikacin has the widest coverage 255. Formation of methyltransferases that alter drug binding sites 556 D. Question could be rephrased into: which acts on the 50s on the 50S ribosomal subunit is the primary mechanism of subunit? Macrolides such as azithromycin do. resistance to which antibiotic: A. Streptomycin B. Tetracycline C. Amikacin D. Azithromycin E. Levofloxacin 256. For the treatment of brain tumors, it is important for a drug 19 B. Procarbazine, Dacarbazine, Nitrosureas (Lomustine, to be able to penetrate the blood brain barrier. Which of the Carmustine, Streptozocin) are chemotherapeutic agents which are following antineoplastic agents cannot cross the blood brain able to penetrate the blood brain barrier. Source: Topnotch barrier? A. Procarbazine B. Dacarbazine C. Streptozocin D. Handout on Pharmacology Carmustine E. 5-FU 257. Furosemide is a loop diuretic that is known for its ototoxicity. 658 D. Kanamycin is an aminoglycoside is very ototoxic Which among the following drugs should you AVOID administering it simultaneously? A. Penicillin B. Ceftriaxone C. Clindamycin D. Kanamycin E. Erythromycin 258. A fusion inhibitor used in the treatment of HIV infection a. 578 C. Efavirenz b. Lamivudine c. Enfuvirtide d. Atazanavir 259. A G7P7 (6107) came in for consult. She asks you that she 173 E Rifampicin is a CYP450 inducer which hastens the excretion wants to take oral contraceptive pills as family planning. If of OCP given in conjunction with OCP, which among the following will decrease the effectivity of OCP? A. Isoniazid B. Cimetidine C. ketoconazole D. erythromycin E. Rifampicin 260. Given a hypothetical species of bacteria that is resistant to 608 A. Chloramphenicol inhibits peptidyl transferase. Clindamycin antibacterial agents that prevent translocation during and erythromycin inhibits translocation. translation, which of the following can be used effectively for this bacterial infection? A. Chloramphenicol B. Clindamycin C. Erythromycin D. A and B E. B and C 261. High dose statins are given during ACS, the MOA of which in 45 B. these cases is: A. Inhibit HMG-CoA reductase B. Stabilize plaque C. Reduce TAG synthesis D. Increase HDL E. AOTA 262. Highly leukogenic anti-neoplastic drugs A. Topoisomerase II 49 D. Toposiomerase II inhibitors and alkylating agents are the inhibitor B. Alkylating agents C. Anthrcycline antibiotics D. most leukogenic anti-neoplastic agents. Both A and B E. AOTA 263. a highly lipophilic derivative of obiquinone that has activity 576 A. Atorvaquone is promising synthetic derivative with potent against plasmodium species and Pneumocystis pneumonia activity against Plasmodium species and the opportunistic among patients intolerant to Co-trimoxazole: a. pathogens Pneumocystis carinii and Toxoplasma gondii. After Atorvaquone b. Quinine c. Proguanil d. artemisinin limited clinical trials, the FDA approved this compound in 1992 for treatment of mild-to-moderate P. carinii pneumonia in patients intolerant to trimethoprim-sulfamethoxazol 264. Hoffman elimination is exhibited by: A. Succinylcholine B. 205 E Hoofman elimination aka exhaustive methylation Diazepam C. Phenobarbital D. Phenytoin E. Atracurium 265. Hydralazine lowers BP by causing arteriolar smooth muscle 425 D. relaxation. This effect is achieved by its ability to alter metabolism of which ion? A. Sodium B. Potassium C. Magnesium D. Calcium 266. A hypertensive patient was found to have elevated 616 D.Fibrates are indicated for hypertriglyceridemia. Fibrates triglycerides, normal LDL and decreased HDL levels. Which typically lower triglycerides by 20% to 50%. Level of the good of the following is most efficacious for this patient? A. cholesterol HDL is also increased. Fibrates may decrease LDL, Simvastatin B. Ezetimibe C. Colestipol D. Fenofibrate E. though generally to a lesser degree than statins. Orlistat 267. If a patient has COPD, which of the following beta-blockers would be more 491 D. Atenolol, metoprolol, esmolol, preferrable to use, theoretically? A. propranolol B. carvedilol C. labetalol D. atenolol betaxolol, and acebutolol (A-BEAM) are more beta1-selective hence are advantageous in patients with comorbid pulmonary disease. 268. If a single dose of a known drug with first order elimination is given intravenously, 261 D. Review first-order elimination. how long will it take for 75% of the drug to be eliminated if the half-life is 1.5 hours? A. 50% is one half-life, 75% is 2 X half-life 1.5 hours B. 2 hours C. 2.5 hours D. 3 hours E. 6 hours and so on.. 269. If the plasma concentration of a drug declines with “first order kinetics,†this 602 A. First Order Kinetics : elimination means that: A. The halflife is constant regardless of plasma concentration B. The drug of the Drug is directly proportional to is largely metabolized in the liver after oral administration and has low bioavailability its plasma concerntration. its dependent C. The rate of elimination is proportionate to the rate of administration at all times D. on its half life, which remains constant. The drug has a high volume of distribution E. The drug is eliminated with a constant First order implies that no matter how concentration regardlesss of the plasma concentration much concerntration of the Drug u give it will be eliminated 50% by its first half- life 270. If you need to give a skeletal muscle relaxant to a burn patient, you will AVOID 277 E Succinylcholine is contraindicated prescribing which of the following drugs? A. Pancuronium B. Baclofen C. Dantrolene D. in patient with rhabdomyolysis, Vecuronium E. Succinylcholine hyperkalemia, burn patients, malignant hyperthermia 271. In a Diabetic patient with impaired renal function which of the following drugs is 417 B. Before prescribing 2nd contraindicated? A. Nateglinide B. Glipizide C. Metformin D. Pioglitazone E. Acarbose generation Sulfonylureases liver and hepatic function must be checked first because they are contraindicated. 272. In anticoagulation, warfarin therapy is usually overlapped with heparin for the first 1-2 217 D. Warfarin acts by vitamin K days. Why is this so? A. To achieve supraoptimal anticoagulation during critical anatagonism. The initial prothrombotic periods of illness as warfarin and heparin have synergistic effects. B. To prevent effect of warfarin is because the bleeding as heparin partially counteracts warfarin's hemorrhagic property. C. synthesis of protein S, an anticoagulant, Warfarin is metabolized slowly thus leading to a delay in anticoagulation if heparin is also depends on vitamin K. Thus, not also given. D. To compensate for warfarin's initial prothrombotic property. E. protein S levels decline initially and the Heparin decreases the clearance of warfarin thus achieving greater plasma drug effect of protein C (for which it serves concentration of warfarin. as a co-factor) is also diminshed. 273. In a patient suffering from organophosphate poisoning with a cardiac rate of 45, what 206 B. Because of its antimuscarinic drug should be given? A. Neostigmine B. Atropine C. Epinephrine D. Pancuronium E. effect on the heart, atropine (the Edrophonium antidote for organophosphate poisoning) has a positive chronotropic effect. 274. In a patient with Grave's Disease this drug is given to decrease peripheral conversion 416 A. of T4 to T3: A. Propranolol B. I 131 C. Methimazole D. PTU E. Lugol's Iodine 275. In calculating the loading dose of a drug, which of the following parameters are not 405 D. LD = (Vd x DPC) / Bioavailability necessary? A. Volume of distribution B. Desired Plasma Concentration C. Bioavailability D. Clearance E. None of the Above 276. incidence of Reye syndrome among patients less than 20 who took Aspirin concurrent 574 A. Less than 0.1% of children who with influenza A/B infection: a. 0.1% b. 1% c. 5% d. 10% took aspirin developed Reye syndrome, but more than 80% of patients diagnosed with Reye syndrome had taken aspirin in the past 3 weeks 277. In dermatologic pharmacology, approximately how many grams of topical 579 C. An amount of topical medication preparation is needed to cover the body surface? a. 10g b. 20g c. 30g d. 40g sufficient to cover affected body surfaces in repeated applications must be dispensed to the patient. A general rule is that approximately 30 g is required to cover the body surface. 278. Injection of a small dose of Ach will cause what? A. 158 D. Ach causes depression of the SA and AV nodes thus slowing the Reflex bradycardia B. Hypotension C. Splanchnic heart rate, hypotension through the relaxation of arterioles, splanchnic stimulation D. All of the above E. None of the above stimulation through the stimulation of the muscarinic receptors 279. Intake of potassium supplements is contraindicated in 157 B. Amiloride is a potassium sparing diuretic which therefore spares patients taking which of the following? A. Ethacrynic renal excretion of potassium raising the potassium levels and therefore is acid B. Amiloride C. Hydrochlorothiazide D. contraindicated in patients taking potassium supplements. Acetazolamide E. None of the above 280. In the acute settings the following drugs are useful 594 D. Propylthiouracil, iodides and propanolol are the first line drugs for the emergent management of thyroid storm for thyroid storm. Hydrocortisone may be added for realtive adrenal EXCEPT? A. Propanolol B. KI C. Propylthiouracil D. insufficiency in the acute attack. Methimazole may be used, but is not Methimazole E. Hydrocortisone usually given in the acute therapy of thyroid storm and PTU is preferred. 281. In the case above, which of the following patient's 604 C. All the other side effects of danazol is reversible, except for manifestations may be IRREVERSIBLE? A. Acne B. deepening of the voice. Male-pattern hair growth C. Deepening of the voice D. All of the above E. B and C 282. In the CHOP regimen for non-Hodgkin's lymphoma, 202 C. Oncovin is vincristine. which chemotherapeutic drug acts by inhibiting the mitotic spindle? A. Cyclophosphamide B. Hydroxyurea C. Oncovin D. Prednisone E. Doxorubicin 283. In the kidney, which one of the following is inhibited 626 A. Thiazide diuretics inhibit sodium chloride reabsorption in DCT. by thiazide diuretics? A. Sodium chloride reabsorption in the early part of the distal convuluted tubule B. Water removal from intracellular space by osmosis C. Reabsorption of Calcium D. Aldosterone action on the nephron E. Excretion of chloride 284. In the stomach, aspirin will exist predominantly as the 201 B. Review the Henderson-Hasselbach equation. Aspirin ______________ form. A. Hydrophilic, ionized, polar B. (acetylsalicyclic acid) is acidic in nature. In an acidic environment like the Lipophilic, unionized, non-polar C. Hydrophilic, stomach, it will exist predominantly as the lipid-soluble, unionized, non- unionized, non-polar D. Lipophilic, ionized, polar E. polar form. Hydrophilic, unionized, polar 285. In the treatment of CHF, this drug is characterized as 695 B. Answer: B Dobutamine is a relatively b-1 selective adrenoreceptor having a selective beta-1 antagonistic activity and agonist that causes peripheral vasodilation in some vascular beds useful devoid of dopaminergic activity: A. Fenoldopam B. for many cases of acute heart failure. Dobutamine C. Dopamine D. Isoproterenol 286. In the treatment of Parkinson's disease, what drug is 210 C. Carbidopa peripherally inhibits the enzyme DOPA decarboxylase, given with levodopa to increase the drug's the enzyme that converts DOPA to dopamine. Dopamine itself is unable bioavailability? A. Entacapone B. Selegiline C. to cross the blood-brain barrier but DOPA can. Inhibition of the enzyme Carbidopa D. Fluoxetine E. Pyridoxine results in more DOPA available to enter the CNS. Within the CNS, DOPA is converted to dopamine by the same enzyme. However, it is not inhibited by carbidopa at this location. 287. In this clinical phase of drug development, the drug is 442 C. Phase 1 trial involves evaluation of dose-response relationship and studied in patients with the target disease to pharmacokinetics in normal volunteer patients with target disease. Phase determine its efficacy: A. Phase 0 B. Phase 1 C. Phase 2 trial involves determination whether the agent has the desired efficacy 2 D. Phase 3 E. Phase 4 at doses that are tolerated by sick patients. Phase 3 trial is a large design including placebo and positive controls in a double-blind crossover design. Phase 4 trial is a postmarketing surveillance. 288. In this phase of clinical trials, the drig is evaluated in a 664 B. I - normal volunteers, pharmacokinteics and dose sensitive acute moderate number of target patients to test dose effects II - moderate number of PATIENTS, dose efficacy III - large # of efficacy of the drug? A. I B. II C. III D. IV E. None of the patients, toxicities IV - post - marketing, idiosyncratic reactions above 289. In what phase of the cell cycle does vinca alkaloid 239 E Vinca alkaloids ( vinblastine, vincristine, vinorelbine) blocks the exerts its effect? A. G0 B. G1 C. S D. G2 E. M formation of mitotic spindle by preventing the assembly of tubin dimers into microtubules. Therefore it acts on the M phase. 290. In which of the following drugs used for the 388 C. SIMILAR TO PREVIOUS BOARD EXAM CONCEPT/PRINCIPLE. treatment of gout is cataract an important side effect? A. Colchicine B. Probenicid C. Allopurinol D. Febuxostate E. Indomethacin 291. In which of the following penicillins is 398 B. SIMILAR TO PREVIOUS BOARD EXAM CONCEPT/PRINCIPLE. interstitial nephritis a known adverse effect, partly leading to a decline in its popularity? A. Nafcillin B. Methicillin C. Carbenicillin D. Bacitracin E. Metampicillin 292. IOP can be decreased by increasing aqueous 52 B. PG analogues decrease IOP by increasing aqueous outflow. CA inhibitors outflow with the following drugs A. Timolol B. and Beta-blocker decrease aqeuous production. Alpha2 agonist inreases Latanoprost C. Mannitol D. Dorzolamide E. outflow and decreases production. NOTA 293. Isoniazid is the most important drug used in 437 B. TB. It inhibits mycolic acid synthesis and is therefore bactericidal. Possible adverse effects include neuritis, insomnia and seizures. To prevent neurotoxicities, patient should be supplemented with: A. Folic acid B. Pyridoxine C. Vitamin A D. Riboflavin 294. It is a drug that is used almost solely as an 448 B. Spectinomycin is an aminoglycoside that inhibits protein synthesis by alternative treatment for gonorrhea in binding to 30s subunit. It is the drug of choice of drug-resistant gonorrhea and patients whose gonococci are resistant to for gonorrhea in penicillin-allergic patients. other drugs is: A. Docycycline B. Spectinomycin C. Ofloxacin D. Azithromycin E. Tetracycline 295. It is a monoclonal anti-IgE antibody used in the 499 D. Trastuzumab aka herceptin is a monoclonal antibody that interferes with treatment of asthma resistant to inhaled the HER2/neu receptor, rituximab is against the protein CD20 on B cells, and steroids and long-acting B2-agonists: A. infliximab is against TNF-alpha. trastuzumab B. rituximab C. infliximab D. omalizumab 296. It is an anticonvulsant that is effective for 450 B. Gabapentin is an anticonvulsant drug, a GABA derivative which blocks neuropathic pain: A. Valproic acid B. calcium channels, increases GABA release, very useful in neuropathic pain also in Gabapentin C. Phenytoin D. Duloxetine E. postherpetic neuralgia. Levetiracetam 297. It is a very efficacious orally active arteriolar 509 A. Katzung 10th ed, 173. vasodilator that is also available in topical preparations used to stimulate hair growth for correction of baldness. A. Minoxidil B. Sodium Nitroprusside C. Hydralazine D. Diazoxide E. Fenoldopam 298. It is during this phase of drug development 504 A. Phase 1 trials determine the probable limits of the safe clinical dosage and testing that the effects of the drug as a range in a small number of healthy volunteers. In Phase 2, the drug is studied in function of dosage are established in a small patients with the target disease to determine its efficacy. In phase 3, the drug is number of healthy volunteers. A. Phase 1 B. evaluated in larger numbers of patients with the target disease to further Phase 2 C. Phase 3 D. Phase 4 E. Phase 5 establish safety and efficacy. Phase 4 constitutes monitoring the safety of the new drug under actual conditions of use in large number of patients. Katzung 10th ed., 70-71. 299. It is the antiarrythmic of choice for termination 162 C. SIMILAR TO PREVIOUS BOARD EXAM CONCEPT/PRINCIPLE. Katzung of ventricular tachycardia and prevention of 11th edition page 239. "Lidocaine is the agent of choice for termination of ventricular fibrillation after cardioversion in ventricular tachycardia and prevention of ventricular fibrillation after the setting of acute ischemia. A. adenosine B. cardioversion the setting of acute ischemia. However, routine prophylactic use esmolol C. lidocainne D. procainamide E. of lidocaine in this setting may actually increase total mortality, possibly by Amiodarone increasing incidence of asystole, and is not the standard of care. Most physicians administer IV lidocaine only to patients with arrythmia" 300. It is the flow or transfer of a drug to the bloodstream. A. 467 C. Absorption is the transfer of a drug to the distribution B. bioequivalence C. absorption D. bioavailability E. bloodstream. Distribution is the entry of a drug to the target Elimination organ. Elimination is the termination of drug action. Bioavailability is the fraction of a drug that reaches the systemic circulation. Bioequivalence is the comparability of 2 related drugsnto achieve peak blood concetration. 301. It is the major second messenger of beta receptor activation that 446 B. cAMP is the major second messenger of beta participates in signal transduction: A. Inositol triphosphate B. receptor activation that participates in signal transduction. cAMP C. cGMP D. Calcium E. Adenylyl cyclase 302. It is the only licensed chemotherapy drug for advanced 487 D. SIMILAR TO PREVIOUS BOARD EXAM melanoma: A. adriamycin B. bleomycin C. cisplatin D. Dacarbazine CONCEPT/PRINCIPLE There was quite a number of questions on oncology drugs. 303. Jan Deo is a newly diagnosed with essential hypertension. He has 163 B. Enalapril is an ACE inhibitor. One of the many no other co morbidities. Aside from lifestyle modification, he was alternative names for ACE is peptidyl dipeptidase. SIMILAR prescribed with Enalapril as his maintainance. Which of the TO PREVIOUS BOARD EXAM CONCEPT/PRINCIPLE following BEST describes the mechanism of action of the drug. A. blocks Angiotensin receptor which will prevent rise in BP B. inhibits peptidyl dipeptidase C. direct vasodilator D. reduces calcium uptake of smooth muscles E. none of the above 304. Ketamine exerts its anaesthetic effect by its inhibitory action on 346 A. these receptors: A. NMDA B. GABA-A C. Glycine D. Neuronal nictonic acetylcholine receptors 305. Knowledge of potency and solubility is important in general 280 A. Low MAC - high potency; High MAC - low potency; anesthetic medications. Which of the following characteristics Low blood:gas coefficient - low solubility; High blood:gas refers to high potency? A. Low minimum alveolar concentration B. coeeficient - high solubility High minimum alveolar concentration C. Low blood:gas partition coefficient D. High blood:gas partition coefficient E. None of the above 306. KVM, a 29 year old female was brought to the ER due to ingestion 160 D. Pralidoxime regenerates acetylcholinesterase and is of insecticide, you ordered pralidoxime to be given, what does it only effective when the bond between the organophosphate do? A. Induces the enzyme Acetylcholinesterase B. Regenerates and acetylcholinesterase hasn't matured which takes 6-8 acetylcholinesterase if given within 6-8 hours after ingestion C. hours, afterwhich, pralidoxime even if given is no longer Breaks the bond between organophosphate and effective. acetylcholinesterase D. B and C only E. All of the above 307. Lactulose is the most frequently administered laxative in post-MI 615 B. Lactulose is an osmotic agent. Bulk forming agents patients. It is classified as: A. Bulk-forming laxative B. Osmotic include insoluble fibers (psyllium). Stool softeners include laxative C. Stool softener D. Emollient E. None of the above docusate Na. Emollients include mineral oil. 308. The least nephrotoxic among the aminoglycosides is: a. 577 D. The nephrotoxic potential varies among individual Tobramycin b. Kanamycin c. Amikacin d. Streptomycin aminoglycosides. The relative toxicity correlates with the concentration of drug found in the renal cortex in experimental animals. Neomycin, which concentrates to the greatest degree, is highly nephrotoxic in human beings and should not be administered systemically. Streptomycin does not concentrate in the renal cortex and is the least nephrotoxic 309. Leukotrienes are among the most potent bronchoconstrictors 547 B. Zileuton is a lipooxygenase inhibitor. implicated in the pathogenesis of bronchial asthma. Which of the following is a leukotriene receptor inhibitor that has found clinical application in asthma prophylaxis? A. Zileuton B. Zafirlukast C. Ipratropium D. Budesonide E. Bosentan 310. Loca is a 23 year old G4P4 (4004) who has been on oral contraceptives for 8 503 E Rifampin revs up the cytochrome months now. What is the expected drug-drug interaction If she was to start on an P450 enzyme system thereby enhancing the anti-tuberculosis drug that acts by inhibitting RNA synthesis and which imparts metabolism and hence the elimination of the orange color to urine and sweat? A. Enhanced efficacy of the oral contraceptive oral contraceptive the patient is taking she is taking B. Enhanced potency of the oral contraceptive she is taking C. resulting to its significantly lower serum reduced activity of the cytochrome P450 isoforms D. Increased elimination of the level. Katzung 10th ed, p.774. anti-tuberculosis drug E. Significantly lower serum level of the oral contraceptive 311. Local anesthetics are not effective in inflamed infected tissues because? A. The 188 E Local anesthetics are weak bases, cationic form is increased B. Protonated form predominates C. The drug is more therefore, in an acidic environment like in water soluble D. A and B E. All of the above infected tissues, the protonated or cationic form of the drug predominates. This is water soluble so the drug undergoes rapid clearance rather than being able to cross biological membranes. 312. Local anesthetics block nerve conduction through inhibition of what electrolyte? 570 B. Local anesthetics block conduction a. Potassium b. Sodium c. Calcium d. All of the above by decreasing or preventing the large transient increase in the permeability of excitable membranes to Na+ that normally is produced by a slight depolarization of the membrane 313. The main indication of this agent is hypertension in pregnancy: A. Reserpine B. 349 C. Prazosin C. Methyldopa D. Clonidine 314. majority of phase2 reactions involving Glucoronic acid conjugation are found in 567 A. conjugating enzymes, notably the what cell compartment? a. ER b. cytosol c. Mitochondrial matrix d. Golgi bodies UGTs, are all located in the endoplasmic reticulum of the cell 315. the major systemic side effects of systemic corticosteroids are much more likely 230 A. Katzung Review of pharmacology to occur if at least the duration of treatment is more than_____? A. 14 days B. 4 8th ed pp 171. . weeks C. 3 weeks D. 20 days E. 2 months 316. A man is being treated for intestinal strongyloidiasis, you know that the drug of 155 B. Ivermectin is the DOC in the choice is: A. Praziquantel B. Ivermectin C. Melarsoprol D. Pyrimethamine-sulfa E. treatment of strongyloidiasis Trimethoprim-sulfamethoxazole 317. Marc, an asthmatic, is on theophylline. He has a number of other medical 481 A. Theophylline has a narrow conditions. Which of the following medications may be safely administered with therapeutic window and drug interactions theophylline? A. Penicillin B. Erythromycin C. Phenytoin D. Rifampicin can lead to severe morbidity. Erythromycin decreases the clearance of theophylline and increases its serum levels. Phenytoin and rifampicin have the oppositve effect. Advice: Memorize the inducers and inhibitors of the CYP450 system! 318. The mechanism of action of Allopurinol : A. Inhibits microtubule assembly B. 133 D. Option A and B - MOA of Colchicine; Decreases macrophage migration and phagocytosis C. Compete with uric acid for Option C and E - MOA of Probenecid; MOA reabsorption in the proximal tubules D. Irreversibly inhibits xanthine oxidase E. of Allopurinol: its active metabolite Increases uric acid excretion irreversibly inhibits xanthine oxidase and lowers production of uric acid. SIMILAR TO PREVIOUS BOARD EXAM CONCEPT. 319. Mechanism of action of chloroquine except:? A. Suppression of B-Lymphocyte 673 A. Chloroquine suppress T - response to mitogens B. Free-Radical Trapping C. Lysosomal enzyme stabilization lymphocyte response D. Decreased leukocyte chemotaxis E. None of the above 320. Mechanism of action of cyanide poisoning? A. Inhibits complex III of ETC B. 670 D. Complex I - amytal, rotenone; Uncoupler C. Inhibits oxidative bursts D. Inhibits complex IV of ETC E. None of the Complex II - malonate; Complex III - above antimycin A, Dimercaprol; Complex IV - CN, CO, Na azide; Complex V - Oligomycin Uncoupler - ASA, 2,4 - DNP 321. Mechanism of action of diazepam? A. Increases 674 B. Increases frequency of channel opening duration of Cl- channel opening B. Increases frequency of Cl- channel opening C. Agonist at 5HT1a and D2 receptors D. Blocks voltage gated Na+ channels E. None of the above 322. Mechanism of action of mebendazole? A. Inhibits 679 A. Mebendazole causes degenerative alterations in the tegument and microtubule assembly B. Inhibits glucose uptake in intestinal cells of the worm by binding to the colchicine-sensitive site of rematudes C. Causes dipolarization induced tubulin, thus inhibiting its polymerization or assembly into microtubules. The paralysis D. Increase permeability of membrane to loss of the cytoplasmic microtubules leads to impaired uptake of glucose Ca2+ causing muscle paralysis E. None of the by the larval and adult stages of the susceptible parasites, and depletes above their glycogen stores. 323. The mechanism of action of Nedocromil: A. 132 B. Option A - Beta 2 agonist (Salbutamol, Terbutaline); Option C- Activates beta-2 receptors in bronchial smooth leukotriene synthesis inhibitor (Zileuton); Option D - Muscarinic receptor muscle B. Prevents calcium influx and stabilizes antagonist (Ipratropium); Option E - Leukotriene antagonist (Zafirlukast). mast cells preventing release of histamine C. Cromolyn, Nedocromil, and Lodoxamide are mast cell stabilizers, which Inhibits 5-lipoxygenase D. Prevents vagal- acts by preventing calcium influx and stabilizes mast cells, preventing stimulated bronchoconstriction E. Blocks cysteinyl degranulation and release of histamine, leukotrienes, and other mediators. leukotriene-1 receptor SIMILAR TO PREVIOUS BOARD EXAM CONCEPT. 324. The metabolism of which drug is inhibited by 333 D. Allopurinol inhibits the enzymatic inactivation of 6- MP and its allopurinol and febuxostat? A. Cytarabine B. derivative, Azathioprine, by xanthine oxidase. Thus, when allopurinol is used Methotrexate C. Fluorouracil D. Mercaptopurine concurrently with oral 6-MP/azathioprine, dosage of the antineoplastic agent must be reduced to 1/2 to 1/3 of usual dose. 325. Methyldopa is a drug of choice in treating pre- 424 C. Activation of alpha-2 adrenergic receptors by methyldopa eclampsia. Its ability to lower blood pressure is metabolites in the brainstem appears to inhibit sympathetic nervous system based on its action of: A. Activating alpha-1 output and lower blood pressure. This is also the mechanism of action of adrenergic receptors B. Inhibiting alpha-1 clonidine. Activation of alpha-1 adrenergic receptors meanwhile, stimulates adrenergic receptors C. Activating alpha-2 vasoconstriction and hence increase in BP. adrenergic receptors D. Inhibiting alpha-2 adrenergic receptors 326. Minoxidil causes vasodilation through: A. 380 A. Choice B - calcium channel blockers; Choice C - Nitroprusside, Hyperpolarization of smooth muscle membrane nitrates; Choice D - fenoldepam through opening of potassium channels B. Blockade of voltage gated calcium channels causing a reduction in calcium influx C. Release of nitric oxide from the drug itself D. Activation of dopamine receptors E. Blockade of beta receptors 327. Monday disease is caused by occupational 465 D. exposure to: A. lead B. dust C. arsenic D. nitrates E. Gun powder 328. The most common anti-arrythmic drug used for 125 A. Lidocaine is the drug of choice for ventricular arrythmias post-MI, cardiac arrest is: A. Lidocaine B. Adenosine C. and digoxin-induced arrythmias. It is the least cardiotoxic among Flecainide D. Sotalol E. Amiodarone conventional anti-arrythmic. SIMILAR TO PREVIOUS BOARD EXAM CONCEPT.. 329. Most common side effect of clomiphene citrate A. 153 A. Hot flushes are the most common side effect of use of clomiphene Hot flushes B. Constipation C. Headache D. Allergic citrate, the rest are occasional side effects skin reactions E. None of the above 330. The most dangerous hallucinogenic agent. A. LSD 461 B. B. phencyclidine C. MDMA D. cocaine E. Marijuana 331. The most important pharmacologic action of 443 E The most important pharmacologic action of digitalis in the treatment digitalis in the treatment of heart failure is its ability of heart failure is its ability to increase myocardial contractile force by to: A. Increase heart rate B. Reduce venous inhibiting Na/K ATPase and increaseing intracellular calcium. pressure C. Reduce arterial pressure D. Produce diuresis in edematous patients E. Increase myocardial contractile force 332. The most powerful hallucinogen known to man is: A. LSD B. marijuana C. 486 A. SIMILAR TO PREVIOUS BOARD EXAM methamphetamine D. phencyclidine CONCEPT/PRINCIPLE 333. Mr. A. Lipin, a farmer from a distant hacienda was brought to the ED and 691 B. Answer: B. Atropine (pp. 141, 149 and 162, admitted due to unconsciousness, profuse salivation and shallow Katzung Pharmacology, 9th edition) Notes: Atropine breathing. It took 7 hours of travel time from the hacienda to the nearest is the prototype antimuscarinic drug to be given Provincial Hospital. BP 140/90 mmHg, PR 52 bpm, temp 30OC and with initially in this patient suffering from constricted pupils. As an ED physician, what medication will you initially organophosphate poisoning. It is best to give both give that will most likely benefit the patient? A. Pralidoxime B. Atropine C. Atropine and Pralidoxime if the golden period of 6 Pralidoxime + Atropine D. Physostigmine hours was not violated. The likelihood of aging of the phosphate enzyme complex occurs after 6 hours; however, recent reports suggest that administration of multiple doses of pralidoxime over several days may be useful in severe poisoning. 334. MRSA, a multi drug resistant organism is kept at bay through 78 C. Structural change in target PBP is mechanism administration of vancomycin. Currently, reports of MRSA resistant to of resistance of MRSA not VRSA. Resistance to vancomycin (VRSA) are being reported. What is the mechanism of vancomycin in vancomycin-resistant S aureus strains resistance to vancomycin? A. Structural change in target PBP B. is due to modification of the D-Ala-D-Ala binding Formation of extended spectrum beta lactamases C. D ala-ala is modified site of the peptidoglycan building block in which the to D ala-lactate D. Changes in porin structure of outer cell wall E. terminal D-Ala is replaced by D-lactate. This results Extrusion of the drug through P-glycoprotein efflux pump in the loss of a critical hydrogen bond that facilitates high- affinity binding of vancomycin to its target and loss of activity. 335. Mrs. T. Anga, 25 yo, G1P1 was misdiagnosed to have preeclampsia 687 A. Answer: A. Methyldopa (pp. 235 [.pdf file], because of the inaccuracy of her history and was given an anti- Katzung Pharmacology, 9th edition). Notes: The hypertensive to control her blood pressure. Fortunately, patient had no symptoms experienced by the patient are consistent postpartum complications and BP was controlled. She continued taking with Autoimmune Hemolytic Anemia which is a rare the previously given antihypertensive but was lost to follow-up. After adverse effect of methyldopa. Methyldopa is a drug maintaining the drug for more than 12 months, patient developed which lowers blood pressure chiefly by reducing the intermittent episodes of headache, myalgias, jaundice and passage of peripheral vascular resistance and variably reduces dark brown urine. She finally went back to her physician for consult and heart rate and cardiac output. Ten to twenty percent laboratory testing revealed a positive Antiglobulin Test. If the condition of patients undergoing therapy for longer than 12 was drug-induced, what is the most likely offending agent? A. months from methyldopa will have positive Methyldopa B. Hydralazine C. Clonidine D. Magnesium Sulfate Coomb’s Test (or Antiglobulin Test). 336. Mr. X was maintaining an unrecalled drug for the past few months. He is 463 C. now complaining of visual changes or "yellow halo vision". What is the drug that is known to cause such effect? A. amiodarone B. ethambutol C. digoxin D. furosemide E. Procainamide 337. Muscarinic antagonists used for the treatment of parkinsonism, except? A. 666 B. Triamterene - potassium sparing diuretic Trinexyphenidyl B. Triamterene C. Benztropine D. Biperiden E. None of the above 338. A neonate was given IV antibiotics for 3 days. However, the baby 447 C. Chloramphenicol is the drug that was given developed hypothermia associated with diarrhea and grayish color of the to neonate causing gray baby syndrome. It inhibits skin. What is the mechanism of action of the drug that was given to the transpeptidation at 50s subunit and it is baby? A. Inhibits transpeptidation in bacterial cell walls B. Inhibits protein bacteriostatic. Penicillin inhibits transpeptidation in synthesis by binding to 30s subunit C. Inhibits transpeptidation at 50s bacterial cell walls. Aminoglycoside inhibits protein subunit D. Inhibits dihydropteroate synthase E. Inhibits DNA replication by synthesis by binding to 30s subunit. Co-trimoxazole binding to DNA gyrase inhibits dihydropteroate synthase. Fluoroquinolone inhibits DNA replication by binding to DNA gyrase. 339. A newly formulated drug with a half life of 6 hours is eliminated via 1st 404 D. 100% elimination will be reached in 4 half order kinetics, how many hours will the drug be 100% eliminated from the lives. (6 x 4=24) body of a normal person? A. 6 hours B. 12 hours C. 18 hours D. 24 hours E. 36 hours 340. Nitrates relieve angina by: A. Decreasing venous return to the heart B. 348 A. Increasing intracardiac volume C. Increasing end diastolic left ventricular volume D. All of the above 341. Notable to these drugs are its capability of inducing CP450 enzyme system 568 B. All except Cimetidine induce CYP450 in the liver except: a. Phenytoin b. Cimetidine c. Carbamazepine d. enzyme complex Griseofulvin 342. NOT True of Phase 2 enzymes in Xenobiotic metabolism: a. they facilitate 566 D. Because of conjugation of these the elimination of drugs b. inactivation of electrophilic and potentially toxic substances, they have a higher molecular weight metabolites produced by oxidation. c. phase 2 reactions produce a metabolite with improved water solubility d. products have decreased molecular weight 343. On a patient with signs of shock, At a dose of 5 mcg/kg/min, dopamine will 66 C. Dopamine has different dose dependent predominanty have what effect? A. Increased peripheral vascular actions. At 1-2 mcg/kg/min, it preferentially resistance B. Activation of the RAAS C. Inotropic effect D. Renal activates D1 receptor. At 2-10 mcg, it activates B1 vasodilation E. Arrythmias receptor and at >10 mcg, it activates A1 receptors 344. Ondansetron, highly effective in preventing chemotherapy-induced nausea 338 C. and vomiting, is an antagonist at which receptor? A. muscarinic B. B2 C. 5- HT3 D. D2 345. Ondansetron is an antagonist of which serotonin receptor? A. 5HT1D B. 391 D. 5HT3 receptor is also the only ligand-gated 5HT1B C. 5HT2 D. 5HT3 E. 5HT4 ion channel among the serotonin receptors. 346. One of the choices does not belong to the third generation cephalosporins. 477 D. All of the choices except cefoxitin are all A. cefotaxime B. ceftazidime C. ceftriaxone D. cefoxitin E. Ceftizoxime third generation cephalosporin. Cefoxitin is a second generation. 347. One of the following drugs causes an increase in metabolism of other drugs 645 B. Rifampicin is an inducer. All other choices when administered simultaneously. A. Valproic Acid B. Rifampicin C. are inhibitors. Venlafaxine D. Ritonavir E. Ketoconazole 348. One of the following drugs is effective in reducing the risk of ulcers in 279 D. chronic NSAID users. Diarrhea is a frequent side effect. Which is it? A. Sucralfate B. Ranitidine C. Omeprazole D. Misoprostol E. Cimetidine 349. The only beta-lactam antibiotcs that does not cross react with penicillins A. 57 B. The monobactams have activity against Ceftriaxone B. Aztreonam C. Meropenem D. Aminoglycosides E. aerobic Gram negative bacilli and dose not cross Vancomycin react with penicillin. 350. On your clinic, an 18 month old infant was brought for consult due to "blank 656 C. This is absence seizure and DOC is stares" with a duration of 5-10 seconds. The best medication to be given for Ethosuximide this case is A. Phenobarbital B. Valproic acid C. Ethosuximide D. Levepiracetam E. Carbamazepine 351. oral administration of this antibiotic will yield greater blood levels 316 A. the pharmacokinetics of chlorampenicol is compared when it is administered intravenously? A. chlorampenicol B. unusual since blood levels are higher when the Quinipristin -dalfopristin C. Azithromycin D. Telithromycin E. None of the drug is given at oral form rather than intravenous above form. Katzung 11th ed page 802 352. Oral chelation treatment in hemochromatosis can be done by giving: A. 430 B. Therapeutic phlebotomy is the main line of Deferoxamine B. Deferasirox C. EDTA D. Penicillamine treatment of hemochromatosis, especially the primary type. Should chelation becomes warranted, it can be done via IV (deferoxamine) or oral (Deferasirox). 353. Organophosphate toxicity is based on its ability to: A. Compete with brain 357 B. pyridoxal phosphate B. Inhibit acetylcholinesterase C. Interrupt heme synthesis D. React with sulfhydryl groups binding to protein and other enzymes 354. Organs involved in xenobiotic metabolism are as follows, EXCEPT: 606 E Xenobiotic metabolizing enzymes are found in most A. Liver B. Small intestines C. Lung D. Nasal mucosa E. None of the tissues in the body with the highest levels located in the above tissues of the gastrointestinal tract (liver, small and large intestines). Other organs that contain significant xenobiotic-metabolizing enzymes include the tissues of the nasal mucosa and lung, which play important roles in the first-pass metabolism of drugs that are administered through aerosol sprays. 355. An otherwise healthy 19-year-old woman comes to the physician 531 A. Oral isotretinoin is never the first option for acne, because of a 3-year history of intermittent facial blemishes. She only used when topical treatments have failed drinks wine occasionally on weekends. She takes no medications. Examination shows multiple 1- to 2-mm red and white papules and on the forehead and cheeks. Which of the following is the most appropriate initial pharmacotherapy? A) Topical benzoyl peroxide B) Systemic corticosteroids C) Oral isotretinoin D) Topical corticosteroids 356. A patient came in at the ER having severe headache and heat 171 C. Take note, the patient has asthma… Katzung 11th intolerance. Examination revealed enlarged thyroid, high blood edition chapter 38 page 677. “If propanolol is pressure and severe tachycardia. On history, his relative told you contraindicated by the presence of severe heart failure or that he also has frequent attacks of asthma. In your findings, you asthma, hypertension and tachycardia may be controlled are suspecting that he is having thyroid storm. Which of the with diltiazem, 90-120mg orally three or four times daily or following drugs can be given immediately to relieve hypertension 5-10 mg/h by IV infusion.†SIMILAR TO PREVIOUS and tachycardia? A. propanolol B. esmolol C. diltiazem D. nifedipine BOARD EXAM CONCEPT/PRINCIPLE. Katzung 11th edition E. clonidine page 677 357. A patient came in for consult due to infertility. It was found out that 161 A. SIMILAR TO PREVIOUS BOARD EXAM the patient is having ovulation disorder so you decided to give here CONCEPT/PRINCIPLE. Katzung 11th edition page 719.. "the clomiphene. It is important to tell to the patient that the most most common adverse effects in patients treated with this common adverse effect of clomiphene is: A. hot flushes B. bleeding drug are hot flushes, which resemle those experienced by C. constipation D. headache E. nausea and vomiting menopausal patients." 358. Patient came in the clinic complaining of amenorrhea and 194 C. Risperidone causes hyperprolactinemia. galactorrhea. History revealed intake of an anti-psychotic drug. What could have caused the symptoms of the patient? A. Clozapine B. Olanzapine C. Risperidone D. Haloperidol E. Chlorpromazine 359. A patient came to the ER with diarrhea, tremors and excessive 65 D. The patinet is exhibting symptoms of sweating. HR is 50, BP is 120/70, RR is 22. Pupils are 1-2 mm. What organophosphate poisoning with increased secretion of all drug can be given to the patient? A. Naloxone B. Neostigmine C. body fluids plus bradycardia, miosis and skeletal muscle Diazepam D. Atropine E. EDTA exictation. Antidote is Atropine and Pralidoxime 360. A patient in PACU is in no apparent distress. The V/S are stable 692 D. Answer: D Patient has atrial fibrillation w/ rapid except for HR of 128 bpm that is irregular with no p wave. Which of ventricular response. B-blocker, Ca Channel blocker or the following treatment option would not be appropriate as initial Digoxin may be used. Supraventricular tachycardia, not therapy? A. Metoprolol B. Diltiazem C. Digoxin D. Adenosine a.fib responds to adenosine. 361. The patient is being maintained on oral anticoagulant. Which of the 122 A. Phenobarbital, a barbiturate, is a Cytochrome P450 following if taken by the patient will decrease the effect of the drug inducer which will increase clearance of other drugs, thus, he is presently taking? A. Phenobarbital B. Sulfamethoxazole C. decreasing their effects. SIMILAR TO PREVIOUS BOARD Valproic acid D. Ketoconazole E. Vancomycin EXAM CONCEPT. 362. Patient is diagnosed with Lennox-Gaustat Syndrome and is taking 196 A. Topiramate is used as an adjunct in Lennox-Gaustat maintenance drug. He later on developed urolithiasis. The patient is Syndrome. Its adverse effects are unusual/weird for anti- probably taking what drug? A. topiramate B. lamotrigine C. seizure drugs (ex. Urolithiasis, myopia, glaucoma) ethosuximide D. carbamazepine E. levetiracetam 363. A patient is taking fenofibrate for elevated trigylceride levels, 253 C. SIMILAR TO PREVIOUS BOARD EXAM which of the following laboratory examinations should the doctor CONCEPT/PRINCIPLE, fibrates can potentially increase request to monitor for potential side effects upon follow up after 3 liver enzymes to 6 months? A. CBC with differential count B. urinalysis C. AST, ALT D. fasting blood glucose levels E. serum total cholesterol and triglycerides 364. A patient presented to your clinic with complaints of 276 B. Methotrexate is known to cause pulmonary fibrosis dyspnea and chronic cough. He states that he frequently gets short of breath and is unable to take a deep breath. History is significant for prolonged treatment of rheumatoid arthritis. Chest X-ray revealed fine reticulonodular densities. Which drug is most likely responsible? A. Glucocorticoids B. Methotrexate C. Infliximab D. Cyclosporine E. Anakinra 365. A patient presents with petit mal seizures. The drug of choice 496 C. Petit mal seizures, also known as absence seizures, for this type of seizure is: A. phenytoin B. topiramate C. involves a brief, sudden lapse of consciousness. The DOC is ethosuximide D. lamotrigine ethosuximide. 366. Patients receiving cyclophosphamide should also be given: 207 A. Leucovorin for methotrexate. Pyridoxine for isoniazid. A. Mesna B. Leucovorin C. Vitamin B6 D. Thiamine E. Thiamine for alcoholics. Amifostine 367. A patient was diagnosed with familial combined 115 C. Page 29 of Topnotch Handout. A synergistic combination hypercholesterolemia, which of the following synergistic for patients with familial combined hypercholesterolemia drug combination will be most suitable for this patient: A. (overproduction of VLDL) are niacin+resin and statin+fibrate. Niacin + Statin B. Statin + Ezetimibe C. Niacin + Resin D. Statin Fibrate nd resin combinations are disadvantageous because they + Resin E. Fibrate + Resin confer an increased risk of cholelithiasis. while Statin and resin combinations cause impaired statin absorption. Niacin and statin combinations are more for familial hypercholesterolemiawhere the defect is in the LDL receptors hence increased LDL. 368. A patient was given Nedocromil for asthma. Which of the 164 D. SIMILAR TO PREVIOUS BOARD EXAM following is the mechanism of action of the drug? A. Inhibits CONCEPT/PRINCIPLE. Katzung 11th edition page 349 the release of histamine B. Direct relaxation of smooth muscle C. Inhibits leukotriene pathway D. mast cell stabilizer E. none of the above 369. A patient was put on standrard dose of warfarin therapy, 63 A., D. INH is also an inhibitor. Rifampicin is an inducer.. The aftter 1 week he suddenly experienced hematochezia. Which patient probably is taking a CYP450 inbibitor such as valproic of the ff drugs he is also currently taking could have caused acid that decreased the metabolism of warafarin leading to its this? A. Valproic acid B. Ethanol C. Carbamazepine D. toxic side effectds. All the rest of the drugs are inducers Isoniazid E. Griseofulvin 370. A patient will be undergoing neurosurgery, the 407 A. Ketamine is capable of Dissociative Anesthesia ( neurosurgeon would like to maintain the patient's Analgesia, Amnesia and Catatonia with retained consciousness) consciousness while the procedure is on going. What would be the preferred anesthetic of choice? A. Ketamine B. Etomidate C. Midazolam D. Fentanyl E. Propofol 371. A patient with CHF was rushed to the ED where you are on- 411 D. Spironolactone and Acetazolamide can aggravate the duty, ABG was done revealing Metabolic Acidosis. His wife ongoing acidosis of the patient, the only ones left are loop told you that he had a history of severe allergic reaction to diuretics, but then Furosemide and Torsemide are both Sulfa Sulfa drugs, but you need to give the patient a powerful containing hence cannot be given to the patient. Ethacrynic acid diuretic, Unfortunately the following drugs are the only ones is most suitable to this patient. available in your Pharmacy, Among which of the following will you give? A. Furosemide B. Spironolactone C. Torsemide D. Ethacrynic Acid E. Acetazolamide 372. A patient with leprosy is taking a drug that casts a red color 203 C. Rifampin causes red discoloration of body fluids but not to the skin. He is probably taking: A. Dapsone B. Rifampin C. the skin. Clofazimine D. Ethambutol E. Phenylbutazone 373. A patient with neuropathic pain was prescribed gabapentin. 549 B. Although gabapentin is a structural analog of GABA, it What is the primary mechanism of action of gabapentin? A. does NOT activate GABA receptors directly. Direct activation of GABA reeptors B. Inhibition of T type Ca2+ channels C. Blockade of voltage-gated Na channels D. Inhibition of GABA transporter (GAT-1) prolonging the action of the neurotransmitter E. Enhancement of K+ channel permeability causing neuronal hyperpolarization 374. A patient with warfarin as a maintenance medication is treated 273 D. Erythromycin inhibits metabolism of warfarin causing an for pneumonia. Prothrombin time monitoring during treatment increase In INR of the respiratory infection reveals elevated INR but it decrease back to baseline levels after treatment with the antibiotic is completed. Which of the following is most likely given? A. Amoxicillin B. Ceftriaxone C. Clindamycin D. Erythromycin E. TMP-SMX 375. Penicillin is the drug of choice for syphilis. It acts through: A. 480 A. Penicillin inhibits transpeptidase enzyme needed for the Inhibition of transpeptidase B. Inhibition of topoisomerase II C. bacterial cell wall synthesis. Vancomycin inhibits D-ala D ala Inhibition of glucan synthase D. Inhibition of D-ala-D-ala synthase. Caspofungin inhibits glucan synthase. terminus E. None of the choices Flouroquinolone inhibits topoisomerase II and IV. 376. A physician prescribes a diabetic patient with an antidiabetic 275 C. Thiozolidinediones - activates PPAR, a nuclear receptor agent that activates a nuclear receptor that increases to improve sensitivity to insulin transcription of GLUT-4 transporters in adipose tissue. The drug prescribed is most likely: A. Methotrexate B. Insulin C. Pioglitazone D. Glyburide E. Sitagliptin 377. Plasma magnesium levels should be maintained at this 488 B. therapeutic range to prevent eclamptic convulsions: A. 2-4 meq/L B. 4-7 meq/L C. 8-10 meq/L D. 10-12 meq/L 378. A post stroke patient is taking in warfarin. Which among the 172 D. Isoniazid in a P450 inhibitor which will delay the following drugs will most likely increase the possibility of excretion of warfarin when given. Other choices are P450 having warfarin toxicity when added in his management? A. inducers Smoking B. Barbiturates C. Carbamazepine D. Isoniazid E. Ethanol 379. A potent antiandrogen that is usually coadministered with 511 E Katzung 10th ed., 679. this is SIMILAR TO PREVIOUS leuprolide that has been used in the treatment of prostatic BOARD EXAM CONCEPT/PRINCIPLE carcinoma that frequently causes gynecomastia and occassionally mild reversible hepatic toxicity: A. Goserelin B. Spironolactone C. Cyproterone D. Ketoconazole E. Flutamide 380. Pralidoxime is an antidote to organophosphate poisoning. 423 E Which of the following is not true about this drug? A. It is a cholinesterase regenerator. B. It requires to be given 6-8 hours before organophosphate-cholinesterase binding occurs. C. It binds with the phosphorus of organophosphates. D. All of the above E. None of the above 381. The preferred antiseizure drug for pregnant women: A. 134 B. Phenobarbital is the preferred antiseizure drug in Valproic acid B. Phenobarbital C. Phenytoin D. Carbamazepine children and pregnant women. SIMILAR TO PREVIOUS BOARD E. Topiramate EXAM CONCEPT. 382. The preferred anti-thyroid drug for thyroid storm: A. 354 C. Methimazole B. Carbimazole C. Propylthiouracil D. Propranolol 383. A pregnant patient complains of multiple external genital 483 C. The most common treatments for genital warts in warts. Which of the following agents can be used in this pregnancy are cryotherapy, laser removal, and trichloroacetic patient? A. Podofilox solution B. Imiquimod cream C. acid. The rest of the choices are under Pregnancy Category C. Trichloroacetic acid D. Sinecatechins ointment 384. The primary indication of Epoprostenol? A. Open angle 228 B. SIMILAR TO PREVIOUS BOARD EXAM glaucoma - increase outflow of aqueous humor B. Severe CONCEPT/PRINCIPLE last aug 2014. Epoprostenol is a pulmonary hypertension C. Postpartum bleeding D. All of the Prostacyclin analog use for dialysis to prevent platelet above E. None of the above aggregation and also for pulmonary hypertension. Latanoprost or PGF2a analog is the one use for glaucoma. Misoprostol or RU 486 (PGE1 analog) has an off label indication for induction of labor. it primarily acts as an abortifacient. 385. The prime target of addictive drugs in the brain: A. Thalamocortical 344 C. Reference: Katzung. Basic and Clinical system B. Nigrostriatal dopamine system C. Mesolimbic dopamine Pharmacology, 11th ed. p. 554 system D. Mesocortical dopamine system 386. The prokinetic MOA of metoclopramide A. Inhibition of dopamine 50 A. Metoclopramide inhibits the D2 receptors in the receptors in the gut B. Inhibition of motilin receptors in the gut C. gut; erythromycin stimulates the molitin receptors in Inhibition of serotonin receptors in the gut D. AOTA E. Both A and B the gut. 387. Prolonged treatment with a PPAR alpha ligand in the treatment of 234 D. PPAR alpha ligands is the MOA of fibric acid hyperlipidemia will increase the risk for the following condition? A. derivatives ( Gemfibrozil and fenofibrate). It increases Cutaneous flushing B. Hyperuricemia leading to Gout C. Intestinal the supersaturation of bile leading cholelithiasis. Risk malabsorption D. Gallstones E. hepatotoxicity for hepatotoxicity only significantly increase if gemfibrozil is combined with HMG CoA inhibitor. Cutaneous flushing and hyperuricemia are toxicity profile of Niacin. Intestinal malabsorption is a primary side effect of resins ( cholestyramine, colestipol, colsevelam) 388. Prolonged treatment with a PPAR alpha ligand in the treatment of 315 D. PPAR alpha ligands is the MOA of fibric acid hyperlipidemia will increase the risk for the following condition? A. derivatives ( Gemfibrozil and fenofibrate). It increases Cutaneous flushing B. Hyperuricemia leading to Gout C. Intestinal the supersaturation of bile leading cholelithiasis. Risk malabsorption D. Gallstones E. hepatotoxicity for hepatotoxicity only significantly increase if gemfibrozil is combined with HMG CoA inhibitor. Cutaneous flushing and hyperuricemia are toxicity profile of Niacin. Intestinal malabsorption is a primary side effect of resins ( cholestyramine, colestipol, colsevelam) 389. Propylthiouracil and Methimazole are both used in the treatment of 6 B. The halflife of PTU is 1.5hrs and methimazole is 6- hyperthyroidism. They are thioamides which inhibits the enzyme 8hrs. PTU is generally given every 6-8hrs while thyroid peroxidase necessary for the synthesis of thyroid hormone. methimazole is given once a day. Thus methimazole is Which of the following is false about propylthiouracil? A. PTU is more more convenient for patients. Methimazole is also 10 highly protein bound than methimazole B. PTU has a longer halflife and times more potent than PTU. Source: Katzung Basic and requires less frequent dosing than methimazole C. PTU is less potent Clinical Pharmacology 11th ed p671 than methimazole. D. PTU has the added activity of preventing peripheral conversion of T4 to T3 E. None of the Above 390. A psychotic patient on haloperidol presents with rigidity, hyperpyrexia 497 B. Tardive dyskinesia is also one of the toxicities of and autonomic instability. This condition is known as: A. Serotonin antipsychotics aside from NMS. It is a condition where syndrome B. Neuroleptic malignant syndrome C. Tardive dyskinesia D. the patient presents with stereotypic oral- facial Malignant hyperthermia movements. 391. the pungency of this inhaled anesthetics leading breath-holding limits 313 D. (SIMILAR TO PREVIOUS BOARD EXAM it use in anesthesia induction? A. desflurane B. Isoflurane C. CONCEPT/PRINCIPLE) Katzung review 8th ed p 211 Sevoflurane D. enflurane E. Halothane 392. Pupillary construction is a characteristic effet of all opioids, except: A. 323 B. Oxycodone B. Meperidine C. Morphine D. Fentanyl 393. A pure opioid antagonist with a greater affinity for μ receptors and 607 B. Naloxone is a pure opioid anatagonist and is used for acute opioid overdose: A. Morphine B. Naloxone C. Codeine D. used to counter the effects of opiate overdose, for Dextrometorphan E. Diphenoxylate example heroin or morphine overdose. The other drugs are opioid agonists. 394. A recently-diagnosed hypertensive was started on a BP lowering 544 D. agent. Due to financial constraints, the patient abruptly stopped taking the prescribed medications after stocks were consumed. A day later, the patient was brought to the clinic for light-headedness; on admission, patient's BP was 180/100. Which of the following anti- hypertensives, known for rebound hypertension, was most likely taken by the patient? A. Enalapril B. Furosemide C. Methyldopa D. Clonidine E. Metoprolol 395. The rectal route is said to have partial avoidance of the first pass 103 A. Page 3 of Topnotch Handout. The superior rectal effect. Thru which of the following venous drainage of the rectum vein will drain to the Inferior mesenteric vein then to the will not bypass the first pass effect : A. Superior rectal vein B. Middle portal vein, hence having a first pass effect. The middle rectal vein C. Inferior rectal vein D. Both B and C E. All of the above rectal vein will drain thru the internal iliac vein then to the inferior vena cava, while the inferior rectal vein will drain via the internal pudendal vein then thru the internal iliac vein then thru the inferior vena cava. 396. Remarkable to these drugs is their ability to inhibit cytochrome P450 434 D. causing significant interactions with other drugs, except: A. Isoniazid B. Cimetidine C. Erythromycin D. Barbiturates 397. Reserpine blocks this step of adrenergic transmission: A. Synthesis 444 B. Reserpine blocks vesicular transport in adrenergic of the neurotransmitter B. Transport of neurotransmitter to vesicles transmission. C. Release of the neurotransmitter D. Reuptake into the presynaptic neuron E. Diffusion in the synaptic cleft 398. RH, a 50 year old bank executive was brought to the ER due to loss 148 A. The patient has been poisoned with arsenic and the of consciousness, upon PE you noted his breath smelling like sweet drug of choice is dimercaprol with garlicky odor, what is the appropriate drug for this case A. Dimercaprol B. EDTA C. Hydration D. Deferoxamine E. Flumazenil 399. Rivastigmine is used for: A. Motion sickness B. Myasthenia gravis C. 469 E Rivastigmine is a cholinomimetic drug used for Schizophrenia D. Parkinson's disease E. Alzheimer's disease Alzheimer's disease. 400. RJLC, a 30 year old female was treated for extraintestinal amebiasis, 149 C. Metronidazole eliminates the trophozoite forms of what is the drug of choice to eliminate the extraintestinal parasites? the parasite in the intestine or in the tissues. A. Paromomycin B. Diloxanide furoate C. Metronidazole D. Pyrantel pamoate E. Praziquantel 401. Route of administration with instantaneous absorption, bypasses 662 A. Compared with other routes of administration, the first pass effect with I00% bioavailabilty? A. IV B. SL C. Oral D. Rectal intravenous route is the fastest way to deliver fluids and E. None of the above medications throughout the body. 402. Salmeterol has a slower onset of action as compared with 601 A. The quickier onset of action of formoterol can be formoterol in terms of producing bronchodilation. This is because of: explained by diffusion microkinetic model. This model A. Increased lipophilicity of salmeterol B. Faster degradation of suggests that formoterol and salmeterol are both retained salmeterol C. More efficient absorption of formoterol D. Higher in the lipid bilayer adjacent to the beta2-adrenergic affinity of formoterol to beta adrenergic receptors E. A and D receptor. Because of salmeterol's lipophilicity, it associates more with the lipid bilayer than with the receptor, accounting for its slower onset of action. 403. a second generation Cephalosporin with intrinsic activity against 562 A. Cefoxitin has anaerobe coverage apart from its anaerobes: a. cefoxitin b. cefipime c. cefaclor d. ceftriaxone 0 activity against gram positive and some gram neg infections 404. Serum concentration of lithium in mEq/L favored for acutely manic 340 A. The serum concentration favored for long-term patients is: A. 0.9-1.1 B. 0.6-0.75 C. 0.5-0.85 D. 0.1-0.5 use in the prevention of reccurent manic-depressive illness, on the other hand, is 0.6-0.75 mEq/L. 405. Single dose drug used in eradication of meningococcal carrer state 46 B. Drugs used to eradicate meningococcal carrier A. RIfampicin B. Ciprofloxacin C. TMP/SMX D. Chloramphenicol E. state: 1. rifampin 2. ciprofloxacin (single dose) 3. NOTA ceftriaxone (single dose) 406. Single dose oral antibacterial agent used to eradicate 47 D. Ceftriaxone - single dose IM s uncomplicated gonorrhoea A. Ceftriaxone B. Cefixime C. Azithromycin D. Both B and C E. AOTA 407. Sisa, 28/F, recently-diagnosed schizophrenic was admited to the 555 D. Neuroleptic malignant syndrome is characterized Psychiatry ward and started on Risperidone. Three days later, the by FEVER, RIGIDITY, and AUTONOMIC INSTABILITY. patient developed fever (39.9C) and rigidity. Patient was notably NMS is linked to intake with several atypical confused. Vital signs were as follows: BP 170/120, HR 118, RR 22. antipsychotics. What could explain Sisa's symptoms? A. Extrapyramidal symptoms B. Tardive dyskinesia C. Serotonin syndrome D. Neuroleptic malignant syndrome E. Malignant hyperthermia 408. SMA, a 68 year old female with hypertension is taking 156 D. Peptidyl dipeptidase is otherwise known as ACE which is the captopril, you know that it is effective in hypertension enzyme inhibited by ACE inhibitors like captopril. because it inhibits: A. Cathepsin C B. Dipeptidyl dipeptidase-2 C. Dipeptidyl dipeptidase-4 D. Peptidyl dipeptidase E. Procarboxypeptidase 409. Somatostatin interacts with a A. Gq- 23 B. Somatostatin (growth-inhibiting factor) binds to Gi- coupled protein proteinâ€"coupled receptor B. Gi-proteinâ€"coupled receptor, initiating exchange of GTP for GDP, which inhibits AC and receptor C. Ligand-activated ion channel D. Receptor- leads to reduced cAMP production. activated tyrosine kinase E. Intracellular nuclear receptor 410. Some drugs like magnesium sulfate needs a loading 582 A. Loadign dose = (Vd) (desired plasma dose in order to reach steady state at a reduced concentration)/(bioavailabitlity) the minimum effective dose is usually the amount of time. The following are used to compute desired plasma concentration. Clearance is used to calculate the for loading dose EXCEPT? A. Clearance B. Volume of maintenance dose Distribution C. Minimum effective dose D. Bioavailabiloity E. No exception 411. Sources of drugs can either be natural or synthetic. 686 D. Answer: D. AmphoteriCin B (pp. 1104 [.pdf file], Katzung These might be derived from plant sources, animal or Pharmacology, 9th edition.) Notes: incomplete list Streptomycin à Strept. marine products, minerals, microorganisms and others griseus Neomycin à Strept. fradiae Paromomycin à Strept. rimosus from laboratory sources. Many antibiotics were Kanamycin (der. Amikacin) à Strept.kanamyceticus Tobramycin à Strept. derived from Genus Streptomyces, Penicillium, tenebrarius Ivermectin à Strept. avermitilis Mitomycin à Strept. Micromonospora, etc. Which of the following drugs is caespitosus Erythromycin à Strept. erythreus Lincomycin (der. a polyene antibiotic derived from Streptomyces Clindamycin) à Strept. lincolnensis Rifamycin (der. Rifampicin) à Strept. nodosus? A. Netilmicin B. Amikacin C. Streptomycin D. mediterranei Gentamicin à Micromonospora purpurea Sisomicin (der. Amphotericin B Netilmicin) à Micromonospora inyoensis 412. Strongest inhibitor of CYP3A4 amongst the protease 44 A. Ritonavir - Strongest inhibitor A. Ritonavir B. Saquinavir C. Lopinavir D. Amprenavir E. Atazanavir 413. Succor Mendiola was diagnosed to have glaucoma. 631 E Pilocarpine induces ciliary muscle contraction thereby opening the Which of the following drugs induces ciliary muscle trabecular meshwork and increasing the outflow contraction thereby opening the trabecular meshwork and increasing the outflow? A. Timolol B. Mannitol C. Epinephrine D. Latanoprost E. Pilocarpine 414. Sulfone in the management of leprosy causes? A. 136 A. Drugs used in leprosy: Dapsone, Rifampicin, and Clofazimine. Inhibition of folic acid synthesis B. Inhibition of PABA Dapsone, a sulfone, is the most active drug used against M. leprae. It is C. Inhibition of DNA synthesis D. Inhibition of protein bacteriostatic and inhibits folic acid synthesis. Clofazimine acts by synthesis E. Inhibition of arabinosyl transferase binding to guanine bases in bacterial DNA. 415. Sympathomimetic drug that causes mydriasis without 613 C. Atropine and tropicamide cause mydriasis and cycloplegia. cycloplegia: A. Atropine B. Tropicamide C. Timolol has no mydriatic effect. Phenyephrine D. Pilocarpine E. Timolol 416. Teratogenesis is one of the dreaded complications of 422 A. improper use of drugs during pregnancy. Of note, lithium, a drug primarily used to treat bipolar disorders, can cause: A. Ebstein's anomaly B. Vaginal clear cell adenocarcinoma C. Phocomelia D. Mobius sequence 417. Theophylline has a pKa of 8.7, what is the 581 D. Above pKa, the unprotonated and uncharged form of a weak predominant form at pH 11.3? A. Ionized and base predominates. protonated B. Non-ionized and protonated C. Charged and unprotonated D. Uncharged and unprotonated E. None of the above 418. This 3rd generation cephalosporin can cross the 192 A. All 3rd generation cephalosporins can cross the blood brain blood brain barrier A. ceftizoxime B. cefpodoxime C. barrier except: ceftibuten, cefpodoxime, cefixime, cefoperazone. ceftibuten D. cefaclor E. cefuroxime 419. This antidiabetic drug because of its unique mechanism of 79 B. Acarbose and miglitol are competitive inhibitors of the action should be taken just before ingesting the first portion intestinal α-glucosidases and reduce postmeal glucose of each meal? A. Exenatide B. Miglitol C. Pramlintide D. excursions by delaying the digestion and absorption of starch Pioglitazone E. Metformin and disaccharide. Both acarbose and miglitol are taken in doses of 25â€"100 mg just before ingesting the first portion of each meal 420. This antihistamine causes depolarization-induced paralysis in 330 C. Mebendazole selectively inhibits microtubule synthesis nematodes: A. Mebendazole B. Ivermectin C. Pyrantel and glucose uptake. Ivermectin interferes with GABA mediated pamoate D. Piperazine neurotransmission. Piperazine is an agonist at GABA receptors. 421. This antihypertensive drug is absolutely contraindicated in 350 D. pregnancy because it can cause renal damage in the fetus: A. Reserpine B. Hydralazine C. Propranolol D. Captopril 422. This antiplatelet inhibits aggregation by interfering with 324 D. Aspirin is a nonselective irreversible COX inhibitor. GpIIb/IIIa binding to fibrinogen and other ligands: A. Aspirin B. Clopidogrel irreversibly inhibits platelet ADP receptor. Clopidogrel C. Dipyridamole D. Tirofiban Dipyridamole inhibits adenosine uptake and inhibits phosphodiesterase enzymes that degrade cyclic nucleotides. 423. This antiviral agent against Influenza A acts by blocking the 516 A. katzung 10th ed., 815. M2 proton ion channel of the virus particle and by inhibitting uncoating of the viral RNA within infected host cells, thus preventing its replication. A. Amantadine B. Ribavirin C. Oseltamivir D. Entecavir E. None of the abpve 424. This autonomic drug should be given with caution in patients 490 D. Reserpine which crosses the blood brain barrier is with depressive disorders: A. guanethidine B. metyrosine C. notorious for causing depression and increasing suicidal vesamicol D. Reserpine tendencies. 425. This beta blocker has a unique mechanism of action owing to 69 E Nebivolol is a β1 receptor blocker with nitric oxide- its it has a nitric oxide (NO)-potentiating vasodilatory effect in potentiating vasodilatory effect used in treatment of addition to beta blockade. A. Carvedilol B. Pindolol C. Nadolol hypertension D. Esmolol E. Nebivolol 426. This cell-cycle specific drug is commonly used for testicular 140 D. cancer. It does not cause myelosuppression, however, dose modification is recommended in the setting of renal dysfunction. What is this chemotherapeutic drug? A. Cyclophosphamide B. Cisplatin C. Busulfan D. Bleomycin E. Etoposide 427. This chemotherapeutic agent is known for its association with 336 D. heart toxicity in cumulative doses: A. 5-FU B. 6-MP C. Methotrexate D. Doxorubicin 428. This direct acting cholinomimetic is used in the treatment of 341 B. postoperative ileum and neurogenic bladder: A. Neostigmine B. Bethanechol C. Pilocarpine D. Physostigmine 429. This diuretic can also be used in the treatment of glaucoma: A. 498 A. Technically, mannitol can also be used but acetazolamide B. hydrochlorothiazide C. mannitol D. amiloride acetazolamide is a better answer. SIMILAR TO PREVIOUS BOARD EXAM CONCEPT/PRINCIPLE 430. This drug activates peroxisome proliferator-activated 494 A. Do not confuse with the mechanism of action of fibrates receptors, a group of nuclear receptors, with greatest which is activation of the nuclear transcription factor PPAR-α. specificity to PPAR-γ receptors: A. pioglitazone B. gemfibrozil C. sitagliptin D. Exenatide 431. This drug belongs to a class of antiarrhythmic drugs which 114 A. Page 24 of Topnotch Handout. Under the Singh- Vaughn prolongs AP duration, PR interval, QRS duration and QT Williams classification. The class being described are Class 1A interval: A. Procainamide B. Lidocaine C. Flecainide D. Antiarrhythmics. Classified underwhich is Procainamide Propanolol E. Dofetilide Quinidine and Disopyramide. 432. This drug class used in asthma exerts its 75 B. SIMILAR TO PREVIOUS BOARD EXAM CONCEPT/PRINCIPLE. Corticosteroids do effect by contraction of engorged not relax airway smooth muscle directly but reduce bronchial reactivity and reduce the vessels in the bronchial mucosa and frequency of asthma exacerbations if taken regularly. Their effect on airway reduces bronchial hyperreactivity? A. obstruction may be due in part to their contraction of engorged vessels in the Beta 2 agonists B. Inhaled bronchial mucosa and their potentiation of the effects of β- receptor agonists, but their corticosteroids C. Methylxanthines D. most important action is inhibition of the infiltration of asthmatic airways by Mast cell stabilizers E. Antimuscarinic lymphocytes, eosinophils, and mast cells. agent 433. This drug inhibits intestinal absorption 174 B. of phytosterol and cholesterol which will eventually decrease the level of LDL. A. rosuvastatin B. ezetimibe C. Niacin D. Gemfibrozil E. Simvastatin 434. This drug inhibits platelet aggregation 500 D. Cilostazol is a phosphodiesterase III inhibitor, tirofiban binds to GP IIb/IIIa by irreversibly blocking ADP receptors: receptor, and alteplase is a thrombolytic (not an anti-platelet) which converts A. cilostazol B. tirofiban C. alteplase D. plasminogen to plasmin. Ticagrelor 435. This drug inhibits the transport of 108 E Page 11 of Topnotch Handout. Storage of Ach into vesicles is inhibited by Acetylcholine into vesicles for storage vesamicol. by VAT: A. Hemicholinium B. Botulinum C. Neostigmine D. Betanechol E. Vesamicol 436. This drug is a derivative of rifamycin 517 B. Katzung 10th ed., 777. Rifabutin is a less potent inducer of the cytochrome P450 which is indicated in place of rifampin enzyme (compared to Rifampin) resulting to slower elimination and longer halflife of for treatment of tuberculosis in HIV- protease inhibitors used in treating HIV patients is cases of HIV with concurrent infected patients who are receiving tuberculosis. Rifapentine like Rifampin is a potent inducer of cytochrome p450 concurrent protease inhibitors: A. enzymes and should not be used to treat HIV-infected patients because of an Rifapentine B. Rifabutin C. Ritonavir D. unaccepptably high relapse rate with rifampin- resistant organisms. Rtonavir is a Rifadin E. Rimactane protease inhibitor. Rifadin and Rimactane are brand names of Rifampin. 437. This drug is a monoclonal antibody that 335 A. Rituximab is a monoclonal antibody that binds to surface protein in NHL binds to VEGF and prevents it from inducing lysis, cytotoxicity and apoptosis. Trastuzumab is a monoclonal antibody that interacting with VEGF receptors: A. recognizes protein in breast CA cells overexpressing HER-2neu receptor for EGF. Bevacuzimab B. Rituximab C. Imatinib inhibits tyrosine kinase activity of the product of bcr-abl oncogene expressed Trastuzumab D. Imatinib in CML. 438. This drug reduces the relapse rates for 151 E Primaquine eradicates hypnozoites in the liver responsible for causing relapse malaria A. Quinine B. Arthemeter- lumefantrine C. Mefloquine D. Doxycycline E. Primaquine 439. This drug requires phosphorylation by 329 D. Efavirenz, delaverdine, and nevirapine are non nucleoside reverse transcriptase host cell kinases to be active: A. inhibitors which do not require host cell kinase for activation, unlike tenofovir, a Efavirenz B. Delavirdine C. Nevirapine nucleoside reverse transcriptase inhibitor which does. D. Tenofovir 440. This drug stimulates platelet production 339 A. Filgrastim stimulates the function and production of neutrophils. and decreases the number of platelet transfusions required by patients undergoing bone marrow suppression in treatment for CA: A. Oprevelkin B. Filgrastim C. Erythropoietin D. Folic acid 441. This drug that produces a permanent 55 A. Radioactive iodine is the only medical therapy that produces a permanent reduction in thyroid activity is A. 131I B. reduction of thyroid activity. Amiodarone C. Propranolol D. Propylthiouracil E. Triiodothyronine 442. This drug when given IV and 225 B. Verapamil can also decrease AV node conduction however via blocking L supraphysiologic dose will cause type Ca channels and not K channels. Fleicanide is a class IC drug that affects conduction block on AV node via increasing primarily Na channels present in both atria and ventricles. Amiodarone is the most K efflux? A. Verapamil IV B. Adenosine C. efficacious of all antiarrhythmic drug since it blocks Na, Ca, K and Beta receptors. Fleicanide D. Amiodarone E. None of the the question is specific on its action in reference to AV node. the best answer is above adenosine. 443. This factor indicates the spread of local 693 B. Answer: B Density of local anesthetic in relation to density of CSF at normal anesthetic in CSF? A. Addition of narcotic to body temp.is termed as baricity. Degree of spread is determnined primarily by local anesthetic. B. Density of local density of anesthetic & patient’s position. anesthetic C. Patient’s body surface area D. Dose administered 444. This halogen in a 1:20,000 solution is 515 D. Katzung 10th ed., 822. Phenolic disinfectants are the oldest of the surgical bactericidal in 1 minute and kills spores in 15 antiseptics. They are used for hard surface decontamination in hospitals and labs. minutes. It is the most active antiseptic for They are no longer used as a disinfectant because of its corrosive effect on tissues, intact skin: A. Phenol B. Hypochlorus acid C. its toxicity when absorbed (hyperbilirubinemia in newborns) and its carcinogenic Sodium hypochlorite D. Iodine E. effect. Sodium hypochlorite (household bleach) in 1:10 dilution which provides 5000 Hexachlorophene ppm of available chlorine is recommended by CDC for disinfection of blood spills and to kill spores. 445. This is a mucosal protective drug used as an 440 D. adjunct treatment of PUD. It binds to injured tissues and forms a protective covering over ulcer beds, resulting in accelerated healing: A. Misoprostol B. Bismuth salicylate C. Calcium carbonate D. Sucralfate 446. This is not a characteristic of phase I clinical 343 B. Determination of drug efficacy is done during phase II of clinical trials. trials in drug development: A. Pharmacokinetic measurements are done B. Determination of drug’s efficacy C. Determination of maximum tolerated dose D. Subjects are healthy volunteers 447. This is the anti-Asthma drug which is only of 644 D. worth if given as prophylaxis. A. Salbutamol B. Budesonide C. Salmeterol D. Cromolyn E. Hydrocortisone 448. This is the only ligand-gated serotonin 489 C. receptor: A. 5HT1 B. 5HT2 C. 5HT3 D. 5HT4 449. This is time of drug solubility and absorption 661 C. polar, ionized - less soluble in lipids, excreted; non - polar, unionized - more and clearance? A. Ionized drugs are better soluble, absorbed absorbed in the GIT B. Polar drugs have decreased clearance C. Non-ionized drugs are better absorbed in the GIT D. Non-polar drugs have increased clearance E. None of the above 450. This opioid antagonist is used in opioid and 190 B. Naltrexone reduces craving in alcohol dependence. Note: there are alcohol dependence A. Nalmefene B. probably 4 questions about opiates in our boards. Naltrexone C. Naloxone D. Nalbuphine E. A and C 451. this oral iron compound has the 563 C. Ferrous sulfate (FEOSOL, others) is the hydrated salt, FeSO4×7H2O, which contains highest elemental iron content: a. 20% iron. Dried ferrous sulfate (32% elemental iron) is also available. Ferrous fumarate ferrous sulfate b. ferrous (FEOSTAT, others) contains 33% iron and is moderately soluble in water, stable, and almost gluconate c. ferrous fumarate d. tasteless. Ferrous gluconate (FERGON, others) also has been successfully used in the iron dextran therapy of iron-deficiency anemia. The gluconate contains 12% iron. Polysaccharide-iron complex (NIFEREX, others), a compound of ferrihydrite and carbohydrate, is another preparation with comparable absorption. The effective dose of all of these preparations is based on iron content. 452. This prostaglandin F2 alpha 116 D. Page 32 of Topnotch Handout. Latanoprost is commonly used off label for eyelash analog is commonly used off label lengthening. It is a prostaglandin F2alpha analog that activates FP receptors, increases for its side effect which is outflow of aquaeous humor and reduces intraocular pressure. Its other side effects are lengthening of eyelashes: A. Blurred vision, eye irritation, or tearing darkening of eyelid skin color, increase in brown Beraprost B. Dinoprostone C. color in colored part of eye. Alprostail D. Latanoprost E. Gemeprost 453. This regimen is utilized in the 334 B. ABVD - adriamycin, bleomycin, vincristine, dacarbazine, prednisone. PEB (cisplatin, treatment of Hodgkin lyphoma: A. etoposide, bleomycin) regimen is used in testicular CA. CHOP (cyclophosphamide, PEB regimen B. ABVD regimen C. doxorubicin, vincristine, prednisone) regimen in non-Hodgkin lymphoma, and GnRH agonist CHOP regimen D. GnRH agonist and androgen receptor antagonist in prostate CA treatment. and androgen receptor antagonist 454. This route of drug administration is 502 C. Katzung 10th ed, p. 41 the most convenient, however bioavailability may be less than 100% because of incomplete absorption and first-pass elimination. A. Intramuscular B. Subcutaneous C. Oral D. Transdermal E. All of the above 455. Though 1% is systemically 159 B. Bronchodilation is still the net effect of ipratropium, the systemic effect is negligent as absorbed when Ipratropium the drug is administered to its site of action which is the lungs. Bromide is given via nebulization, what is its net effect? A. Reflex tachycardia B. Bronchodilation C. Bronchoconstriction D. Laryngeal spasm E. None of the above 456. Throbbing headaches in patients 426 B. taking nitrates are mainly attributed to: A. Chemical toxicity B. Meningeal artery vasodilation C. Reflex tachycardia D. Abrupt lowering of BP 457. Thru what route is the usual portal 101 C. Page 94 of Topnotch Handout. The usual setting of either acute or chronic inorganic of entry in Inorganic Mercury mercury poisoning is thru Inhalation of inorganic mercury vapor. poisoning? A. Ingestion B. Topical application C. Inhalation D. Rectal E. None of the above 458. Tocainide is an antiarrhythmic 390 B. agent classified under which group? A. Class IA B. Class IB C. Class IC D. Class II E. Class III 459. To improve the quality of life of patients with on-going 412 C. Flutamide an Androgen Antagonist is given in adjunct to treatment for Prostate Adenocarcinoma with Leuprolide to lessen the side effects of Leuprolide like Gynecomastia, Leuprolide, which of the following medications should decreased libido, Apoplexy and hot flushes.(SIMILAR TO PREVIOUS be added? A. Anastrazole B. Examestane C. Flutamide BOARD EXAM CONCEPT/PRINCIPLE) D. Prednisone E. Morphine 460. Tolerance is seen in which of the following drugs: A. 56 A. Tolerance develop with nitrates when used for more than 8 Nitrates B. Dopamine C. Epinephine D. Aspirin E. AOTA hours without interruption 461. A topical preparation that are concentric spherical 580 A. Newer vehicles include liposomes and microgel formulations. shells of phospholipids in an aqueous medium that may Liposomes are concentric spherical shells of phospholipids in an enhance percutaneous absorption. a. Liposome b. aqueous medium that may enhance percutaneous absorption. Microgrel c. Transfersome d. Micelle Variations in size, charge, and lipid content can influence liposome function substantially. Liposomes penetrate compromised epidermal barriers more efficiently (Korting et al., 1991). Microgels are polymers that may enhance solubilization of certain drugs, thereby enhancing penetration and diminishing irritancy. 462. Tranexamic acid is used to prevent and treat acute 432 A. Plasminogen are converted to plasmin whose function is to lyse bleeding episodes in patients who are at high risk, the formed fibrin. When it is inhibited, hemostasis is promoted. especially those who have hemophilia and intracranial aneurysms. The mechanism of action of this drug involves: A. Inhibition of plasminogen activators B. Inhibition of plasmin-degrading factors C. Potentiation of factor VIII activity D. Delivery of Vitamin K to the clotting factor-synthesizing cells 463. Treatment for inflammatory bowel disease: A. 433 B. Sulfasalazine is poorly absorbed in the GIT and therefore it is Sulfadiazine B. Sulfasalazine C. Sulfamethoxazole D. believed that it has its main action in lumen. It is used to treat IBDs by Sulfatide reducing the synthesis of inflammatory mediators known as eicosanoids and inflammatory cytokines. However, unlike glucocorticoids, sulfasalazine is also a mild immunosuppressant. 464. A tricyclic antidepresssant drug long used to reduce 506 B. Katzung 115: Ipratropium-is an anti-muscarinic drug used as an incontinence in institutionalized elderly patients and in inhalational drug for asthma. Scopolamine- is an antimuscarinic drug children with bedwetting problems because of its strong used to remedy seasickness. Oxybutinin- is an antimuscarinic selective anti-muscarinic actions: A. Ipratropium B. Imipramine C. for M3 receptors used to relieve bladder spasm after urologic surgery Scopolamine D. Oxybutinin E. Fluoxetine and is also valuable in reducing involuntary voiding in patients with neurologic disease. Fluoxetine is an SSRI. 465. True of Acetaminophen: a. It covalently binds to COX 572 B. Observational studies suggest that acetaminophen, which is a , inhibiting prostaglandin synthesis b. very weak very weak antiinflammatory agent at the typical daily dose of 1000 mg antiinflammatory agent at the typical daily dose of 1000 is associated with a reduced incidence of gastrointestinal adverse mg c. incidence of Gastrointestinal adverse effects effects compared to tNSAIDs comparable to Ibuprofen d. its inhibition of thromboxane A2 synthesis discourages its use in dengue cases 466. Type II DM with insulin resistance will be best treated 360 D. with which of the following: A. Sulfonylureas B. Gliptins C. Insulin glargine D. TZDs 467. Upon taking Penicillin, a 20 year-old male suddenly 453 D. Epinehrine is a sympathomimetic, nonselective and direct- experienced respiratory distress. A drug would be most acting. It is helpful in patients with anaphylactic shock. Respiratory helpful to treat the respiratory symptoms was given in distress is treated by Epinephrine's action on beta 2 receptors resulting this patient. In comparison to norepinephrine, to bronchodilation. epinephrine has more effects on: A. alpha 1 receptors B. alpha 2 receptors C. beta 1 receptors D. beta 2 receptors E. dopamine receptors 468. Use of aspirin as an anti-platelet inhibits primarily which 342 B. prostanoid? A. Prostacyclin B. Thromboxane C. Prostaglandin D. Epoprosterenol 469. Using drugs to block which of the following chemical mediators 252 D. SIMILAR TO PREVIOUS BOARD EXAM could potentially decrease leukocyte chemotaxis? A. CONCEPT/PRINCIPLE, but leukotrienes could also serve as prostaglandin H B. cyclooxygenase C. bradykinin D. chemotactic factors leukotrienes E. serotonin 470. Using the Henderson-Hasselbach equation, in a pH above the 102 D. Page 2 of Topnotch Handout. According to the dissociation constant, a weak acid is present in the _____________ Henderson-Hasselbach equation Weak acid above the pKa is form. A. unprotonated, uncharged B. protonated, charged C. unprotonated and charged. protonated, uncharged D. unprotonated, charged E. unpredictable 471. Vancomycin is administered orally for this condition: A. Sepsis B. 352 C. Endocarditis C. Antibiotic-associated enterocolitis D. Meningitis 472. A vinca alkaloid acting primarily in the M phase of cancer cell 439 B. cycle that is useful in the treatment of acute leukemias, lymphomas and neuroblastoma: A. Paclitaxel B. Vincristine C. Doxorubicin D. Cytarabine 473. Warfarin is prescribed to a 40 year-old male. The excessive 454 E Bleeding with warfarin can be reversed by anticoagulant effect results to bleeding in this patient can be administration of Vitamin K, FFP and by discontinuance of the reversed by: A. Administration of vitamin K B. Administration of drug. fresh frozen plasma C. Discontinuance of the drug D. A and C E. All of the above 474. What anti-retroviral agent causes peripheral neuropathy and 184 C. SIMILAR TO PREVIOUS BOARD EXAM pancreatitis? A. Zidovudine B. Zalcitabine C. Didanosine D. CONCEPT/PRINCIPLE.. Zidovudine causes bone marrow Abacavir E. Foscarnet suppression. Abacavir causes hypersensitivity. Didanosine causesperipheral neuropathy and pancreatitis. Stavudine and Zalcitabine causes peripheral neuropathy. 475. What anti-TB medication readily penetrates into phagocytic 80 B. As stated verbatim in Katzung, rifampicin readily cells and can kill organisms that are poorly accessible to many penetrates most tissues and penetrates into phagocytic cells. other drugs, such as intracellular organisms and those It can kill organisms that are poorly accessible to many other sequestered in abscesses and lung cavities? A. Isoniazid B. drugs, such as intracellular organisms and those sequestered Rifampicin C. Ethambutol D. Pyrazinamide E. Streptomycin in abscesses and lung cavities. 476. What class of anti-hypertensive drugs are also used in BPH? A. 419 C. Alpha 1-A receptor is specifically seen in the Urinary Alpha 1A Agonists B. Alpha 1B Agonists C. Alpha 1A Antagonists tract most especially the Prostate.Alpha 1A D. Alpha 1B Antagonists E. Alpha 2 Agonists antagonists/Quinazolines include Prazosin Terazosin, and Tamsulosin 477. What drug acts on epithelial sodium channels in cortical 199 D. Triamterene and Amiloride are potassium sparing collecting ducts and is used to reduce potassium excretion in diuretics that act on epithelial sodium channels (ENaC) in cases of hypokalemia? A. furosemide B. acetazolamide C. cortical collecting duct, causing decreased Na reabsorption spironolactone D. triamterene E. hydrochlorothiazide and K excretion. Spironolactone and eplerenone are also potassium sparing diuretics but they act on aldosterone receptors. 478. What drug should be given to an immunocompromised patient 220 D. Ganciclovir is the drug of choice for CMV. Foscarnet is with CMV retinitis? A. Acyclovir B. Vidarabine C. Indinavir D. an alternative. Ganciclovir E. AZT 479. What enzyme do statins inhibit? A. HMG-CoA synthase B. HMG- 15 B. Statins inhibit the rate-limiting step of cholesterol CoA reductase C. HMG-CoA lyase D. HMG-CoA transferase A. synthesis. Don’t get confused with HMG CoA synthase All of the above which is the rate limiting step of ketogenesis. Source: Katzung Basic and Clinical Pharmacology 11th ed p 612 480. What hypertensive drug commonly used for the outpatient 587 C. Methyldopa is a selective alpha 2 agonist. treatment of preeclampsia is prominent for its side-effect of hemolytic anemia with a positive Coomb's test? A. α1 adrenergic agonist B. α2 adrenergic antagonist C. α1 adrenergic agonist D. α2 adrenergic antagonist E. non-selective α antagonist 481. What is the anti-hypertensive drug of choice for patients with 70 C. ACE inhibitors are renal efferent arteriole dilators diabetic nephropathy? A. Nifedipine B. Clonidine C. Enalapril D. decreasing albumin excretion and slowing progression of Atenolol E. Thiazides CKD. 482. what is the dose limiting toxicity of vincristine ? A. areflexia B. 320 A. vincristine does not cause serious myelosuppresive Bone marrow suppression C. Acute tubular necrosis D. effect however it induces neurotoxicity such as areflexia, Hepatocyte degeneration E. None of the above ileus and peripheral neuritis. Katzung review 8th ed p 455. 483. What is the drug of choice for all forms of schistosomiasis and 238 B. DEC is the drug of choice for loa loa infection, it is paragonimiasis? A. Diethylcarbamazine B. Praziquantel C. also an alternative drug for elephantiasis. Ivermectin - DOC Ivermectin D. Albendazole E. Piperazine for onchocerciasis, cutaneous larva migransm and strongyloides. Albendazole- ascariasis, hookworm, pinworm, whipworm, cysticercosis and hydatid disease. Piperazine is an alternative drug for ascariasis. 484. What is the drug of choice for paroxysmal supraventricular 474 B. Esmolol, sotalol, verapamil are all used for the tachycardia? A. Esmolol B. adenosine C. verapamil D. Sotalol E. treatment of supraventricular tachycardia. For paroxysmal Any of the choices supraventricular tachycardia, the drug of choice is adenosine. 485. what is the drug of choice for restless leg syndrome? A. 314 D. the pathologic basis for restless leg syndrome is haloperidol B. aripriprazole C. Citalopram D. Ropinirole E. dopamine blockade. Agonist to dopamine receptors such as Propanolol pramipexole or ropinirole is very effective in treating this condition. Katzung review 8th ed p 235 486. What is the drug of choice for the cyst carrier state of Entamoeba 438 A.Diloxanide furoate is a luminal amoebicide which is histolytica? A. Diloxanide furoate B. Metronidazole C. Tinidazole hydrolysed in the gut, thus releasing the free diloxanide D. Paramomycin which acts as an amoebicide. It is given alone in asymptomatic cyst passers. For patients with active amoebic infections, it can be administered along with metronidazole. 487. What is the drug of choice for the eradication of dormant liver 514 B. Primaquine is the only available agent active against forms of Plasmodium vivax and Plasmodium ovale? A. Quinidine the dormant hypnozoite stages of p. vivax and p. ovale. B. Primaquine C. Mefloqione D. Chloroquine E. Lumefantrine (radical cure) Katzung 10th ed., 852 488. What is the drug of choice for the rapid termination of 72 A. Adenosine is the drug of choice for paroxysmal SVT. paroxysmal supraventricular tachycardia? A. Adenosine B. Magnesium is the drug of choice for Torsades de pointes. Magnesium C. Lidocaine D. Amiodarone E. Verapamil Lidocaine is the drug of choice for post MI arrythmias and digoxin induced arrythmias. 489. What is the drug of choice for trigeminal neuralgia? A. 647 D. SIMILAR TO PREVIOUS BOARD EXAM Lamotrigine B. Gabapentin C. Pregabalin D. Carbamazepine E. B CONCEPT/PRINCIPLE or C 490. What is the drug of choice in acute arrhythmias A. Metoprolol B. 144 B. Lidocaine C. Quinine D. Bisoprolol E. None of the above 491. What is the drug of choice in preventing the recurrence of 145 A. arryhthmia A. Adenosine B. Procainamide C. Lidocaine D. Carvedilol E. None of the above 492. What is the half-life of alcohol? A. 2 hours B. 4 hours C. 8 hours D. 541 E Ethanol, over most of its plasma concentration range, 12 hours E. None of the above exhibits zero-order elimination. Thus, there is no constant half life of elimination as in drugs exhibiting first order elimination. (Katzung Board Review 9e, p.6) 493. What is the major advantage of Etomidate over other intravenous 292 B. Etomidate has no analgesic effects. Distribution is anesthetics? A. Has greater analgesic effects B. Causes minimal rapid from the brain to highly perfused tissue hence a cardiovascular and respiratory depression C. Relatively longer relatively short duration of its anesthetic effect. It has a high duration of effect D. Has a lower incidence of postoperative incidence of postoperative nausea and vomiting, pain and nausea and vomiting E. Has a rapid onset of recovery when myoclonic activity. Initial recovery is less rapid compared compared with Propofol with propofol 494. What is the maximum dose of Vincristine? A. 100 micrograms B. 482 D. SIMILAR TO PREVIOUS BOARD EXAM 200 micrograms C. 1 milligram D. 2 milligrams CONCEPT/PRINCIPLE 495. What is the mechanism of action of 181 B. A- fluoroquinolones. C- nitrofurantoin. metronidazole? A. Inhibits DNA replication by binding to DNA gyrase B. Reactive reduction by ferredoxin forming free radicals C. Forms multiple reactive intermediates when acted upon by bacterial enzyme D. Unknown E. None of the above 496. what is the mechanism of action of nystatin? A. 317 A. nystatin and ampothericin B are both polyene antifungal antibiotics. It Formation of artificial pore in the fungal binds to ergosterol and creates an artificial pores in the fungal membrane membrane B. Inhibit ergosterol synthesis C. Inhibit leading to increase membrane permeability. Katzung review 8th ed p 401 microtubular formation D. Inhibits squalene epoxidase E. Inhibit the synthesis of B(1-2) glycan 497. What is the mechanism of action of PTU? A. 191 D. There are 5 steps in thyroid hormone synthesis. 1. iodide trapping; 2. Blocks iodination and organification B. Inhibits iodination/organification; 3. coupling; 4. secretion; 5. peripheral conversion. iodide trapping C. Blocks coupling reaction D. Steps 2 to 5 are actually blocked by PTU but the main effect among these is Blocks peripheral conversion of T4 to T3 E. the inhibition of peripheral conversion. Inhibits secretion 498. What is the most important drug in pulmonary 322 A. tuberculosis? A. INH B. Pyrazinamide C. Rifampin D. Ethambutol 499. What is the most serious side effect of Methanol 231 A. Methanol or wood alcohol is metabolized by alcohol dehydrogenase ingestion? A. Retinal damage B. Hepatotoxicity C. to produce a product called Formaldehyde and formic acid. This metabolite Blood dyscrasia D. Nephrotoxicity E. causes severe acidosis and prominent retinal damage. Nephrotoxicity is a Neurotoxicity- status epilepticus major side effect of ethylene glycol poisoning via its oxalic acid metabolite. treatment for both alcohol is by administration of Fomepizole ( inhibitor alcohol dehydrogenase) and ethanol ( competitive substrate for alcohol dehydrogenase). 500. What is the MOST-SLECTIVE alpha 1 selective 669 B. However, all of the mentioned drugs are alpha-1 selective antagonists antagonist used in the treatment of BPH? A. used in the treatment of BPH (Prazosin, Tamsulosin, Terazosin, Doxazosin). Prazosin B. tamsulosin C. Terazosin D. Doxazosin Tamsulosin is the most selective alpha-1 selective antagonist. . Tamsulosin is a E. all of the above selective α1 receptor antagonist that has preferential selectivity for the α1A receptor in the prostate versus the α1B receptor in the blood vessels. When alpha 1 receptors in the bladder neck and the prostate are blocked, this causes a relaxation in smooth muscle and therefore less resistance to urinary flow. Due to this the pain associated with BPH can be reduced. 501. What is the pharmacologic basis of using Timolol 221 C. MOA: letter A- alpha agonist and epinephrine; B- Cholinomimetics ( in open angle glaucoma? A. Increaed outflow via physostigmine, pilocarpine) D- there is no such thing cAMP dependent dilatation of uveoscleral veins B. It leads to decrease in bicarbonate; however inhibition of Carbonic anhydrase leads to opening of trabecular meshwork C. It suppresses decrease humor production via decreased bicarbonate levels. the ciliary epithelium from producing aqueous humor D. lt decrease production of aqueous humor by decreasing levels of bicarbonate through cAMP mediated pathway. E. none of the above 502. What is the predominant form of a weakly acidic 466 A. The physiologic pH is at 7.35-7.45 which is above the pKa of this drug drug with pKa of 5 at physiologic pH? A. 5. At pH above pKa, weakly acidic drug is predominantly unprotonated Unprotonated charged B. Protonated charged C. charged, if it is below pKa, the predominant form is protonated uncharged. Unprotonated uncharged D. Protonated uncharged E. None of the choices 503. What is the predominant reason why 248 D. SIMILAR TO PREVIOUS BOARD EXAM CONCEPT/PRINCIPLE, high nitroglycerin is generally preferred not to be first pass effect is the best answer, low bioavailability through the oral route given via the oral route? A. could potentially due to liver metabolism cause dangerous systemic hypotension B. is considerably less effective when given through this route C. is immediately inactivated by exposure to low gastric pH D. there is high first pass effect E. food in the stomach significantly retards absorption 504. what is the primary drug of treatment for 319 A. B,C,D has no clinical effectivity in hydatid disease. ascariasis, pinworm, whipworm and hydatid disease? A. albendazole B. Mebendazole C. Pyrantel pamoate D. Diethylcarbamazine E. All of the above 505. what is the recommended treatment for a 29 y/o 312 B. single dose administration of spectinomycin is effective in treating G1PO female diagnosed with Gonnorrhea Gonnorhea in a patient who has severe beta lactam hypersensitivity. It has infection who has severe hypersensitivity to lesser risk of teratogenicity compared to conventional aminglycosides. pennicilin? A. erythromycin B. Spectinomycin C. Clindamycin is active only on gram positive and anaerobic organisms. Cetriaxone D. Doxycycline E. Clindamycin ceftriaxone is a 3rd generation cephalosporin effective against Gonorrhea infection however it should be avoided since the patient has beta lactam hypersensitivity. Doxycycline should never be given for pregnant patients. erythromycin is not effective treatment for gonorrhea infection. Katzung 11th ed p 813. 506. What is the second messenger system for the 209 A. Beta-2 receptors are coupled to the cAMP second messenger system. receptors blocked by metoprolol? A. cAMP B. IP3 C. DAG D. cGMP E. PLC 507. What is the site of action of corticosteroids in 229 C. Corticosteroids exerts its anti inflammatory effect by inhibiting the terms of its antiinflammatory effect? A. Phospholipase A2 enzyme which is responsible for generation of arachidonic Cyclooxygenase 1 and 2 B. Phospholipase C C. acid from membrane lipids. Phospholipase C is not connected with Phospholipase A2 D. Thromboxane synthase E. corticosteroids and its function is to generate IP3 and DAG. Cyclooxygenase All of the above and thromboxane synthase is primarily inhibit by Coxibs and NSAIDS. 508. What is the treatment for malignant 12 A. Source: Topnotch handout on Pharmacology hyperthermia? A. Dantrolene B. Nitroglycerin C. Paracetamol D. Halothane E. All of the above 509. What part of the pharmacokinetics of penicillin is 204 C. Probenecid raises plasma concentration of penicillin by inhibiting its affected by the drug probenecid? A. Glomerular tubular secretion in the renal tubules. filtration B. Tubular reabsorption C. Tubular secretion D. Hepatic uptake E. Protein-binding with albumin 510. What type of muscarinic receptors is used by the 468 B. M1 - nerve endings, M2-heart, M3 - smooth muscle, glands, heart? A. M1 B. M2 C. M3 D. N1 E. N2 endothelium 511. When a pregnant 18 year old female delivered 247 C. SIMILAR TO PREVIOUS BOARD EXAM CONCEPT/PRINCIPLE, know her child, her baby was noted to have the common syndromes and teratogen, answer here is fetal alcohol syndrome characteristic small palpebral fissures, smooth philtrum and thin upper lip, which of the following substances when ingested by the mother during pregnancy could potentially be the cause of such changes? A. phenytoin B. warfarin C. ethanol D. captopril E. isotretinoin 512. Which aminoglycoside has the narrowest 435 C. therapeutic window but with the widest coverage? A. Gentamicin B. Streptomycin C. Amikacin D. Kanamycin 513. Which among the 258 E know the mechanism understand them, know and master normal physiology, digoxin is a negative following cardiac drugs chronotropic but positive inotropic inc contractility, captopril inhibits ACE, acts on the RAAS decreases can decrease preload, preload by decreasing aldosterone and sodium water retention hence decrease blood volume dec afterload and venous return, amlodipine calcium antagonist predominantly on the vessels decrease afterload by contractility? A. digoxin dilating resistance vessels, hydrochlorthiazide decreases preload by increasing secretion of both B. captopril C. sodium water, propranolol, non selective beta blocker, decreases renin secretion hence RAAS through amlodipine D. B1 blockage this includes angiotensin 2 so decreased both preload afterload, decreases contractilitty hydrochlorthiazide E. through B1 blockade, propranolol 514. Which among the 652 A. Carbonic anhydrase inhibitors following diuretics can also be used for mountain sickness? A. Acetazolamide B. Furosemide C. Thiazide D. Mannitol E. None 515. Which among the 168 D. SIMILAR TO PREVIOUS BOARD EXAM CONCEPT/PRINCIPLE. Katzung 11th edition page 514. following drugs deliver Fluoxetine is a SSRI. Venlafaxine is a SNRI. Imipramine is a TCA. Selegiline is MAOi. Trazadone is a its action through serotonin antagonist. selectively inhibiting reuptake of serotonin? A. venlafaxine B. imipramine C. selegiline D. fluoxetine E. trazodone 516. Which among the 659 A. following drugs is commonly used in controlling the manic phase of a bipolar patient? A. Lithium B. Selegiline C. Reserpine D. Haloperidol E. Biperiden 517. Which among the 654 C. Loperamide is an opioid that acts on Kappa receptors to increase gastric emptying time. following is/are opioid agonist? A. Hyoscine B. Metoclopramide C. Loperamide D. Attapulgite E. AlMg(OH) 518. Which among the 143 A. The opioid ohmefentanyl is the most potent opioid following is the most potent opioid A. Ohmefentanyl B. Morphine C. Fentanyl D. Pethidine E. Remifentanil 519. Which among the lipid- 648 B. lowering medicatins causes flushing? A. Statins B. Niacin C. Cholestyramine D. A and B E. B and C 520. which anong the following drugs 564 B. Phenobarbitals should be avoided in patients diagbosed with porphyria should be avoided in patients with porphyria? a. phenytoin b. secobarbital c. midazolam d. sodium valproate 521. Which antifungal drug inhibits β- 362 E Terbinafine - inhibits squalene oxidase; Ketoconazole - inhibits synthesis of glucan synthase causing a ergosterol; Fluconazole - inhibits thymidylate synthase; Ampho B - polyene causes disruption in cell wall synthesis? A. disruption of fungal cell walls Katzung 12th ed pg 857 Terbinafine B. Ketoconazole C. Amphotericin B D. Fluconazole E. Caspofungin 522. Which antihelminthic drugs rapidly 367 B. MOA of mebendazole: Mebendazole probably acts by inhibiting microtubule kills adult worms presumably synthesis; Praziquantel: increase the permeability of trematode and cestode cell membranes through inhibition of oxidative to calcium, resulting in paralysis, dislodgement, and death; Pyrantel pamoate: phosphorylation or stimulation of neuromuscular blocking agent that causes release of acetylcholine and inhibition of ATPase activity? A. Mebendazole B. cholinesterase which results in paralysis of worms followed by expulsion; Ivermectin: Niclosamide C. Praziquantel D. paralyze nematodes and arthropods by intensifying γ-aminobutyric acid Pyrantel pamoate E. Ivermectin (GABA)â€"mediated transmission of signals in peripheral nerves. Katzung 12th ed pg 941- 945 523. Which antimalarial drug should not 219 E be given to a patient with glucose- 6-phosphate dehydrogenase deficiency? A. Chloroquine B. Quinine C. Atovaquone-proquanil D. Mefloquine E. Primaquine 524. Which anti-parasitic drug exerts its 649 A. SIMILAR TO PREVIOUS BOARD EXAM CONCEPT/PRINCIPLE actions on the microtubules of helminths? A. Mebendazole B. Stibogluconate C. Ivermectin D. Diloxanide Furoate E. Metronidazole 525. Which antiretroviral drug acts 212 D. Amantadine is also effective for influenza. through the M2 ion channel? A. Efavirenz B. Nevirapine C. AZT D. Amantadine E. Indinavir 526. Which class of antibiotics can be 351 D. safely given during pregnancy? A. Tetracycline B. Aminoglycoside C. Fluoroquinolones D. Beta-lactams 527. Which diuretic is found to have 427 C. long-term benefits and can reduce mortality in chronic heart failure? A. Thiazide B. Furosemide C. Spironolactone D. Ethacrynic acid 528. Which drug is most effective in 332 A. raising HDL? A. Niacin B. Fibrates C. Statins D. Bile acid sequestrants 529. Which drugs reduces the risk of 593 A. Clozapine is the anit-psychotic that reduces the risk of suicide. suicide? A. Clozapine B. Clonidine C. Clonazepam D. Clofazimine E. Clofibrate 530. Which drug used in the treatment of depression 211 A. Imipramine is a tricyclic antidepressant. possess a tricyclic ring? A. Imipramine B. Fluoxetine C. Trazodone D. Venlafaxine E. Bupropion 531. Which drug when taken together with 300 C. Azathioprine is converted to mercaptopurine which is responsible for Azathioprine has resulted in severe its immunosuppressant and hematotoxicity. Allopurinol inhibits xanthine myelosupression by inhibiting the enzyme oxidase, the enzyme that metabolizes mercaptopurine. responsible for its metabolism ? A. Theophylline B. Celecoxib C. Allopurinol D. Digoxin E. Rifampicin 532. Which has the highest partition coefficient 195 A. Methoxyflurane has the highest partition coefficient which means that it among the inhalation anesthetics? A. has the slowest onset and recovery. methoxyflurane B. Nitrous oxide C. isoflurane D. sevoflurane E. desflurane 533. Which helminthic infection does not respond to 54 A. In hydatid disease, praziquantel has marginal efficacy because it does treatment with praziquantel? A. Hydatid disease not affect the inner germinal membrane of Echinococcus granulosus present B. Opisthorchiasis C. Paragonimiasis D. Pork in hydatid cysts. tapeworm infection E. Schistosomiasis 534. which in the following antipsychotic medications 310 D. Olanzapine -weight gain, clozapine - agranulocytosis, haloperidol- has the greatest risk of inducing Torsades de extrapyramidal symptoms. Thioridazine - retinal deposits. Among the atypical pointes arryhthmia? A. Fluphenazine B. antipsychotics, ziprasidone has the greatest risk of inducing QT prolongation Clozapine C. Thioridazine D. Ziprasidone E. leading to ventricular arrhythmias. Kazung review p 242. Haloperidol 535. Which in the following choices can be primarily 237 D. Sodium stibogluconate - treatment of all forms of leishmaniasis use for the treatment of hemolymphatic stage of infection; Metronidazole is not effective in the said infection, it is use for trypanosomiasis and for Pneumocystis jiroveci giardia and trichomonas infection. Nifurtimox is for chagas diseae ( T cruzi infection A. Sodium Stibogluconate B. infection). Cotrimoxazole is used for P jiroveci but not effective in Metronidazole C. Nifurtimox D. Pentamidine E. Trypanosomiasis infection. Cotrimoxazole 536. Which in the following drugs is classified as 227 D. Naratriptan and its prototype Sumatriptan are 5HT 1D agonist use for 5HT1D agonist? A. Tegaserod B. Ketanserin C. the treatment of acute migraine and cluster headache. Tegaserod indicated Ondansetron D. Naratriptan E. None of the for constipation is a 5HT4 partial agonist.Ondansetron is a 5-HT3 antagonist above ise for antiemesis in post operative and postchemotherapy patients. Ketanserin is a 5HT2 antagonist use for controlling the systemic effects of carcinoid syndrome. 537. Which nondepolarizing neuromuscular blocker 120 C. Page 53 of Topnotch Handout. Atracurium is broken down to inactive undergoes rapid breakdown by Hoffman metabolites by (minor) ester hydrolysis and spontaneous Hoffman elimination elimination? A. Vecuronium B. Mivacurium C. (major pathway) to Laudanosine. Atracurium D. Tubcurarine E. Rocuronium 538. Which novel and innovative drug is used to treat 599 B. Abciximab - Glycoprotein Iib/IIIa (anti-platelet) Omalizumab - IgE rheumatoid arthritis through its action as an (asthma) Infliximab - TNF (inflammatory bowel disease) Trastuzumba - antibody against the protein CD20, found in the Her2neu (Breast cancer) surface of B-cells? A. Abciximab B. Rituximab C. Omalizumab D. Infliximab E. Trastuzumab 539. Which of the ff anticonvulsant agents do not 76 E Ethosuxime blocks the T type Calcium current in the thalamus and is the exerts its effect through Na channel blocakade? drug of choice for absence seizures. All the other drugs block Na channels. A. Phenytoin B. Carbamazepine C. Topiramate D. Valproic acid E. Ethosuximide 540. Which of the ff drugs acts to improve aqueous 67 A. Pilocarpine is a cholinomimeti agent that contracts the ciliary muscle humor outflow in patients with glaucoma? A. opening the trabecular meshwork. All of the other drugs decreases secretion Pilocarpine B. Mannitol C. Apraclonidine D. of aqueous humor Timolol E. Acetazlamide 541. Which of the ff electrolyte abnormality will increase digitalis 71 B. Digitalis toxicity is increased in patients with toxicity? A. Hyponatremia B. Hypokalemia C. Hypocalcemia D. hypokalemia, hypomagnesemia and hypercalcemia. Hypermagnesemia E. Hyperphosphatemia Potassium and digitalis interact by inhibiting each other’s binding to Na+/K+-ATPase; therefore, hyperkalemia reduces the enzyme-inhibiting actions of cardiac glycosides, whereas hypokalemia facilitates these actions. 542. Which of the ff statements regarding drug elimination is correct? A. 61 B. Elimination of parent molecules does not terminate Elimination of parent molecules terminats the drug's action for drugs the drug's action for drugs with active metabolites. First with active metabolites B. First order eliminination occurs when the order kinetics exhibits the characteristic half life of most concentration of the drug decreases exponentially over time C. Zero drugs. Zero order kinetics occurs when drugs have order kinetics exhibits the characteristic half life of most drugs D. saturated their elimination mechanisms. Phenobarbital First order kinetics occurs when drugs have saturated their does not display zero roder kinetics elimination mechanisms E. Phobarbital is an example of a drug that exhibit zero order kinetics 543. Which of the ff teratogenic drugs causes atrialization of the right 64 B. Ebstein's anomaly from lithium is a congenital heart ventricle? A. Misoprostol B. Lithium C. Ethanol D. Penicillamine E. defect in which the septal and posterior leaflets of the Phenytoin tricuspid valve are displaced towards the apex of the right ventricle of the heart. There is subsequent 'atrialization' of a portion of the morphologic right ventricle 544. Which of the following adrenergic receptors in the ciliary body of 389 B. Activation of alpha-2 receptors decreases the eye causes a decrease in aqueous humor production when aqueous humor production, while activation of either activated? A. alpha-1 B. alpha-2 C. beta-1 D. beta-2 E. beta-3 beta-1 or beta-2 receptors increases aqeous humor production. Alpha-1 and beta-3 receptors have no known function in relation to aqueous humor production. 545. Which of the following agents will increase myocardial contractility 627 D. Dobutamine increases myocardial contractility, but with the LEAST effect on total peripheral resistance? A. Epinephrine it has little effect on TPR. Beta 1 selective. B. Phenylephrine C. Terbutaline D. Dobutamine E. Carbachol 546. Which of the following analgesics would you avoid in a patient with 387 C. history of epilepsy as it is known to lower the seizure threshold? A. Ketorolac B. Aspirin C. Tramadol D. Celecoxib E. Indomethacin 547. Which of the following anesthetic agent has analgesic property and 127 E Nitrous oxide , not nitric oxide, is used as an used as an adjunct to other anesthetics? A. Halothane B. Nitric oxide anesthetic agent with analgesic property. Fentanyl is C. Etomidate D. Desflurane E. Fentanyl often used for anesthesia and analgesia. SIMILAR TO PREVIOUS BOARD EXAM CONCEPT. 548. Which of the following antiarrhythmics primarily exerts its action on 588 C. Class 2 anti arrhythmics or B blockers exert its phase 4 of the cardiac action potential, prolonging PR interval? A. effort on phase 4 and prolongs the PR interval Procainamide B. Lidocaine C. Propanolol D. Amiodarone E. Verapamil 549. Which of the following antibiotic agents is bacteriostatic? A. 4 D. Generally, cell wall inhibitors are â€"cidal, drugs Piperacillin B. Vancomycin C. Gentamycin D. Doxycycline E. Co- which interfere with DNA are â€"cidal except trimoxazole sulfonamides which are only static unless they are given with trimethoprim/pyrimethamine (in this case synergism results to cidal activity) Protein synthesis inhibitors are â€"static except aminoglycosides because they cause frameshift mutations by misreading of the genetic code. Source: Topnotch handout on Pharmacology 550. Which of the following antibiotics are highly protein bound which can 259 C. SIMILAR TO PREVIOUS BOARD EXAM act to displace other substances from albumin binding and increase CONCEPT/PRINCIPLE, sulfa drugs are usually highly free drug levels A. chloramphenicol B. ciprofloxacin C. protein bound drugs which can displace other drugs cotrimoxazole D. tetracycline E. gentamicin making the free drug available to easily permeate cell membranes or bind to receptors, it can also displace bilirubin from albumin raising bilirubin levels in neonates 551. Which of the following antibiotics does NOT 596 C. The broad class of Beta lactams including the class of monobactams follow time-dependent killing action? A. and aztreonams exhibit time- dependent killing action. Bacitracin and Amoxicillin B. Aztreonam C. Metronidazole D. vancomycin that also act on the cell wall exhibit time dependent killing. Bacitracin E. Vancomycin Metronidazole, FQ and AG are concentration- dependent. 552. Which of the following antibiotics is a protein 179 E Ciprofloxacin is a fluoroquinolone which inhibits topoisomerase. The synthesis inhibitor at 30S sub unit? A. other choices inhibits at 50s subunit. Ciprofloxacin B. azithromycin C. clindamycin D. linezolid E. Doxycycline 553. Which of the following antibiotics work by 399 D. SIMILAR TO PREVIOUS BOARD EXAM CONCEPT/PRINCIPLE. binding to the D-Ala-D-Ala terminus of bacterial Piperacillin: inhibits transpeptidase and binds to penicillin-binding proteins in cell wall precursors, thus preventing the cell membrane. Cycloserine: blocks incorporation of D-Ala into the peptidoglycan polymerization? A. Imipenem B. pentapeptide side chain of peptidoglycan. Bacitracin: interferes with a late Cycloserine C. Bacitracin D. Vancomycin E. stage in cell wall synthesis. Piperacillin 554. Which of the following anti fungals will enhance 142 C. Ketoconazole is a potent cytochrome P450 inhibitor and therefore the activity of cyclosporine? A. Itraconazole B. inhibits the metabolism of other drugs and increases their effects, griseofulvin Fluconazole C. Ketoconazole D. Griseofulvin E. is a cytochrome inducer and thus facilitates the metabolism of other drugs Terbinafine thus decreasing their effect. 555. Which of the following anti hypertensive 281 C. Intake of ACE inhibitors can produce teratogenic effect in the fetus medications when taken during pregnancy can which includes renal dysgenesis and hypoplastic skulls (hypocalvaria) thus it is cause hypocalvaria in the fetus? A. Carvedilol B. contraindicated during pregnancy Amlodipine C. Captopril D. HCTZ E. Hydralazine 556. Which of the following anti-hypertensives has 605 C. Reserpine is an indole alkaloid antipsychotic and antihypertensive drug been recalled from the market due to its side that has been used for the control of high blood pressure and for the relief of effect of producing rebound suicide? A. psychotic symptoms. Reserpine mediated depletion of monoamine Guanethidine B. Phenoxybenzamine C. neurotransmitters in the synapses is often cited as evidence to the theory that Reserpine D. Trimetaphan depletion of the neurotransmitters causes subsequent depression in humans 557. Which of the following antihypertive agents 628 D. Hydralazine is CI in ptients with myocardial ischemia. commonly used in hypertension in pregnancy is contraindicated to patients having myocardial ischemia? A. Methyldopa B. Methergine C. Clonidine D. Hydralazine E. Magnesium sulfate 558. Which of the following anti-lipid drugs would 393 C. you avoid giving to a diabetic patient due to its adverse effect of inducing hyperglycemia? A. Cholestyramine B. Egetimibe C. Niacin D. Gemfibrozil E. Colestipol 559. Which of the following antimalarial drugs may 154 D. Quinine is precipitates hemolytic crisis in patients with malaria with precipitate a hemolytic crisis in a patient with G6PD deficiency. G6PD A. Lumefantrine B. Doxycycline C. Artemether D. Quinine E. None of the above 560. Which of the following anti-malarial is 597 D. The fetus is G6PD deficient, so primaquine cannot be used. To achieve contraindicated for pregnant women? A. radical cure for pregnant women, primaquine is given after delivery of the Chloroquine B. Quinine C. Quinidine D. baby. Primaquine E. Mefloquine 561. Which of the following antimicrobial agent acts 397 A. as a dihydrofolate reductase inhibitor? A. Trimethoprim B. Sulfamethoxazole C. Metronidazole D. Nitrofurantoin E. Rifampicin 562. Which of the following anti-microbial agent is 185 E Bacteriostatic agents: All protein synthesis inhibitors except bacteriostatic? A. Co-trimoxazole B. aminoglycoside; ethambutol, nitrofurantoin; those with anti-metabolite Pyrazinamide C. Metronidazole D. mechanism of action- sulfonamide and trimethoprim (but they become Aminoglycoside E. Tetracycline bactericidal when combined). 563. Which of the following anti-neoplastic drug is highly lipophilic, 598 C. Carmustine and Lomustine are highly lipophilic, used to use for the treatment of cancers that are not readily permeable treat brain cancer. through vascular channels? A. Bleomycin B. Busulfan C. Carmustine D. Cyclophosphamide E. Carboplatin 564. Which of the following antineoplastic drugs can cause 177 A. cisplatin are nephrotoxic and ototoxic. Vincristine causes hemorrhagic cystitis? A. cyclophosphamide B. cisplatin C. peripheral neuropathy. Methotrexate usually causes vincristine D. methotrexate E. Bleomycin myelosuppression. Bleomycin is usually associated with pulmonary fibrosis 565. Which of the following antiparkinsonian drugs can cause 233 B. Katzung Review of pharmacology 8th ed pp 233. Both exacerbation of psychosis and produce ergot related effects Levodopa and bromocriptine can cause exacerbation of such as pulmonary fibrosis and erythromelalgia A. Amantadine schizophrenia by increasing brain dopamine levels. However, B. Bromocriptine C. Entacapone D. Haloperidol E. None of the only bromocriptine can cause ergot related side effects. above 566. Which of the following antiretroviral would cause 137 B. Indinavir (Protease Inhibitor) - all Pis/'navirs' may cause hyperlipidemia and hyperglycemia? A. Enfuvitide B. Indinavir C. hyperlipidemia, hyperglycemia, and insulin resistance as Zidovudine D. Delavirdine E. Didanosine potential adverse events; Efuvirtide is a fusion inhibitor, SE: Hypersensitivity, increased incidence of bacterial pneumonia, injection site reaction; Zidovudine and Didanosine (NRTI) - all NRTI carry the risk of lactic acidosis with hepatic steatosis 567. which of the following antiseizure drugs can inhibit cytochrome 232 D. Oxcarbazepine, carbamazepine, Barbiturates, Phenytoin isoenzymes leading to increase plasma concentration of other are all drug metabolizing inducers. Only Valproic acid in the drugs? A. Phenobarbital B. Lorazepam C. Oxcarbazepine D. choice is an inhibitor. Benzodiazepines do not affect the Valproic acid E. None of the above function of the cytochromes. 568. Which of the following anti-TB drug is matched incorrectly with 2 D. Anti-TB drug most associated with skin rashes is its adverse effects? A. Most hepatotoxic â€" Pyrazinamide B. streptomycin. Source: National TB Control Program Manual of Psychosis â€" Isoniazid C. Most ototoxic â€" Streptomycin D. Procedures Most associated with skin rashes due to hypersensitivity â€" Rifampicin E. Most associated with visual impairment - Ethambutol 569. Which of the following anti TB drugs is known to cause 637 C. hyperuricemia and is noted to be the most hepatotoxic? A. Rifampicin B. Ethambutol C. Pyrazinamide D. Streptomycin E. Isoniazid 570. Which of the following antitumor antibiotics is cell-cycle 396 C. Bleomycin is specific for G2 phase of the cell cycle. The specific? A. Mitomycin B. Doxorubicin C. Bleomycin D. rest of the choices are non-cell-cycle specific. Actinomycin E. None of the above 571. Which of the following antivirals is a reverse transcriptase 618 C. Lamivudine is used in hepatitis B as well as in HIV inhibitor that is used in the treatment of Hepatitis B infection? A. infection. Amantadine B. Ganciclovir C. Lamivudine D. Interferon alpha E. Acyclovir 572. Which of the following beta blocker drugs has/have intrinsic 629 E Pindolol and Acebutolol have intrinsic sympathomimetic sympathomimetic activity? A. Timolol B. Acebutolol C. Pindolol activity. D. Betaxolol E. B and C 573. Which of the following beta-blockers are less likely to cause 111 C. Page 16 of Topnotch Handout. The two beta blockers bronchospasm in patients with asthma due to its partial agonist with intrinsic sympathomimetic activity are Pindolol and activity? A. Carvedilol B. Labetalol C. Pindolol D. Propanolol E. Acebutolol. Nadolol 574. Which of the following beta-blockers demonstrates the relative 586 D. Because of its relatively β1 selectivity, betaxolol has selectivity in the manner described? A. Betaxolol: relatively fewer side effects. Timolol and levobunolol are non- selective selective for β2-receptors B. Timolol: relatively selective for β-blockers β1-receptors C. Levobunolol: relatively selective for β2- receptors D. Betaxolol: relatively selective for β1-receptors E. Levobunolol: relatively selective for β1-receptors 575. Which of the following Beta Blockers has an 414 B. Pindolol has ISA, which makes it a B blocker with a partial Alpha agonist Intrinsic Sympathomimetic Activity? A. action. Carvedilol B. Pindolol C. Nadolol D. Metoprolol E. Timolol 576. Which of the following beta-blockers have local 200 C. Beta-blockers with partial agonist effect on adrenergic receptors have anesthetic effect? A. Nadolol B. Penbutolol C. intrinsic sympathetic activity: LAPPCC (Labetalol, Acebutolol, Penbutolol, Pindolol D. Carteolol E. Atenolol Pindolol, Carteolol, Celiprolol). Beta-blockers with membrane stabilizing activity have local anesthetic effect: LAMPP (Labetalol, Acebutolol, Metoprolol, Pindolol, Propranolol) 577. Which of the following beta blockers possess 296 E Acebutolol and Pindolol have partial agonist activity (ISA) intrinsic sympathomimetic activity? A. Metoprolol B. Esmolol C. Pindolol D. Acebutolol E. Both C and D 578. Which of the following can't be used topically? 58 C. Topical clindamycin and erythromycin are used topically againts acne. A. Clindamycin B. Erythromycin C. Amoxicllin D. Topical fusidic acid and mupirocin are used topically againts other superficial Fusidic acid E. Mupirocin bacterial infection. 579. Which of the following cephalosporins can be 180 D. antipseudomonal cephalosporins are ceftazidime, cefepime, and used to patients with Pseudomonas infection? A. cefoperazone cefuroxime B. cefazolin C. cefoxitin D. Ceftazidime E. Ceftriaxone 580. Which of the following cephalosporins is highly 617 C. Ceftazidime, 3rd gen cephalosporin, is considered to be most effective against pseudomonas? A. Cefazolin B. efficacious agaianst pseudo monas among cephalosporins. Cefuroxime C. Ceftazidime D. Cefaclor E. Ceftriaxone 581. Which of the following Cephalosporins is 418 C. 3rd gen Cephalosporins have syndergistic effects with Aminoglycosides, synergistic with Aminoglycosides? A. Cefazolin 1st and 2nd gen cephalosporines can increase nephrotoxic effects of B. Cefaclor C. Cefixime D. Cefipime E. aminoflycosides. Loracarbef 582. which of the following changes will promote 306 C. any condition that can predisposed to hyperkalemia, hypermagnesemia digitalis induced cardiototoxicity? A. Acute and hypocalcemia will inhibit the binding of digitalis to Na K ATPase resulting tubular necrosis B. hypermagnesemia C. to decrease toxicity. Triamterene and ATN can induce hyperkalemia. Katzung hyperparathyroidism D. Triamterene 11th ed p 216. administration E. All of the above 583. Which of the following chemotherapeutic 257 A. SIMILAR TO PREVIOUS BOARD EXAM CONCEPT/PRINCIPLE, know the agents can produce pulmonary fibrosis as characteristic adverse effects and toxicities, the peculiar ones, the ones that toxicity? A. bleomycin B. doxorubicin C. make the drugs famous, pulmo fibrosis = bleomycin irinotecan D. etoposide E. carmustine 584. Which of the following chemotherapeutic drugs 401 A. SIMILAR TO PREVIOUS BOARD EXAM CONCEPT/PRINCIPLE may be used for Lentigo Maligna? A. Actinomycin B. Bleomycin C. Doxorubicin D. Vinblastine E. Cisplatin 585. Which of the following chemotherapeutic drugs 402 D. SIMILAR TO PREVIOUS BOARD EXAM CONCEPT/PRINCIPLE may be used for Lung Cancer? A. Procarbazine B. Cytarabine C. Topotecan D. Cisplatin E. Cyclophosphamide 586. Which of the following chemotherapeutic 245 B. SIMILAR TO PREVIOUS BOARD EXAM CONCEPT/PRINCIPLE - MOPP regimens could be used for the treatment of nitrogen mustard, oncovin, procarbazine, prednisone, take time to know the advanced hodgkin's disease? A. FOLFOX B. common regimens for other cancers as well, ex: folfox colorectal, CHOP MOPP C. CHOP D. 7+3 E. BEP nonhodgkins, BEP testicular, 7+3 AML 587. Which of the following Cholinomimetic drugs 264 E Donepezil, Rivastigmine, Tacrine used exclusively in Alzheimer's are used exclusively in the treatment of Alzheimer's disease? A. Donepezil B. Physostigmine C. Rivastigmine D. A and B E. A and C 588. Which of the following correctly describes FDA Drug category B? A. 176 D. A: category C------ B. Category A-------- C. Either animal studies revealed adverse effects on the fetus and Category D there are no controlled studies in women or studies in women and animals are not available B. Control studies in women fail to demonstrate a risk to the fetus in the first trimester and the possibility of fetal harm is remote C. there is positive evidence of human fetal risk but the benefits from use in pregnant women may be acceptable despite the risk D. Animal reproduction studies have shown an adverse effect that was not confirmed in controlled studies in women in the first trimester E. None of the above 589. Which of the following diuretics do not act on the luminal side of the 383 C. SIMILAR TO PREVIOUS BOARD EXAM renal tubules? A. Mannitol B. Furosemide C. Spironolactone D. CONCEPT/PRINCIPLE. Spironolactone is an aldosterone Acetazolamide E. Hydrochlorothiazide antagonist that blocks its binding to mineralocorticoid receptors in the basolateral membrane of the cortical collecting tubules. The rest of the choices act on the luminal side. Mannitol is an osmotic diuretic. Furosemide inhibits Na/K/Cl transporters in the luminal side of the loop of Henle. Acetazolamide inhibits carbonic anhydrase activity at the luminal side of the PCT. Hydrochlorothiazide inhibits Na/Cl transporters in the luminal side of the DCT. 590. Which of the following diuretics is incorrectly paired with its 546 A. Thiazides increase reabsorption of Ca and prevent associated toxicity? A. Thiazide - renal stone B. Acetazolamide - Ca stone formation. Hepatic encephalopathy can ensue hepatic encephalopathy C. Bumetanide - hypokalemic metabolic with acetazolamide because it causes alkalinization of alkalosis D. Eplerenone - hyperkalemic metabolic acidosis E. urine due to decreased bicarbonate reabsorption. This Mannitol - pulmonary edema prevents ammonia from turning to ammonium. Ammonia is more easily reabsorbed. 591. Which of the following diuretics may cause hyperkalemia? A. 126 A. Amiloride (also Triamterene, Eplerenone, Amiloride B. Ethacrynic acid C. Indapamide D. Mannitol E. Spironolactone) is a potassium sparing diuretics. All other Acetozalamide options are potassium-wasting. SIMILAR TO PREVIOUS BOARD EXAM CONCEPT. 592. Which of the following drug acts by selectively inhibitting 508 B. Katzung 10th ed., 169. Metoprolol, Esmolol, stimulation of B1 adrenoceptor and may be advantageous in treating Atenolol, Acebutolol, Betaxolol, Bisoprolol are selective hypertensive patients who also suffer from asthma, diabetes, or B1-blockers. Propranolol and Nadolol are non-selective peripheral vascular disease? A. Nadolol B. Metoprolol C. B-blockers. Labetalol and Carvedilol have combined Propranolol D. Labetalol E. Carvedilol alpha and b-blocking activity. 593. Which of the following drugs are beneficial in delaying organ 18 E SIMILAR TO PREVIOUS BOARD EXAM remodeling such as in cardiac remodeling in heart failure and CONCEPT/PRINCIPLE. Source: Topnotch Handout on reduces progression of DM nephropathy? A. Captopril B. Losartan C. Pharmacology Eplerenone D. A and B only E. All of the above 594. Which of the following drug/s can be used for eliminating 365 E Minocycline, 200 mg orally daily for 5 days, can meningococcal carriage states? A. Minocycline B. Pen G C. Rifampin eradicate the meningococcal carrier state, but because of D. Demeclocycline E. A and C side effects and resis- tance of many meningococcal strains, rifampin is preferred. Katzung 12th ed pg 812 595. Which of the following drugs can be used to reduce intraocular 624 E pressure in the treatment of glaucoma? A. Pilocarpine B. Acetazolamide C. Neostigmine D. Timolol E. All of the above 596. Which of the following drugs can cause cataract formation? A. 244 A. SIMILAR TO PREVIOUS BOARD EXAM allopurinol B. pyrazinamide C. propanolol D. omeprazole E. CONCEPT/PRINCIPLE, one of the hardes pharma ciprofloxacin questions we encountered, we did not know at that time that allopurinol was associated with cataracts :) 597. Which of the following drugs can cause prolonged QT interval 226 C. antiarrhytmic drugs that affects the K channel at leading to Torsade de Pointes arrhytmia? A. Esmolol B. phase 3 causes prolongation of Action Potential leading Propafenone C. Procainamide D. Verapamil E. None of the above prolonged QT interval. These drugs are of the Class IA and III. Class IA drugs - procainamide, quinidine, disopyramide. Class III- Ibutilide, sotalol, dofetilide 598. Which of the following drugs/drug regimen is primarily used for 240 A. B- testicular cancer, C- pancreatic cancer, D- Hodgkin's Lymphoma A. ABVD- adriamycin, bleomycin, vincristine, choriocarcinoma. Aug 2014 SIMILAR TO PREVIOUS dacarbazine, prednisone B. PEB - cisplatin, etoposide, bleomycin C. BOARD EXAM CONCEPT/PRINCIPLE. Gemcitabine + 5 FU + radiation D. EMACO- etoposide, methotrexate, actinomycin D, cyclophosphamide, vincristine E. all of the above 599. which of the following drugs exhibit zero order kinetics ? A. Ethanol 302 E zero order kinetics refers to a contant rate of B. tolbutamide C. warfarin D. heparin E. All of the above elimination of a drug regardless of its concentration. The following drugs exhibit zero order kinetics: aspirin, ethanol, warfarin,heparin, penytoin, tolbutamide, theophyline phenylbutazone, and salicylates. 600. Which of the following drugs for glaucoma increased aqueous 585 B. Beta blockers, CAIs and osmotic agents (mannitol) humor outflow through the non-conventional pathway without decreases aqueous humor production. Alpha adrenergic decreaing the secretion of aqueous humor by the ciliary body? A. agonist increases outflow and also decreases the secretion β-blockers B. Prostaglandin analogues C. α-adrenergic agonist D. of the ciliary body. Prostaglandins increased outflow but Carbonic anhydrase inhibitors E. Mannitol has no effect on production. 601. Which of the following drugs given contains an effect which 250 D. SIMILAR TO PREVIOUS BOARD EXAM suppresses nausea and vomiting ? A. propanolol B. procarbazine C. CONCEPT/PRINCIPLE, difficult because chlorpromazine chlorambucil D. chlorpromazine E. chloramphenicol usually is used as a antipsychotic but has antiemetic effects belonging to the phenothiazine class along with other drugs used to control nausea vomiting like promethazine 602. Which of the following drugs is a direct acting cholinoninetic used 665 B. Bethanecol for bowel and bladder atony for treatment of glaucoma, sjoqren and sicca syndrome? A. Betanechol B. Pilocarpine C. Physostigmine D. Neostigmine E. None of the above 603. Which of the following drugs is an effective prophylactic drug 403 E SIMILAR TO PREVIOUS BOARD EXAM against migraine headache attacks? A. Ondansetron B. Sumatriptan CONCEPT/PRINCIPLE C. Ergotamine D. Methylergonovine E. Propranolol 604. Which of the following drug/s is/are notorius in causing gingival 633 E hyperplasia? A. Nifedipine B. Cyclosporine C. Phenytoin D. Verapamil E. All of the above 605. Which of the following drugs is classified as Category X during 381 B. The rest of the choices are category D drugs. pregnancy? A. Valproate B. Statins C. Aspirin D. Captopril E. Phenytoin 606. which of the following drugs is highly selective to ischemic or 307 A. phenytoin, lidocaine and mexiletine are classified as prolonged depolarized purkinje fibers? A. Mexiletine B. class IB antiarrhythmics. This group affects primarily Procainamide C. Propafenone D. ibutilide E. None of the above ischemic or depolarized ventricular tissue and does not have any use in atrial arrhytmia. B- procainamide is a class IA drug, C- propafenone is a Class IB agent, D- ibutilide is class III drugs. 607. Which of the following drugs is used to induce ovulation in 297 B. SIMILAR TO PREVIOUS BOARD EXAM anovulatory women by selectively blocking estrogen receptors in CONCEPT/PRINCIPLE the pituitary, thus increasing FSH and LH output? A. Raloxifene B. Clomiphene C. Fulvestrant D. Anastrozole E. Progestins 608. Which of the following drugs lower blood 505 E Metyrosine inhibits tyrosine hydroxylase which is responsible for the pressure by preventing normal physiologic conversion of Tyrosine to Dopa, the rate-limiting step in catecholamine release of norepinephrine from postganglionic synthesis. Cocaine blocks re-uptake of NE from the synaptic cleft thereby sympathetic neurons? A. Metyrosine B. Cocaine prolonging its effect. Reserpine blocks dopamine transport into the vesicle. C. Reserpine D. Botulinum toxin E. Guanethidine Botulinum blocks Acetylcholine release from the nerve terminal into the junctional cleft. Study figures 6-3 and 6-4 of Katzung 10th ed, p. 79 and 81. 609. Which of the following drugs of abuse is non 288 D. Some drugs of abuse do not lead to addiction. This is the case for addictive? A. Amphetamine B. Cocaine C. substances that alter perception without causing sensation of reward and Morphine D. LSD E. Heroin euphoria. These include LSD, Phencyclidine and ketamine 610. Which of the following drugs that used in gout 267 B. SIMILAR TO PREVIOUS BOARD EXAM CONCEPT/PRINCIPLE has been associated with cataract formation? A. Colchicine B. Allopurinol C. Probenecid D. Indomethacin E. Acetaminophen 611. Which of the following drugs used in the 386 B. treatment of peptic ulcer disease works by forming a protective coating over ulcer beds and has known adverse effects of melena and darkening of the tongue? A. Sucralfate B. Bismuth salicylate C. Psyllium D. Senna E. Kaolin 612. Which of the following drugs with mechanism of 260 A. SIMILAR TO PREVIOUS BOARD EXAM CONCEPT/PRINCIPLE, please action that inhibits protein synthesis by know the mechanism of action of each drug class inhibiting binding of the tRNA to the mRNA ribosome complex A. tetracycline B. clindamycin C. gentamycin D. erythromycin E. chloramphenicol 613. which of the following effects in parathion 304 C. pralidoxime is a chemical antagonist acting as cholinesterase poisoning will not be antagonized by timely regenerator. It cannot enter the BBB because of its positively charged group administration of pralidoxime? A. cyclospasm B. and poor lipid solubility. Katzung review pp 69. Urinary incontinence C. convulsion D. Negative dromotrophy E. None of the above 614. which of the following effects of acetazolamide 308 B. the major effect of acetazolamide is bicarbonate diuresis resulting to is self limiting? A. Sodium reabsorption B. metabolic acidosis. As increased sodium is presented to the cortical collecting Tubular bicarbonate excretion C. CSF acidosis tubule, some of the excess sodium is reabsorbed and potassium is secreted D. Aqueous humor production E. None of the resulting to significant potassium wasting. as a result of bicarbonate depletion, above sodium bicarbonate excretion slows even with continued administration. Katzung review 8th ed p 135 615. Which of the following ergot alkaloids is known 639 A. to cause retroperitoneal fibrosis? A. Methylsergide B. Dihydroergotamine C. Methylergonovine D. Sumatriptan E. None of the above 616. Which of the following has the lowest incidence 635 A. Clozapine is a new drug with least extrapyramidal effect but is known to of extrapyramidal symptoms but is known to cause agranulocytosis. cause agranulocytosis? A. Clozapine B. Fluphanazine C. Chlorpromazine D. Pimozide E. Haloperidol 617. Which of the following has the most potent 7 D. Contraindication to thrombolysis - History of CVS hemorrhage at ANY mineralocorticosteroid activity? A. time - History of CVS infarct within the past year - Marked hypertension Hydrocortisone B. Triamcinolone C. (>180/110) - Suspicion of aortic dissection - Active internal bleeding (excluding Betamethasone D. Fludrocortisone E. menstruation) Source: Topnotch handout on Pharmacology Dexamethasone 618. Which of the following have been proven to 374 E Topnotch cause pulmonary fibrosis? A. Methothrexate B. Bleomycin C. Busulfan D. Amiodarone E. All of the above 619. Which of the following inhibit/s cortisol 378 D. Aminoglutethimide blocks the conversion of cholesterol to pregnenolone synthesis? A. Aminoglutethimide B. and causes a reduction in the synthesis of all hormonally active steroids. Ketoconazole C. Mifepristone D. A and B E. Ketoconazole is a potent nonselective inhibitor of adrenal and gonadal steroid All of the above synthesis through inhibiton of P450 enzymes. Mifepristone is a glucocorticoid receptor antagonist with strong antiprogesterone activity. Katzung 12th ed pg 709- 711 620. Which of the following insulin preparations 595 E The long acting insulin mimics the normal secretion of the pancreas in the mimics the secretion pattern of insulin by the non-fed state. normal pancreas in the fasting state? A. Lispro B. Regular C. NPH D. Glulisine E. Detemir 621. Which of the following is a balanced 382 B. Balanced vasodilators include ACEIs, ARBs, alpha- adrenergic blockers, vasodilator--meaning, both an arterial and a alpha-central agonists, nitroprusside and nesiritide. venous vasodilator--used to treat both congestive heart failure and coronary artery vasospasm? A. Verapamil B. Captopril C. Nifedipine D. Propranolol E. Hydralazine 622. Which of the following is a contraindication 355 B. Reference: Katzung. Basic and Clinical Pharmacology, 11th ed. p. 743 to the use of thiazolidinediones? A. Alcohol ingestion B. Heart failure C. Cholelithiasis D. Renal insufficiency 623. Which of the following is a drug of choice for 236 E AIDS patient with a CD4 count of less than 50/UL are prone to have daily dose prophylaxis in AIDS patient with a Mycobacterium Avum Intracellulare infection. The only approved prophylactic CD4 count of 50/UL? A. Cotrimoxazole B. treatment are daily dose of clarithromycin with or without rifabutin and once Kanamycin C. Azithromycin D. Erythromycin weekly dose of azithromycin. E. Clarithromycin 624. Which of the following is a good choice to 33 C. Buspirone is a partial serotonin 5-HT1A-receptor agonist that has efficacy treat newly diagnosed generalized anxiety comparable to that of benzodiazepines for the treatment of anxiety, but is disorder in a truck driver? A. Alprazolam B. significantly less sedating. Alprazolam is an intermediate-acting benzodiazepine Triazolam C. Buspirone D. Trazodone E. used in the treatment of generalized anxiety disorder (GAD) but still has some Thiopental sedation, which would be undesirable in this situation. Triazolam is a short-acting benzodiazepine, and trazodone is a heterocyclic antidepressant, both used to induce sleep. Thiopental is a barbiturate sometimes used to induce anesthesia. 625. Which of the following is a major adverse 294 B. SIMILAR TO PREVIOUS BOARD EXAM CONCEPT/PRINCIPLE. A major effect associated with use of fenofibrates? A. toxicity of fenofibrates is increased risk of gallstone which may be due to Constipation B. Gallstones C. Hyperuricemia enhanced biliary excretion D. Cardiac arrythmia E. Liver damage 626. Which of the following is a major toxicity 590 B. Gemfibrozil is associated with cholelithiasis. Myopathy is only expected in associalted with gemfibrozil therapy? A. combination with statins. Bloating and constipation B. Cholelithiasis C. Hyperuricemia D. Myopathy E. Severe cardiac arryhtmia 627. Which of the following is a mechanism of 123 A. Hydralazine binds to and activates potassium channels on vascular smooth action of Hydralazine? A. Alters intracellular muscle resulting to efflux of potassium and subsequent hyperpolarization of the calcium B. Opens potassium channel causing cell. This prevents calcium- mediated activation and constriction of smooth arteriolar and venular dilatation C. Blocks muscle, resulting in vasodilation. It dilates arterioles, but not veins. SIMILAR TO alpha-1 adrenergic receptor D. PREVIOUS BOARD EXAM CONCEPT. Competitively bloocks Nn nicotinic receptor E. Blocks voltage-gated L-type calcium channels 628. Which of the following is an extraluminal 135 A. Tissue amebicides act in the bowel wall and liver: Metronidazole, emetine, amoebicide? A. Metronidazole B. Diloxanide chloroquine, tinidazole; Luminal amebicide: Diloxanide furoate, Iodoquinol, furoate C. Lumefrantrine D. Pentamidine E. Paromomycin. SIMILAR TO PREVIOUS BOARD EXAM CONCEPT Nifurtimox 629. Which of the following is an inhalational anesthetic of 249 A. SIMILAR TO PREVIOUS BOARD EXAM CONCEPT/PRINCIPLE, choice for asthmatic patient because it causes the least halothane generally is the inhaled agent of choice, sevoflurane is bronchospasm? A. halothane B. desflurane C. debatable, desflurane is pungent, the others are IV anesthetics sevoflurane D. thiopental E. propofol 630. Which of the following is an orally active direct 286 E Apixaban and Rivaroxaban are both oral Factor Xa inhibitors. thrombin inhibitor? A. Apixaban B. Rivaroxaban C. Enoxaparin and Fondaparinux are both SC factor Xa inhibior. Enoxaparin D. Fondaparinux E. Dabigatran 631. Which of the following is a potential side effect of 32 C. Agranulocytosis occurs more frequently with clozapine than with clozapine? A. Cholestatic jaundice B. QT prolongation other agents, requiring routine blood tests. It is the only agent that C. Agranulocytosis D. Photosensitivity E. Galactorrhea improves the negative symptoms of schizophrenia. Cholestatic jaundice and photosensitivity are common with chlorpromazine. Galactorrhea is a side effect of older high-potency agents that block dopamine. QT prolongation is a complication of agents such as thioridazine and ziprasidone. 632. Which of the following is a reversible inhibitor of 406 C. Allopurinol is an IRREVERSIBLE Xanthine Oxidase inhibitor, Xanthine Oxidase used in Chronic Gout? A. Allopurinol Febuxostat is a REVERSIBLE Xanthine Oxidase inhibitor, B. Colchicine C. Febuxostat D. Mycophenolate Mofetil E. Probenecid 633. Which of the following is considered as a reliever of 429 D. Terbutaline is a beta-2 receptor agonist used as a fast-acting asthma rather than a controller? A. Montelukast B. bronchodilator for asthma and is also used in preventing premature Cromolyn C. Omalizumab D. Terbutaline labor (tocolysis). 634. Which of the following is correct regarding isosorbide 224 C. ISMN is the oral form of nitroglycerin. Its mechanism is by mononitrate? A. Can be use along with PDE inhibitors activating guanylyl cyclase causing an increase in cGMP eventually compared to other forms of nitroglycerin B. It has a leading dephosphorylation of MLC. Its antihypertensive effect is similar mechanism of action with Nitroprusside by primarily via venodilation. It has no direct inotropic or chronotropic phosphorylation of myosin light chain phosphate C. It effect to the heart. all forms of nitroglycerin should not concomitantly has no direct effect on cardiac muscle D. the primary use with anyform of PDEI ( sildenafil) since it will cause profound mechanism for decrease in BP is due to decrease in hypotension and hypoperfusion of critical organs afterload E. B and C are correct 635. Which of the following is incorrectly matched? A. 9. Bonus. No correct answ.Menotropin â€" FSH and LH analog Menotropin â€" FSH and LH analog B. Ocreotide â€" Ocreotide â€" Somatostatin analog Desmopressin â€" ADH analog Somatostatin analog C. Desmopressin â€" ADH analog Ganirelix â€" GnRH antagonist Levothyroxine â€" T4 Source: Topnotch D. Ganirelix â€" GnRH analog E. Levothyroxine â€" T4 handout on Pharmacology 636. Which of the following is not a 3rd generation 13 E 1st gen â€" Cefazolin, Cephalotin, Cephalexin, Cephradine, cephalosporin? A. Ceftazidime B. Ceftriaxone C. Cephapirin 2nd gen â€" Cefaclor, Cefuroxime, Cefoxitin, Cefotetan, Ceftizoxime D. Cefixime E. Cefepime Cefamandole, Cefonicid, Cefprozil, Cefometazole 3rd gen â€" Ceftriaxone, Cefoperazone, Cefotaxime, Cefixime, Ceftazidime, Ceftizoxime, Cefpodoxime proxetil, Cefdinir, Cefditoren, Ceftibuten 4th gen â€" Cefepime Source: Katzung Basic and Clinical Pharmacology 11th ed 637. Which of the following is NOT an effect of Naloxone? 130 A. Naloxone is an opioid antagonist used for opioid overdose. It A. It will cause respiratory depression B. It may cause competitively blocks opioid receptors and rapidly reverses effects of nausea and vomiting C. It induces abstinence syndrome opioid agonists. In individuals who are acutely depressed by an D. It decreases constipation E. None of the above overdose of an opioid, the antagonist effectively normalizes respiration, LOC, pupil size, bowel activity, and awareness of pain. In dependent subjects who appear normal while taking opioids, naloxone or naltrexone almost instantaneously precipitates an abstinence syndrome. SIMILAR TO PREVIOUS BOARD EXAM CONCEPT 638. Which of the following is not an indication for SSRIs? A. 5 E SIMILAR TO PREVIOUS BOARD EXAM CONCEPT/PRINCIPLE. SSRIs Major depressive disorder B. OCD C. Anxiety and panic are the first line drugs for MDD. OCD â€" SSRI! Source: Topnotch attacks D. Phobias E. None of the above handout on Pharmacology 639. Which of the following is not associated with 11 E All of the following are associated with pulmonary fibrosis. Source: pulmonary fibrosis? A. Bleomycin B. Busulfan C. Topnotch handout on Pharmacology Amiodarone D. Methotrexate E. None of the above 640. Which of the following is not considered a controller drug 20 B. Terbutaline is a bronchodilator used for acute relief of for asthma? A. Budesonide B. Terbutaline C. Montelukast D. asthma exacerbations. Source: Topnotch handout on Nedocromil E. Omalizumab Pharmacology 641. Which of the following is NOT true about insulin action? A. 619 D. The insulin receptor is a transmembrane receptor that is It stimulates glycogen synthesis in muscle fiber B. It inhibits activated by insulin, IGF-I, IGF-II and belongs to the large class of lipolysis in the adipocyte C. It stimulates fatty acid synthesis tyrosine kinase receptors. in the hepatocytes D. It binds to G receptor to increase intracelluar cAMP 642. Which of the following is not true in the pharmacologic 235 E Aminoglycoside exerts concentration dependent killing and profile of Aminoglycosides? A. The primary mechanism of post antibiotic effect. It also has curare like action at high dose resistance is due to plasmid mediated group transferases B. leading to respiratory paralysis. Aminoglycosides needs Oxygen Bactericidal action continuous even if the plasma drug dependent transport to exerts inhibitory effect 30s ribosome. concentration is below measuring levels C. It does not have therefore, it has no activity to anaerobes. acetylation by plasmid an activity against anaerobic organisms D. high dose can mediated group transferase is the major mode of resistance. cause respiratory paralysis E. none of the above 643. Which of the following is not true regarding Noncompetitive 8 A. Competitive antagonism: ED50 increases, potency decreases, antagonism? A. No effect on Emax B. No effect on ED50 C. No effect of maximum efficacy Noncompetitive antagonism: No No effect on potency D. A and B E. All of the above effect ED50, No effect on potency, decreased Emax Source: Topnotch handout on Pharmacology 644. Which of the following is not used for the treatment of 395 A. Chlordiazepoxide is used for the treatment of alcohol alcohol dependence? A. Chlordiazepoxide B. Disulfiram C. withdrawal, not dependence. Diazepam may also be used for Naltrexone D. Acamprosate E. None of the above alcohol withdrawal. 645. Which of the following is the drug of choice for partial 592 C. Carbamazepine, phenytoin and lamotrigine are drug of seizures? A. Valproic acid B. Phenobarbital C. Lamotrigine choices for partial seizures D. Ethosuximide E. Topiramate 646. Which of the following is the drug of choice for tic 385 A. Carbamazepine is also the drug of choice for trigeminal disorders? A. Carbamazepine B. Lamotrigine C. Valproic neuralgia. acid D. Topiramate E. Phenytoin 647. Which of the following is true? A. Azithromycin has lesser 41 E Choramphenicol is one of the few antibiotics with higher oral bioavailability when taken with food B. Metronidazole has bioavailability than parenteral form. the same oral and parenteral bioavailability. C. The oral bioavailabity of chloramphenicol is higher than parenteral form. D. Both B and C E. AOTA 648. Which of the following is true about Buspirone? A. 311 C. Buspirone is a selective anxiolytic medication that does not Anxiolytic effect is secondary to direct stimulation of cause, sedative, hypnotic, euphoric, and acute withdrawal or GABAergic neurons B. It is highly effect in acute panic rebound effect. It has no effect in psychomotor skills (driving attacks C. Less psychomotor retardation D. Abrupt skills). The mechanism of anxiolysis is secondary to partial discontinuation can lead acute withdrawal syndrome E. all antagonism to 5HT1A receptors in the brain. it has no direct of the above GABAergic activity. weeks are needed before buspirone takes effect, therefore, it is not effect in treating acute panic attacks. Katzung 11ed p374. 649. Which of the following is uricosuric? A. Allopurinol B. 331 C. Allopurinol is a xanthine oxidase inhibitor. Colchicine inhibits Colchicine C. Sulfinpyrazone D. Celecoxib microtubule assembly. Celecoxib is a COX-2 inhibitor. 650. Which of the following IV anesthetics is the only one that 554 D. causes CV stimulation? Its use in neurosurgical operations is limited due to its propensity to increase ICP. A. Midazolam B. Propofol C. Fentanyl D. Ketamine E. Thiopental 651. Which of the following loop diuretics is not prone to induce 589 E Ethacrynic acid does not contain a sulfa group. an allergic response? A. Furosemide B. Bumetanide C. Brinzolamide D. Torsemide E. Ethacrynic acid 652. Which of the following mechanism indicates high-level 270 E E is INH high level resistance; A is INH low level resistance in a TB patient being treated with Isoniazid? A. resistance; B is rifampicin resistance deletions in inhA gene that encodes target enzyme B. changes in drug sensitivity to RNA polymerase C. expression of drug efflux system D. expression of inactivating enzymes E. deletion in katG gene involved in the bioactivation of the drug 653. Which of the following medication-rescue therapy pair is 640 E INCORRECT? A. Methotrexate : Leucovorin B. Cyclophosphamide : MESNA C. Doxorubicin : Dexrazoxane D. Cisplatin : Amifostine E. None of the above 654. Which of the following medications can be given once a week 290 A. because of its long elimination half life (3-4 days), for prophylaxis against bactermia caused by M. avium complex weekly administration of azithromycin has proved to be (MAC) in AIDS patients? A. Azithromycin B. Kanamycin C. equivalent to daily administration of clarithromycin when used Ethambutol D. Rifampicin E. Cycloserine for prophylaxis against MAC in AIDS 655. Which of the following medications can be used as part of a 254 D. SIMILAR TO PREVIOUS BOARD EXAM regimen for migraine prophylaxis? A. ibuprofen B. CONCEPT/PRINCIPLE, prophylaxis: pizotifen, TCA, acetaminophen C. sumatriptan D. propranolol E. anticonvulsants and beta blockers, methysergide, flunarizine dihydroergotamine 656. Which of the following medications can decrease conduction 255 B. SIMILAR TO PREVIOUS BOARD EXAM through the AV node? A. nifedipine B. verapamil C. lithium D. CONCEPT/PRINCIPLE, verapamil is a non dihydropyridine prazosin E. dypyridamole calcium channel blocker which exerts its actions more on the heart decreasing AV nodal conduction 657. Which of the following medications causes mydriasis but 246 D. SIMILAR TO PREVIOUS BOARD EXAM without cycloplegia? A. tropicamide B. homatropine C. CONCEPT/PRINCIPLE also a difficult one, only phenylephrine cylopentolate D. phenylephrine E. atropine among the choices causes mydriasis without affecting accomodation 658. Which of the following medications could be given to high risk 251 E SIMILAR TO PREVIOUS BOARD EXAM or immunocompromised patients with RSV infections? A. CONCEPT/PRINCIPLE, acyclovir and ganciclovir for herpes Acyclovir B. Fosamprenavir C. Indinavir D. Ganciclovir E. group, indinavir and fosamprenavir for hiv Ribavirin 659. Which of the following medications exert its anticoagulant 278 B. Fondaparinux and LMW heparin combines with effect through the inactivation of thrombin? A. Fondaparinux B. antithrombin and selectively inhibits factor X but NOT thrombin. Unfractionated heparin C. Low molecular weight heparin D. Apixaban is a direct Xa inhibitor Apixaban E. All of the above 660. Which of the following medications is considered as an 243 D. SIMILAR TO PREVIOUS BOARD EXAM inhibitor of glucocorticoid synthesis which could be used in the CONCEPT/PRINCIPLE, I couldn’t believe this was actually treatment of cushing's syndrome and some types of cancers? asked, aminoglutethimide inhibits desmolase blocking A. Methylprednisolone B. anastrozole C. Danazol D. conversion of cholesterol to pregnenolone reduces synthesis Aminogluthetimide E. Flutamide of all hormonally active steroids, anastrozole inhibits only estrogen synthesis, danazol is an androgen, flutamide is androgen antagonist 661. Which of the following nonselective , irreversible alpha 630 B. DOC for pheochromocytoma is Phenoxybenzamine, it is blocker is the drug of choice as a preoperative agent used in a nonselective, IRREVERSIBLE alpha blocker.. Phentolamine is a Pheochromocytoma? A. Phentolamine B. Phenoxybenzamine C. reversible alpha blocker. Prazosin D. Yohimbine E. Clonidine 662. Which of the following ophthalmic agents decreases 400 B. Bimatoprost is a prostaglandin analogue that increase intraocular pressure by increasing outflow of aqueous humor outflow via the uveoscleral pathway. Apraclonidine is a alpha-2 through the uveoscleral or unconventional pathway? A. receptor agonist that decreases aqueous humor production. Homatropine B. Bimatoprost C. Apraclonidine D. Mannitol E. Mannitol works via osmosis. Timolol is a beta-receptor Timolol antagonist that also decreases aqueous humor production. Homatropine has no effect on IOP. 663. Which of the following regarding anticholesterol drugs is/are TRUE? A. 370 D. Fibrates stimulate PPAR alpha, most effective Simvastatin, a reversible inhibitor of HMG-CoA reductase, is most for decreasing elevated triglyceride levels. Katzung effective in reducing LDL levels B. Niacin stimulates PPAR-α causing an 12th ed pg 626, 628, 631 increase in HDL and a decrese in triglycerides C. There is increased risk of gallstone formation when fibrates are combined with bile acid binding resins D. A and C E. All of the above 664. Which of the following statements about the mechanism of action of 636 B. ipratropium? A. It acts cebtrally to decrease vagal Ach release B. It inhibits pulmonary Ach receptors C. It decreases mast cell release of histamine D. It blocks the action of histamine at H1 receptors E. None of the above 665. Which of the following statements concerning barbiturates is true? A. 634 B. Barbiturates can increase bleeding time when administered to patients taking anticoagulants B. Barbiturates are contraindicated in patients with acute intermittent porphyria C. Patients tolerant to the therapeutic actions of barbiturates are also tolerant to analgesic effect of morphine D. Barbiturates are used to prevent withdrawal symptoms associated with heroin dependence E. All of the above 666. Which of the following statements is/are true regarding the binding 105 E Page 4 of Topnotch Handout. Binding affinity affinity of a drug: A. Kd is the concentration required to bind 100% of the refers to the fraction of receptors bound by a drug drug receptors B. The smaller the Kd the lesser the affinity of the drug plotted against the log of the drug concentration. Kd for its receptor C. A smaller Kd means that the drug is less potent D. is the concentration required to bind 50% of the Binding affinity refers to the fraction of receptors bound by a drug receptors. The smaller the Kd, the greater the affinity plotted against the log of the drug effect E. None of the above of a drug for its receptor, the more potent it is. 667. Which of the following statements is true for cromolyn: A. It reduces 117 B. SIMILAR TO PREVIOUS BOARD EXAM synthesis of leukotrienes B. It has no bronchodilator action C. It binds IgE CONCEPT/PRINCIPLE. Page 33 of Topnotch antibodies on mast cells D. It reduces the expression of cyclooxygenase Handout. Cromolyn is a mast cell stabilizer by the E. It prevents vagal-stimulated bronchoconstriction prevention of calcium influx, therefore preventing degranulation and release of histamine, leukotrienes and other mediators. A is moa of zileuton, C is for omalizumab, D is for corticosteroids while E is for Ipratropium 9muscarinic receptor antagonist. 668. Which of the following statements regarding diuretics is not true? A. 507 D. The collecting tubule and not the distal tubule Diuretics lower blood pressure primarily by depleting body sodium is the final site NaCl reabsorption. It is responsible for stores. B. The most common adverse effect of thiazide diuretics is tight regulation of body fluid volume and for potassium depletion. C. The intercalated cells in the collecting tubule are determining the final sodium concentration of the +the primary sites of H secretion. D. The distal convoluted tubule is the urine. Katzung 10th ed., 239. final site of NaCl reabsorption and is responsible for the final sodium concentration of the urine. E. The thick ascending limb of the loop of Henle is the "diluting segment". 669. Which of the following statements regarding the pharmacokinetics of 35 B. The metabolism of theophylline depends on theophylline is correct? A. It is primarily metabolized by the kidney B. Its age; the half-life of the drug in children is much metabolism depends on age C. It is poorly absorbed after oral shorter than in adults. The methylxanthines are all administration D. It has a wide therapeutic index E. It stimulates well absorbed and are metabolized in the liver. The phosphodiesterase PHARMACODYNAMIC porperties of theophylline include its adenosine-receptor antagonist activity and the inhibition of phosphodiesterase. 670. Which of the following steroid antagonists inhibits cytochrome P450 272 A. Ketoconazole is a potent cytochrome p450 enxymes necessary for all steroids? A. Ketoconazole B. inhibitor. Aminoglutethimide inhibits conversion of Aminogluthetimide C. Metyrapone D. Spinorolactone E. Mifepristone cholesterol to pregnenolone. Metyrapone inhibits cortisol synthesis. Spinorolactone inhibits aldosterone. Mifepristone inhibits progesterone 671. Which of the following substances 600 A. Remember that activated charcoal is effective only to organic substances, so if it is can be adsorbed by activated inorganic like ions or acids/bases, it is not effective. Alcohol, though organic does not charcoal, effectively enhance its elimination with activated charcoal. decontaminating the gastrointestinal tract? A. Barbiturates B. HCl C. Lithium D. Cyanide E. Lye 672. Which of the following substances 584 C. Cocaine and TCA prevents reuptake of norepinephrine. Review the inhibitors for will increase the available each step in the process of neurotransmitter release for both the sympathetic and adrenergic neurotransmitter in the parasympathetic system. synaptic cleft by inibiting reuptake? A. Botulinum B. MAOIs C. Cocaine D. Guanethidine E. Reserpine 673. Which of the following 138 B. Because of its short half-life, Tolbutamide is the safest sulfonylurea for elderly sulfonylurea is safest drug for diabetics. SIMILAR TO PREVIOUS BOARD EXAM CONCEPT. elderly diabetics? A. Chlorpropramide B. Tolbutamide C. Tolazamide D. Glyburide E. Glimepiride 674. Which of the following teratogenic 107 B. Page 9 of Topnotch Handout. Sulfonamides:kernicterus; Misoprostol: Mobius drug and effect is correctly sequence; Phenytoin: Fetal hydantoin syndrome; Lithium: Ebstein's anomaly. paired? A. Sulfonamides: ototoxicity B. Penicillamine: cutis laxa C. Misoprostol: Ebstein's anomaly D. Phenytoin: Mobius sequence E. Lithium: kernicterus 675. Which of the following teratogenic 262 B.A is carbamazepine; C is Lithium, D is phenytoin effects is associated with valproic acid use in pregnancy? A. Craniofacial anomalies B. Neural tube defects C. Ebstein anomaly D. Fetal hydantoin syndome E. Microcephaly 676. Which of the following traditional 394 A. Promethazine is histamine H1-antagonist. Haloperidol and chlorpromazine are typical antipsychotic agents has now antipsychotics mainly used for psychotic disorders, not so much as antiemesis. been more widely used for Metoclopramide is a D2- antagonist used as GI prokinetic agent. antiemetic treatment of nausea and vertigo? A. Prochlorperazine B. Promethazine C. Haloperidol D. Chlorpromazine E. Metoclopramide 677. Which of the following 14 B. The mechanism of action of diazoxide is by opening K channels causing vasodilators can worsen Diabetes hyperpolarization and relaxation of vascular smooth muscles. However, recall the Mellitus? A. Hydralazine B. mechanism of release of insulin in the pancreatic beta cells. Glucose enters cells resulting to Diazoxide C. Verapamil D. increased ATP production. This closes ATP dependent K channels resulting to Nifedipine E. Nitroprusside depolarization, entry of Calcium and release of insulin. Diazoxide causes opening of K channels resulting to hyperpolarization and inhibition of insulin release. Diazoxide is actually used in treatment of insulinomas. Source: Katzung Basic and Clinical Pharmacology 11th ed p 180 678. Which of the following vasodilators has also been used for the treatment of male pattern 384 D. baldness due to its side effect of inducing hair growth A. Verapamil B. Fenoldopam C. Nitroprusside D. Minoxidil E. Hydralazine 679. Which of the following vasodilators has the common side effect of headache, hypotension and 632 B. Nitroprusside is a CYANIDE toxicity? A. Fenoldopam B. Nitroprusside C. Nifedipine D. Dopamine E. Minoxidil parenteral vasodilator known to cause CYANIDE toxicity. 680. Which of the following will increase the arrythmogenic side effect of digoxin? A. Decrease 223 D. hypokalemia, sympathetic discharge B. Increase serum Potassium C. Decrease serum Calcium D. Increase hypomagnesemia, Parasympathetic discharge E. None of the above hypercalcemia, and increase vagal tone increases the risk for digoxin induced arrhytmia 681. Which of the inhaled anesthetics forms a toxic metabolite, Compound A, that is nephrotoxic at 77 D. Well known side effects: high doses? A. Halothane B. Isoflurane C. Enflurane D. Sevoflurane E. Desflurane Halothane-- Hepatitis. Isoflurane- Coronary steal syndrome. Enflurane- Seizures. Sevoflurane- Renal insufficiency. Desflurane- Pulmonary irritant 682. Which of these anesthetics is known to cause acute porphyria? A. Nitrous oxide B. Halothane 591 D. Barbiturates such as C. Ketamine D. Thiopental E. Propofol thiopental can cause acute porphyria. 683. Which of these drugs will be the fastest to be eliminated from the body? A. 100mg drug with 187 C. zero order kinetics; elimination rate is 25mg/2hrs B. 100mg drug with first order kinetics; elimination t 1/2 is 2 hours C. 150mg drug with zero order kinetics; elimination rate is 25mg/hr D. 150mg drug with first order kinetics; elimination t 1/2 is 2 hours E. All of these will be eliminated at the same time 684. Which properties of digoxin make it an effective treatment in cases of heart failure? A. 216 B. Digoxin also slows AV Positive inotrope, positive chronotrope B. Positive inotrope, negative chronotrope C. Negative conduction (negative inotrope, negative chronotrope D. Negative inotrope, positive chronotrope E. None of the dromotrope) above 685. Which statement is correct regarding the aminoglycosides? A. They work in anaerobic 208 B. Aminoglycosides only conditions. B. They exhibit concentration-dependent killing. C. Most drugs of this class are work in arebic conditions. They myelosuppresive. D. They cannot be given to patients allergic to penicillins. E. They are acidic do not have cross-reactivity in nature. with the penicillins. Most drugs are nephrotoxic and ototoxic. 686. Which tetracycline has the broadest spectrum of microbial coverage? A. Tetracyline B. 325 D. Doxycyline C. Minocycline D. Tigecycline 687. Why is nitroglycerin preferred given sublingually? A. Giving it orally will require a higher 147 D. Sublingual route is dose B. Giving it orally will produce a greater effect C. It D. Protect it from first-pass preferred as there is less dose metabolism E. None of the above needed and it is protected from hepatic first-pass metabolism 688. Xenobiotic metabolizing enzymes have historically been grouped into the phase 1 reactions 565 D. Phase 1 reaction does and the phase 2 reactions. Which is NOT TRUE regarding the former? a. enzymes carry out little to the water solubility of a oxidation, reduction, or hydrolytic reactions b. enzymes lead to the introduction of what are drug metabolized called functional groups resulting in a modification of the drug c. the reaction process adds an -OH, -COOH, -SH, -O- or NH2 group. d. the addition of functional groups significantly increase the water solubility of the drug 689. You are assigned to go on duty as an intern of the toxicology department of the hospital. You 263 A. Iron, lithium, cyanide, recall that activated charcoal will NOT be effective adsorbing which of the following? A. Iron alcohol are poorly adsorbed B. Theophylline C. Phenobarbital D. Amitryptiline E. Digoxin by activated charcoal 690. You are treating a patient HIV-patient for an opportunistic viral infection 268 C. Cidofovir is phosphorylated by host kinases affecting the eyes. If you suspect viral strains that are thymidine-kinase while Foscarnet doesn’t reqauire deficient, you decide to choose an antiviral drug that is exclusively phosphorylation. Both are used in thymidine- phosphorylated by host cell kinases but which doesn't require viral kinase deficient resistant strains of CMV phosphorylation. Which of the following will you choose? A. Acyclovir B. Ganciclovir C. Cidofovir D. Foscarnet E. Lamivudine 691. You are treating a patient with hyperthyroidism and you prescribed 141 B. Thioamides like methimazole and PTU inhibit methimazole, you know that this drug inhibits which enzyme? A. Sodium- the organification of iodine to tyrosine via the Iodide co-transporter B. Thyroid peroxidase C. Proteases in the thyroid enzyme thyroid peroxidase. gland D. All of the above E. None of the above 692. You encountered a 2 year old patient suffering from an acute asthmatic 657 C. Salbutamol is a short-acting beta agonist attack. You immediately nebulized him with Salbutamol. Knowing its mode and may stimulate beta 1 receptors of the heart, of action, you would expect that the patient may have A. Seizures B. Dry leading to transient tachycardia. mouth C. Tachycardia D. Somnolence E. Excessive salivation 693. A young boy with G6PD deficiency would note tea-colored urine after the 493 D. intake of which drug/s: A. primaquine B. cotrimoxazole C. nitrofurantoin D. All of the above 694. A young couple present to their primary care physician stating that they 30 D. Pregnancy and lactation are the states of are trying to conceive. They would like to know if the future mom-to-be increased iron demands. While increased bleeding needs to be on any supplements. Along with recommending a multivitamin tendency, dietary deficiency, and malabsorption with folic acid, the doctor also suggests an iron supplement. Pregnant are all true causes of iron deficiency anemia, they women develop iron deficiency anemia because of A. Increased bleeding are not the culprits during pregnancy. Iron storage tendency B. Increased dietary deficiency C. Malabsorption D. Increased is regulated at the level of absorption, and very iron demands E. Increased excretion little of it is lost from the body. 695. A young, male patient presents with mental deterioration and tremors. He 694 B. Answer: B Wilson’s disease: excess has brown pigmentation in a ring surrounding the periphery of his cornea copper deposition (cornea = Kayser-Fleischer ring) and altered liver function tests. Which treatment modality should he receive? A. Dimercaprol B. Penicillamine C. EDTA D. Deferoxamine 696. You prescribe a cancer patient who is undergoing chemotherapy with 265 A. Azathioprine/Mercaptopurine metabolism is allopurinol to address hyperuricemia. Upon reviewing his chart, you realize inhibited by allopurinol. caution is needed because of risk of toxicity. Which of the following chemotherapeutic drus is most likely being given to the patient? A. Azathioprine B. Methotrexate C. Fluorouracil D. Cytarabine E. All of the above 697. Your patient is in cardiogenic shock. Which of the following is the drug of 110 E Page 15 of Topnotch Handout. In cardiogenic choice to increase cardiac contractility without the vasoconstricting shock the DOC is Dobutamine. It is a selective Beta effects? A. Norepinephrine B. Dopamine C. Epinephrine D. Phenylephrine E. 1 Agonist hence it has no Alpha 1 vasoconstricting Dobutamine effects. 698. Your patient requires a local anesthetic with long duration of action. What 183 D. SIMILAR TO PREVIOUS BOARD EXAM will you give? A. procaine B. lidocaine C. cocaine D. bupivacaine E. None of CONCEPT/PRINCIPLE.. my mnemonics for this: the above BeT-long. Local anesthetics with long duration of action are Bupivacaine and Tetracaine. The longest acting is Ropivacaine ("at the end of a long ROPe") 699. Zidovudine is a very remarkable drug for HIV. Howeve,r precaution for its 650 C. SIMILAR TO PREVIOUS BOARD EXAM use should be observed because of this side effect. A. pancreatitis B. CONCEPT/PRINCIPLE diarrhea C. myelosuppression D. Pulmonary fibrosis E. Optic neuritis 700. Zileuton is useful in the treatment of asthma because it A. Inhibits 25 B. By inhibiting 5-lipoxygenase, zileuton reduces prostaglandin biosynthesis B. Inhibits leukotriene synthesis C. Inhibits leukotriene biosynthesis; it does not inhibit (and in leukotriene receptors D. Inhibits 12-lipoxygenase E. None of the above. fact it might increase) prostaglandin synthesis.
Copyright © 2024 DOKUMEN.SITE Inc.